You are on page 1of 83

CMI M.

SC (MATHS) ENTRANCE EXAM SYLLABUS

Important note
The syllabus includes topics for PhD entrants too and so contains material which may often be
found only in MSc courses and not BSc courses in the country. Our policy generally has been to
have a common question paper for MSc and PhD levels but have separate cut-offs for them.

The Syllabus
Algebra.
(a) Groups, homomorphisms, cosets, Lagrange’s Theorem, group actions, Sylow Theorems,
symmetric group Sn , conjugacy class, rings, ideals, quotient by ideals, maximal and prime
ideals, fields, algebraic extensions, finite fields
(b) Matrices, determinants, vector spaces, linear transformations, span, linear independence,
basis, dimension, rank of a matrix, characteristic polynomial, eigenvalues, eigenvectors,
upper triangulation, diagonalization, nilpotent matrices, scalar (dot) products, angle, rota-
tions, orthogonal matrices, GLn , SLn , On , SO2 , SO3 .
References:
(i) Algebra, M. Artin
(ii) Topics in Algebra, Herstein
(iii) Basic Algebra, Jacobson
(iv) Abstract Algebra, Dummit and Foote
Complex Analysis.
Holomorphic functions, Cauchy-Riemann equations, integration, zeroes of analytic functions, Cauchy
formulas, maximum modulus theorem, open mapping theorem, Louville’s theorem, poles and sin-
gularities, residues and contour integration, conformal maps, Rouche’s theorem, Morera’s theorem
References:
(i) Functions of one complex variable, John Conway
(ii) Complex Analysis, L V Ahlfors
(iii) Complex Analysis, J Bak and D J Newman
Calculus and Real Analysis.
(a) Real Line: Limits, continuity, differentiablity, Reimann integration, sequences, series, lim-
sup, liminf, pointwise and uniform convergence, uniform continuity, Taylor expansions,
(b) Multivariable: Limits, continuity, partial derivatives, chain rule, directional derivatives,
total derivative, Jacobian, gradient, line integrals, surface integrals, vector fields, curl, di-
vergence, Stoke’s theorem
(c) General: Metric spaces, Heine Borel theorem, Cauchy sequences, completeness, Weierstrass
approximation.
References:
(i) Principles of mathematical analysis, Rudin
(ii) Real Analysis, Royden
(iii) Calculus, Apostol
Topology. Topological spaces, base of open sets, product topology, accumulation points, bound-
ary, continuity, connectedness, path connectedness, compactness, Hausdorff spaces, normal spaces,
Urysohn’s lemma, Tietze extension, Tychonoff’s theorem,
References: Topology, James Munkres
CHENNAI Mathematical Institute
Graduate Programme in Mathematics
Entrance Examination, 2010
Part A
State whether True or False and give brief reasons in the sheets provided (e.g., if you feel
that a statement is “False” then give a counter-example). Marks will be given only when
reasons are provided.

1. Suppose A is an m × n matrix, V an m × 1 matrix, with both A and V having rational


entries. If the equation AX = V has a solution in Rn , then the equation has a solution
with rational entries. (Here and in Question 5 below of Part A, Rn is identified with
the space of n × 1 real matrices.)

2. A closed and bounded subset of a complete metric space is compact.

3. Let p be a prime number. If P is a p-Sylow subgroup of some finite group G, then for
every subgroup H of G, H ∩ P is a p-Sylow subgroup of H.

4. There exists a real 3 × 3 orthogonal matrix with only non-zero entries.

5. A 5 × 5 real matrix has an eigenvector in R5 .

6. A continuous function on Q ∩ [0, 1] can be extended to a continuous function on [0, 1].

7. Let f : R → R be a differentiable function. Then f 0 (x) is continuous.

8. There is a continuous onto function from the unit sphere in R3 to the complex plane
C.

9. f : C → C is an entire function such that the function g(z) given by g(z) = f ( z1 ) has
a pole at 0. Then f is a surjective map.

10. Every finite group of order 17 is abelian.

11. Let n ≥ 2 be an integer. Given an integer k there exists an n × n matrix A with integer
entries such that det A = k and the first row of A is (1, 2, . . . , n).

12. There is a finite Galois extension of R whose Galois group is nonabelian.

1
13. There is a non-constant continuous function from the open unit disc

D = {z ∈ C | |z| < 1}

to R which takes only irrational values.

14. There is a field of order 121.

2
Part B
Answer all questions.

1. Let α, β be two complex numbers with β 6= 0, and f (z) a polynomial function on


C such that f (z) = α whenever z 5 = β. What can you say about the degree of the
polynomial f (z)?

2. Let f, g : Z/5Z → S5 be two non-trivial group homomorphisms. Show that there is a


σ ∈ S5 such that f (x) = σg(x)σ −1 , for every x ∈ Z/5Z.

3. Suppose f is continuous on [0, ∞), differentiable on (0, ∞) and f (0) ≥ 0. Suppose


f 0 (x) ≥ f (x) for all x ∈ (0, ∞). Show that f (x) ≥ 0 for all x ∈ (0, ∞).

4. A linear transformation T : R8 → R8 is defined on the standard basis e1 , . . . , e8 by

T ej = ej+1 j = 1, . . . , 5
T e6 = e7
T e7 = e6
T e8 = e2 + e4 + e6 + e8 .

What is the nullity of T ?

5. If f and g are continuous functions on [0, 1] satisfying f (x) ≥ g(x) for every 0 ≤ x ≤ 1,
R1 R1
and if 0 f (x)dx = 0 g(x)dx, then show that f = g.

6. Let {an } and {bn } be sequences of complex numbers such that each an is non-zero,
limn→∞ an = limn→∞ bn = 0, and such that for every natural number k,
bn
lim = 0.
n→∞ ak
n

Suppose f is an analytic function on a connected open subset U of C which contains


0 and all the an . Show that if f (an ) = bn for every natural number n, then bn = 0 for
every natural number n.

7. Let T : R3 → R3 be an orthogonal transformation such that det T = 1 and T is not


the identity linear transformation. Let S ⊂ R3 be the unit sphere, i.e.,

S = {(x, y, z) | x2 + y 2 + z 2 = 1}.

Show that T fixes exactly two points on S.

3
8. Compute

x1/3
Z
dx.
0 1 + x2

9. Let f (x) = xn + an−1 xn−1 + · · · + a0 be a polynomial with integer coefficients and whose
degree is at least 2. Suppose each ai (0 ≤ i ≤ n − 1) is of the form

17!
ai = ±
r!(17 − r)!

with 1 ≤ r ≤ 16. Show that f (m) is not equal to zero for any integer m.

10. Suppose ϕ = (ϕ2 , . . . , ϕn ) : Rn → Rn−1 is a C 2 function, i.e. all second order partial
derivatives of the ϕi exist and are continuous. Show that the symbolic determinant
∂ ∂ϕ2 ∂ϕn
∂x1 ∂x1
... ∂x1
.. .. ..
. . .
∂ ∂ϕ2 ∂ϕn
∂xn ∂xn
... ∂xn

vanishes indentically.

4
CHENNAI MATHEMATICAL INSTITUTE
Graduate Programme in Mathematics
Entrance Examination, 2011
Part A
State whether True or False and give brief reasons. Marks will be given only when reasons are
provided. Answer any 10 questions in this part. All questions carry 5 marks.

T∞
1. There is a sequence of open intervals In ⊂ R such that n=1 In = [0, 1].
m
2. The set S of real numbers of the form with m, n ∈ Z and n ≥ 0 is a dense subset of R.
10n
3. There is a continuous bijection from R2 → R.

4. There is a bijection between Q and Q × Q.

5. If {an }∞ ∞
n=1 , {bn }n=1 are two sequences of positive real numbers with the first converging to
zero, and the second diverging to ∞, then the sequence of complex numbers cn = an eibn also
converges to zero.

6. For any polynomial f (x) with real coefficients and of degree 2011, there is a real number b
such that f (b) = f 0 (b).

7. If f : [0, 1] → [−π, π] is a continuous bijection then it is a homeomorphism.

8. For any n ≥ 2 there is an n × n matrix A with real entries such that A2 = A and trace (A) =
n + 1.

9. There is 2 × 2 real matrix with characteristic polynomial x2 + 1.

10. There is a field with 10 elements.

11. There are at least three non-isomorphic rings with 4 elements.

12. The group (Q, +) is a finitely generated abelian group.


√ √
13. Q( 7) and Q( 17) are isomorphic as fields.

14. A vector space of dimension ≥ 2 can be expressed as a union of two proper subspaces.

15. There is a bijective analytic function from the complex plane to the upper half-plane.

16. There is a non-constant bounded analytic function on C \ {0}.


Part B
Answer any five questions. All questions carry 10 marks

1. (a) Consider the ring R of polynomials in n variables with integer coefficients. Prove that
the polynomial f (x1 , x2 , . . . , xn ) = x1 x2 · · · xn has 2n+1 − 2 non-constant polynomials in
R dividing it.
(b) Let p1 , p2 , . . . , pn be distinct prime numbers. Then show that the number N = p1 p22 p33 · · · pnn
has (n + 1)! positive divisors.

2. Let f (x) = (x2 − 2)(x2 − 3)(x2 − 6). For every prime number p, show that f (x) ≡ 0 (mod p)
has a solution in Z.

3. Let S denote the group of all those permutations of the English alphabet that fix the letters
T,E,N,D,U,L,K,A and R. Other letters may or may not be fixed. Show that S has elements
σ, τ of order 36 and 39 respectively, but does not have any element of order 37 or 38.

4. Show that there are at least two non-isomorphic groups of order 198. Show that in all those
groups the number of elements of order 11 is the same.

5. Suppose f, g, p
h are functions from the set of positive real numbers into itself satisfying
f (x)g(y) = h( x2 + y 2 ) for all x, y ∈ (0, ∞). Show that the three functions f (x)/g(x), g(x)/h(x),
and h(x)/f (x) are all constant.

6. Let a, b > 0.

(a) Prove that limn→∞ (an + bn )1/n = max{a, b}.


(b) Define a sequence by x1 = a, x2 = b and xn = 21 (xn−1 + xn−2 ) for n > 2. Show that
{xn } is a convergent sequence.

7. Let f : C → C be an entire function withP


the following property: In the power series expan-
sion around any a ∈ C, given as f (z) = ∞ n
n=o cn (a)(z − a) , the coefficient cn (a) is zero for
some n ( with n depending on a). Show that f (z) is in fact a polynomial.

8. (a) Show that in a Hausdorff topological space any compact set is closed.
(b) If (X, d1 ) and (Y, d2 ) are two metric spaces that are homeomorphic then does complete-
ness of (X, d1 ) imply the completeness of (Y, d2 )? Give reasons for your answer.

9. Fix an integer n > 1. Show that there is a real n × n diagonal matrix D such that the
condition AD = DA is valid only for a diagonal matrix A.
CHENNAI MATHEMATICAL INSTITUTE
Graduate Programme in Mathematics - M.Sc./Ph.D.
Entrance Examination, 2012
100
Part A

State whether True or False and give brief reasons in the sheets provided. Marks will be
given only when reasons are provided. Try to answer 10 questions. Each question carries 5
marks.

1. The function f : Rn → R, defined as f (x1 , · · · , xn ) = M ax{|xi |}, i = 1, · · · , n, is


uniformly continuous.

2. Let xn be a sequence with the following property: Every subsequence of xn has a


further subsequence which converges to x. Then the sequence xn converges to x.

3. Let f : (0, ∞) −→ R be a continuous function. Then f maps any Cauchy sequence to


a Cauchy sequence.

4. Let {fn : R −→ R} be a sequence of continuous functions. Let xn −→ x be a convergent


sequence of reals. If fn −→ f uniformly then fn (xn ) −→ f (x).

5. Let K ⊂ Rn such that every real valued continuous function on K is bounded. Then
K is compact (i.e closed and bounded).

6. If A ⊂ R2 is a countable set, then R2 \ A is connected.

7. The set A = {(z, w) ∈ C2 | z 2 + w2 = 1} is bounded in C2 .

8. Let f, g : C −→ C be complex analytic, and let h : [0, 1] −→ C be a non-constant


continuous map. Suppose f (z) = g(z) for every z ∈ Im h, then f = g. (Here Im h
denotes the image of the function h.)

9. There is a field with 121 elements.


!
π π
10. The matrix 22 is diagonalizable over C.
0 7

11. There are no infinite group with subgroups of index 5.

12. Every finite group of odd order is isomorphic to a subgroup of An , the group of all
even permutations.

13. Every group of order 6 abelian.

1
14. Two abelian groups of the same order are isomorphic.

15. There is a non-constant continuous function f : R → R whose image is contained in Q.

2
Part B
Each question carries 10 marks. Try to answer 5 questions.

1. Suppose f : R 7→ Rn be a differentiable mapping satisfying kf (t)k = 1 for all t ∈ R.


Show that hf 0 (t), f (t)i = 0 for all t ∈ R. (Here k.k denotes standard norm or length
of a vector in Rn , and h., .i denotes the standard inner product (or scalar product) in
Rn .)

2. Let A, B ⊂ Rn and define A + B = {a + b; a ∈ A, b ∈ B}. If A and B are open, is


A + B open? If A and B are closed, is A + B closed? Justify your answers.

3. Let f : X →
7 Y be continuous map onto Y , and let X be compact. Also g : Y 7→ Z is
such that g ◦ f is continuous. Show g is continuous.

4. Let A be a n × m matrix with real entries, and let B = AAt and let α be the supremum
of xt Bx where supremum is taken over all vectors x ∈ Rn with norm less than or equal
to 1. Consider
k
Bj .
X
Ck = I +
j=1

Show that the sequence of matrices Ck converges if and only if α < 1.

5. Show that a power series n≥0 an z n where an → 0 as n → ∞ cannot have a pole on the
P

unit circle. Is the statement true with the hypothesis that (an ) is a bounded sequence?

6. Show that a biholomorphic map of the unit ball onto itself which fixes the origin is
necessarily a rotation.

7. (i) Let G = GL(2, Fp ). Prove that there is a Sylow p−subgroup H of G whose normal-
izer NG (H) is the group of all upper triangular matrices in G.

(ii) Hence prove that the number of Sylow subgroups of G is 1 + p.


√ 2πi
8. Calculate the minimal polynomial of 2e 3 over Q.

9. Let G be a group F a field and n a positive integer. A linear action of G on Fn is


a map α : G × Fn → Fn such that α(g, v) = ρ(g)v for some group homomorphism
ρ : G → GLn (F). Show that for every finite group G, there is an n such that there is
a linear action α of G on Fn and such that there is a nonzero vector v ∈ Fn such that
α(g, v) = v for all g ∈ G.

10. Let R be an integral domain containing a field F as a subring. Show that if R is a


finite-dimensional vector space over F , then R is a field.

3
Chennai Mathematical Institute
MSc/PhD Entrance Examination, 2013
15th May 2013

Problems in Part A will be used for screening purposes. Your solutions to the questions in Part B will
be marked only if your score in Part A places you over the cut-off. However, the scores in both the sections
will be taken into account to decide whether you qualify for the interview.
Notation: Z, R, and C stand, respectively, for the sets of integers, of the real numbers, and of the complex
numbers.

Part A
This section consists of fifteen (15) multiple-choice questions, each with one or more correct answers. Record
your answers on the attached bubble-sheet by filling in the appropriate circles. Every question is worth four
(4) marks. A solution receives credit if and only if all the correct answers are chosen, and no incorrect answer
is chosen.
1. Pick the correct statement(s) below.
(a) There exists a group of order 44 with a subgroup isomorphic to Z/2 ⊕ Z/2.
(b) There exists a group of order 44 with a subgroup isomorphic to Z/4.
(c) There exists a group of order 44 with a subgroup isomorphic to Z/2⊕Z/2 and a subgroup isomorphic
to Z/4.
(d) There exists a group of order 44 without any subgroup isomorphic to Z/2 ⊕ Z/2 or to Z/4.
2. Let G be group. The following statements hold.
(a) If G has nontrivial centre C, then G/C has trivial centre.
(b) If G 6= 1, there exists a nontrivial homomorphism h : Z → G.
(c) If |G| = p3 , for p a prime, then G is abelian.
(d) If G is nonabelian, then it has a nontrivial automorphism.

3. Let C[0, 1] be the space of continuous real-valued functions on the interval [0, 1]. This is a ring under
point-wise addition and multiplication. The following are true.
(a) For any x ∈ [0, 1], the ideal M (x) = {f ∈ C[0, 1] | f (x) = 0} is maximal.
(b) C[0, 1] is an integral domain.
(c) The group of units of C[0, 1] is cyclic.
(d) The linear functions form a vector-space basis of C[0, 1] over R.

1
2 9
4. Let A : R2 → R2 be a linear transformation with eigenvalues 3 and 5. Then, there exists a non-zero
vector v ∈ R2 such that
(a) kAvk > 2kvk;
(b) kAvk < 12 kvk;
(c) kAvk = kvk;
(d) Av = 0;
5. Let F be a field with 256 elements, and f ∈ F [x] a polynomial with all its roots in F . Then,
(a) f 6= x15 − 1;
(b) f 6= x63 − 1;
(c) f 6= x2 + x + 1;
(d) if f has no multiple roots, then f is a factor of x256 − x.
6. Let h : C → C be an analytic function such that h(0) = 0; h( 12 ) = 5, and |h(z)| < 10 for |z| < 1. Then,
(a) the set {z : |h(z)| = 5} is unbounded by the Maximum Principle;
(b) the set {z : |h0 (z)| = 5} is a circle of strictly positive radius;
(c) h(1) = 10;
(d) regardless of what h0 is, h00 ≡ 0.

7. Suppose that f (z) is analytic, and satisfies the condition |f (z)2 − 1| = |f (z) − 1| · |f (z) + 1| < 1 on a
non-empty connected open set U . Then,
(a) f is constant.
(b) The imaginary part of f , Im(f ), is positive on U .
(c) The real part of f , Re(f ), is non-zero on U .
(d) Re(f ) is of fixed sign on U .
8. Consider the following subsets of R2 : X1 = {(x, sin x1 )|0 < x < 1}, X2 = [0, 1] × {0}, and X3 = {(0, 1)}.
Then,
(a) X1 ∪ X2 ∪ X3 is a connected set;
(b) X1 ∪ X2 ∪ X3 is a path-connected set;
(c) X1 ∪ X2 ∪ X3 is not path-connected, but X1 ∪ X2 is path-connected;
(d) X1 ∪ X2 is not path-connected, but every open neighbourhood of a point in this set contains a
smaller open neighbourhood which is path-connected.
9. For a set A ⊂ R, denote by Cl(A) the closure of A, and by Int(A) the interior of A. There is a set
A ⊂ R such that
(a) A, Cl(A), and Int(A) are pairwise distinct;
(b) A, Cl(A), Int(A), and Cl(Int(A)) are pairwise distinct;
(c) A, Cl(A), Int(A), and Int(Cl(A)) are pairwise distinct;
(d) A, Cl(A), Int(A), Int(Cl(A)), and Cl(Int(A)) are pairwise distinct.

Page 2
10. Let f, g : [0, 1] → R be given by

x2

if x is rational,
f (x) :=
0 if x is irrational;

if x = pq is rational, with gcd(p, q) = 1,



1/q
g(x) :=
0 if x is irrational.
Then,
(a) g is Riemann integrable, but not f ;
(b) both f and g are Riemann integrable;
R1
(c) the Riemann integral 0 f (x)dx = 0;
R1
(d) the Riemann integral 0 g(x)dx = 0.
11. Let C be the ellipse 24x2 + xy + 5y 2 + 3x + 2y + 1 = 0. Then, the line integral (x2 ydy + xy 2 dx)
H

(a) lies in (0, 1);


(b) is 1;
(c) is either 1 or −1 depending on whether C is traversed clockwise or counterclockwise;
(d) is 0.
P∞ 2
12. The series n=1 an where an = (−1)n+1 n4 e−n
(a) has unbounded partial sums;
(b) is absolutely convergent;
(c) is convergent but not absolutely convergent;
(d) is not convergent, but partial sums oscillate between −1 and +1.
13. Let f be continuously differentiable on R. Let fn (x) = n f (x + n1 ) − f (x) . Then,


(a) fn converges uniformly on R;


(b) fn converges on R, but not necessarily uniformly;
(c) fn converges to the derivative of f uniformly on [0, 1];
(d) there is no guarantee that fn converges on any open interval.
14. Let f : X → Y be a nonconstant continuous map of topological spaces. Which of the following statements
are true?
(a) If Y = R and X is connected then X is uncountable.
(b) If X is Hausdorff then f (X) is Hausdorff.
(c) If X is compact then f (X) is compact.
(d) If X is connected then f (X) is connected.
15. Let X be a set with the property that for any two metrics d1 , and d2 on X, the identity map

id : (X, d1 ) → (X, d2 )

is continuous. Which of the following are true?


(a) X must be a singleton.
(b) X can be any finite set.
(c) X cannot be infinite.
(d) X may be infinite but not uncountable.

Page 3
Part B
Solve six (6) problems from below, clearly indicating which problems you would like us to mark. Every
problem is worth ten (10) marks. Justify all your arguments to receive credit.
1. Let G be a finite group, p the smallest prime divisor of |G|, and x ∈ G an element of order p. Suppose
h ∈ G is such that hxh−1 = x10 . Show that p = 3.

2. (a) Show that there exists a 3 × 3 invertible matrix M 6= I3 with entries in the field F2 such that
M 7 = I3 .
(b) Let A be an m × n matrix, and b an m × 1 vector, both with integer entries.
1. Suppose that there exists a prime number p such that the equation Ax = b seen as an equation
over the finite field Fp has a solution. Then does there exist a solution to Ax = b over the real
numbers?
2. If Ax = b has a solution over Fp for every prime p, is a real solution guaranteed?

3. Let Mn (C) denote the set of n × n matrices over C. Think of Mn (C) as the 2n2 -dimensional Euclidean
2
space R2n . Show that the set of all diagonalizable n × n matrices is dense in Mn (C).
4. Compute the integral Z ∞
x
dx.
−∞ (x2 + 2x + 2)(x2 + 4)

5. Show that there does not exists an analytic function f defined in open unit disk for which f ( n1 ) is 2−n .

6. Let f be a real valued continuous function on [0, 2] which is differentiable at every point except possibly
at x = 1. Suppose that limx→1 f 0 (x) = 2013. Show that f is differentiable at x.
7. (a) Show that there exists no bijective map f : R2 → R3 such that f and f −1 are differentiable.
(b) Let f : Rm → Rn be a differentiable map such that the derivative Df (x) is surjective for all x. Is f
surjective?
8. (a) Let f ∈ Z[x] be a non-constant polynomial with integer coefficients. Show that as a varies over the
integers, the set of divisors of f (a) includes infinitely many different primes.
(b) Assume known the following result: If G is a finite group of order n such that for integer d > 0,
d|n, there is no more than one subgroup of G of order d, then G is cyclic. Using this (or otherwise)
prove that the multiplicative group of units in any finite field is cyclic.
9. Let K1 ⊃ K2 ⊃ . . . be a sequence of connected compact subsets of R2 . Is it true that their intersection
K = ∩∞i=1 Ki is connected also? Provide either a proof or a counterexample.

10. Let A be a subset of R2 with the property that every continuous function f : A → R has a maximum in
A. Prove that A is compact.

Page 4
CHENNAI MATHEMATICAL INSTITUTE
Postgraduate Programme in Mathematics
MSc/PhD Entrance Examination
15 May 2014
Important: Questions in Part A will be used for screening. Your solutions to the questions
in Part B will be marked only if your score in Part A places you over the cut-off. However, the
scores in both the sections will be taken into account while making the final decision.
For qualifying for the PhD interview, you should answer at least two (2) from
among the starred questions (17∗ )–(20∗ ) in Part B.

Notation: N, Q, R and C stand, respectively, for the sets of the natural numbers, of the
rational numbers, of the real numbers, and of the complex numbers. For a field F , Mm×n (F )
stands for the set of m × n matrices
qP over F . We treat Mm×n (R) and Mm×n (C) as metric spaces
2
with the metric d(A, B) = i,j |aij − bij | where A = (aij ) and B = (bij ).

Part A
Instructions: Each of the questions 1–8 has one or more correct answers. Record your
answers on the attached bubble-sheet by filling in the appropriate circles. Every question is
worth four (4) marks. A solution receives credit if and only if all the correct answers are chosen,
and no incorrect answer is chosen.

(1) Let f : R → R be a function such that f (x + 1) = f (x) for all x ∈ R. Which of the
following statement(s) is/are true?
(A) f is bounded.
(B) f is bounded if it is continuous.
(C) f is differentiable if it is continuous.
(D) f is uniformly continuous if it is continuous.

(2) Let W ⊂ Rn be a linear subspace of dimension at most n − 1. Which of the following


statement(s) is/are true?
(A) W is nowhere dense.
(B) W is closed.
(C) Rn \ W is connected.
(D) Rn \ W is not connected.

(3) Let G be a finite group. An element a ∈ G is called a square if there exists x ∈ G such
that x2 = a. Which of the following statement(s) is/are true?
(A) If a, b ∈ G are not squares, ab is a square.
(B) Suppose that G is cyclic. Then if a, b ∈ G are not squares, ab is a square.
(C) G has a normal subgroup.
(D) If every proper subgroup of G is cyclic then G is cyclic.

(4) Let A ∈ Mm×n (R) and let b0 ∈ Rm . Suppose the system of equations Ax = b0 has a
unique solution. Which of the following statement(s) is/are true?
(A) Ax = b has a solution for every b ∈ Rm .
(B) If Ax = b has a solution then it is unique.
(C) Ax = 0 has a unique solution.
(D) A has rank m.
1
(5) Let A ∈ Mn×n (C). Which of the following statement(s) is/are true?
(A) There exists B ∈ Mn×n (C) such that B 2 = A.
(B) A is diagonalizable.
(C) There exists an invertible matrix P such that P AP −1 is upper-triangular.
(D) A has an eigenvalue.

(6) Let f : C → C be a function. Which of the following statement(s) is/are true?


∂fi
(A) Consider f as a function (f1 , f2 ) : R2 → R2 . Suppose that for i = 1, 2, both ∂X
and ∂f i
∂Y exist and are continuous. Then f is entire.
(B) Assume that f is entire and |f (z)| < 1 for all z ∈ C. Then f is constant.
(C) Assume that f is entire and Im(f (z)) > 0 for all z ∈ C. Then f is constant.

(7) Let C(R) be the R-vector space of continuous functions from R to R. Let a1 , a2 , a3 be
distinct real numbers. For i = 1, 2, 3, let fi ∈ C(R) be the function fi (t) = eai t . Which
of the following statement(s) is/are true?
(A) f1 , f2 and f3 are linearly independent.
(B) f1 , f2 and f3 are linearly dependent.
(C) f1 , f2 and f3 form a basis of C(R).

(8) Which of the following statement(s) is/are true?


∞ 2
e−n converges.
P
(A) The series
n=1

P (−1)n
(B) The series n converges.
n=1

P (−1)n
(C) The series n converges absolutely.
n=1

P sin(nx)
(D) The series n2
converges uniformly on R.
n=1

Instructions: The answers to questions 9 and 10 are integers. You are required to
write the answers in decimal form in the attached bubble-sheet. Every question is worth
four (4) marks.

(9) What is the dimension of the ring Q[x]/((x + 1)2 ) as a Q-vector space?
 Pn 
π i=1 sin( iπ
n )
(10) Evaluate lim n .
n→∞

2
Part B
Instructions: Answer six (6) questions from below. Provide sufficient justification.
Write your solutions on the page assigned to each question. Each of the questions is worth
ten (10) marks. For qualifying for the PhD interview, you should answer at
least two (2) from among starred questions (17∗ )–(20∗ ). Clearly indicate which
six questions you would like us to mark in the six boxes in the bubble sheet. If the
boxes are unfilled, we will mark the first six solutions that appear in your answer-sheet.
If you do not want a solution to be considered, clearly strike it out.

(11) Show that the set of rank two matrices in M2×3 (R) is open.
(12) (A) Let F be a finite field extension of Q. Show that any field homomorphism φ : F → F
is an isomorphism. (Note that φ(1) = 1 by definition.)
(B) Let F be a finite field whose characteristic is not 2. Let F × denote the multiplicative
group of nonzero elements of F . An element a ∈ F × is called a square if there exists
x ∈ F × such that x2 = a. Show that exactly half the elements F × are squares.
(13) Let n ∈ N. Show that the determinant map det : Mn×n (R) → R is infinitely differen-
tiable and compute the total derivative d(det) at every point A ∈ Mn×n (R). Find a
necessary and sufficient condition on the rank of A for d(det) = 0 at A.
nP o
(14) Let ai , i ∈ R be non-negative real numbers such that sup ai | F ⊆ R a finite subset
i∈F
is finite. Show that ai = 0 except for countably many i ∈ R. Give an example to show
that ‘countably’ cannot be replaced by ‘finite’. (Hint: consider Fn := {i | ai ≥ n1 }.)
(15) Let G be a finite group of order 2n for some integer n. Consider the map φ : G → G
given by φ(a) = a2 . Show that φ is not surjective.
(16) Let f : C → C be an entire function.
(A) Construct a sequence {zn } in C such that |zn | → ∞ and ezn → 1.
(B) Show that the function g(z) = f (ez ) is not a polynomial.
(17∗ ) For F = R and F = C, let On (F ) = {A ∈ Mn×n (F ) | AAt = In }.
(A) Show that On (R) is compact.
(B) Is On (R) connected? Justify.
(C) Is On (C) compact? Justify.
(18∗ ) Let Ω be a region in C. Let {an } be a sequence of nonzero elements in Ω such that
an → 0 as n → ∞. Let {bn } be a sequence of complex numbers such that lim abnk = 0
n→∞ n
for every nonnegative integer k. Suppose that f : Ω → C is an entire function such that
f (an ) = bn for all n. Show that bn = 0 for every n.
(19∗ ) Let G be a finite group of order n and let H be a subgroup of G of order m. Assume
n
that ( m )! < 2n. Show that G is not simple, that is: G has a nontrivial proper normal
subgroup. (Hint: Think along the lines of Cayley’s theorem.)
(20∗ ) Let
C0 (R) = {f : R → R | f is continuous , lim |f (x)| = 0 and lim |f (x)| = 0}, and
x→∞ x→−∞
C0∞ (R) = {f ∈ C0 (R) | f is infinitely differentiable}.
Let φ ∈ C0 (R). For f ∈ C0 (R), define φ∗ (f ) = f ◦ φ.
(A) Show that φ∗ (f ) ∈ C0 (R) if f ∈ C0 (R).
(B) If φ∗ (C0∞ (R)) ⊆ C0∞ (R), then show that φ is infinitely differentiable.
3
CHENNAI MATHEMATICAL INSTITUTE
Postgraduate Programme in Mathematics
MSc/PhD Entrance Examination
18 May 2015
Important: Questions in Part A will be used for screening. There will be a cut-off for Part A,
which will not be more than twenty (20) marks (out of 40). Your solutions to the questions in
Part B will be marked only if your score in Part A places you over the cut-off. (In particular, if
your score in Part A is at least 20 then your solutions to the questions in Part B will be marked.)
However, note that the scores in both the sections will be taken into account while making the
final decision. In order to qualify for the PhD Mathematics interview, you must
obtain at least fifteen (15) marks from among the starred questions (17∗ )–(20∗ ).

Notation: N, Z, Q, R and C stand, respectively, for the sets of the natural numbers, of the
integers, of the rational numbers, of the real numbers, and of the complex numbers. For a
prime number p, Fp is the field with p elements. For a field F , Mm×n (F ) stands for the set of
m × n matrices over F and GLn (F ) is the set of invertible n × n matrices
qPover F . If F = R or
2
F = C, we treat these sets as metric spaces with the metric d(A, B) = i,j |aij − bij | where
A = (aij ) and B = (bij ).

Part A
Instructions: Each of the questions 1–7 has one or more correct answers. Record your
answers on the attached bubble-sheet by filling in the appropriate circles. Every question is
worth four (4) marks. A solution receives credit if and only if all the correct answers are chosen,
and no incorrect answer is chosen.

(1) Which of the following topological spaces is/are connected?


(A) GL1 (R)
(B) GL1 (C)
(C) 
GL2 (R)  
x −y
(D) : x, y ∈ R, x2 + y 2 = 1
y x

(2) Consider f : {z ∈ C : |z| > 1} −→ C, f (z) = z1 . Choose the correct statement(s):


(A) There are infinitely many entire functions g such that g(z) = f (z) for every z ∈ C
with |z| > 1.
(B) There does not exist an entire function g such that g(z) = f (z) for every z ∈ C
with |z| > 1.
(C) g : C −→ C with (
1 − 1 z 2 , |z| ≤ 1
g(z) = 1 2
z, |z| > 1
is an entire function such that g(z) = f (z) for every z ∈ C with |z| > 1.

(3) Let
n  n 
a b 1 b
o o
G= : a, b ∈ R, a > 0 , N = : b ∈ R .
0 a−1 0 1
Which of the following are true?
(A) G/N is isomorphic to R under addition.
(B) G/N is isomorphic to {a ∈ R : a > 0} under multiplication.
(C) There is a proper normal subgroup N 0 of G which properly contains N .
(D) N is isomorphic to R under addition.
1
(4) Choose the correct statement(s):
(A) There is a continuous surjective function from [0, 1) to R;
(B) R and [0, 1) are homeomorphic to each other;
(C) There is a bijective function from [0, 1) to R;
(D) Bounded subspaces of R cannot be homeomorphic to R.

(5) Which of the following complex numbers has/have a prime number as the degree of its
minimal polynomial over Q?
(A) ζ√7 , a primitive
√ 7th root of unity;
(B) √2 + 3;
(C) √−1;
(D) 3 2.

(6) Let R be an integral domain such that every non-zero prime ideal of R[X] (where X is
an indeterminate) is maximal. Choose the correct statement(s):
(A) R is a field;
(B) R contains Z as a subring;
(C) Every ideal in R[X] is principal;
(D) R contains Fp as a subring for some prime number p.

R∞
(7) Let f : R −→ R be such that −∞ |f (x)|dx < ∞. Define F : R −→ R by F (x) =
Rx
−∞ f (t)dt. Choose the correct statement(s):
(A) f is continuous;
(B) F is continuous;
(C) F is uniformly continuous;
(D) There exists a positive real number M such that |f (x)| < M for all x ∈ R.

Instructions: The answers to questions 8–10 are integers. You are required to write
the answers in decimal form in the attached bubble-sheet. Every question is worth four
(4) marks.

(8) Let ω ∈ C be a primitive third root of unity. How many distinct possible images of ω
are there under all the field homomorphisms Q(ω) −→ C.

(9) Let C := {z ∈ C : |z| = 5}. What is value of M such that


Z
1
2πıM = 2
dz?
C z − 5z + 6

(10) Consider the set R[X] of polynomials in X with real coefficients as a real vector space.
Let T be the R-linear operator on R[X] given by
d2 f df
T (f ) = 2
− + f.
dX dX
What is the nullity of f ?
2
Part B
Instructions: Answer six (6) questions from below. Provide sufficient justification.
Write your solutions on the page assigned to each question. Each of the questions is worth
ten (10) marks. In order to qualify for the PhD Mathematics interview, you
must obtain at least fifteen (15) marks from among the starred questions
(17∗ )–(20∗ ). Clearly indicate which six questions you would like us to mark in the six
boxes in the bubble sheet. If the boxes are unfilled, we will mark the first six solutions
that appear in your answer-sheet. If you do not want a solution to be considered, clearly
strike it out.

(11) Let f ∈ R[x, y] be such that there exists a non-empty open set U ⊆ R2 such that
f (x, y) = 0 for every (x, y) ∈ U . Show that f = 0.

(12) Let A ∈ Mn×n (C).


(a) Suppose that A2 = 0. Show that λ is an eigenvalue of (In + A) if and only if
λ = 1. (In is the n × n identity matrix.)
(b) Suppose that A2 = −1. Determine (with proof) whether A is diagonalizable.

(13) Let f be a non-constant entire function satisfying the following conditions:


(a) f (0) = 0;
(b) For every positive real number M , the set {z : |f (z) < M } is connected.
Prove that f (z) = cz n for some constant c and positive integer n.

(14) Let (amn )m≥1,n≥1 be a double sequence of real numbers such that
(a) For every n, bn := lim amn exists;
m→∞
(b) For all strictly increasing sequences (mk )k≥1 and (nk )k≥1 of positive integers,
lim amk nk = 1.
k→∞
Show that the sequence (bn )n≥1 converges to 1.

(15) Let f ∈ C[x, y] be such that f (x, y) = f (y, x). Show that there is a g ∈ C[x, y] such
that f (x, y) = g(x + y, xy).

(16) Let X be a topological space and f : X −→ [0, 1] be a closed continuous surjective map
such that f −1 (a) is compact for every 0 ≤ a ≤ 1. Prove or disprove: X is compact. (A
map is said to be closed if it takes closed sets to closed sets.)

(17∗ ) Determine the cardinality of set of subrings of Q. (Hint: For a set P of positive prime
numbers, consider the smallest subring of Q that contains { p1 | p ∈ P }.)

(18∗ ) Let
X sin( x )
n
f (x) = .
n
n≥1
Show that f is continuous. Determine (with justification) whether f differentiable.

(19∗ ) Let m and n be positive integers and p a prime number. Let G ⊆ GLm (Fp ) be a subgroup
of order pn . Let U ⊆ GLm (Fp ) be the subgroup that consists of all the matrices with
1’s on the diagonal and 0’s below the diagonal. Show that there exists A ∈ GLm (Fp )
such that AGA−1 ⊆ U .

(20∗ ) Let m and n be positive integers and 0 ≤ k ≤ min{m, n} an integer. Prove or disprove:
The subspace of Mm×n (C) consisting of all matrices of rank equal to k is connected.
(You may use the following fact: For t ≥ 2, GLt (C) is connected.)
3
CHENNAI MATHEMATICAL INSTITUTE
Postgraduate Programme in Mathematics
MSc/PhD Entrance Examination
18 May 2016
Important: Questions in Part A will be used for screening. There will be a cut-off for Part A,
which will not be more than twenty (20) marks (out of 40). Your solutions to the questions in
Part B will be marked only if your score in Part A places you over the cut-off. (In particular, if
your score in Part A is at least 20 then your solutions to the questions in Part B will be marked.)
However, note that the scores in both the sections will be taken into account while making the
final decision. In order to qualify for the PhD Mathematics interview, you must
obtain at least fifteen (15) marks from among the starred questions (17∗ )–(20∗ ).

Notation: N, Z, Q, R and C stand, respectively, for the sets of natural numbers, of integers,
of rational numbers, of real numbers, and of complex numbers. For a prime number p, Fp is
the field with p elements. For a field F , Mm×n (F ) stands for the set of m × n matrices over F
and GLn (F ) is the set of invertible n × n matrices over F . The symbol ı denotes a square-root
of −1.

Part A
Instructions: Each of the questions 1–8 has one or more correct answers. Record your
answers on the attached bubble-sheet by filling in the appropriate circles. Every question is
worth four (4) marks. A solution receives credit if and only if all the correct answers are chosen,
and no incorrect answer is chosen.

(1) We say that two subsets X and Y of R are order-isomorphic if there is a bijective map
φ : X −→ Y such that for every x1 ≤ x2 ∈ X, φ(x1 ) ≤ φ(x2 ), where ‘≤’ denotes the
usual order on R. Choose the correct statement(s) from below:
(A) N and Z are not order-isomorphic;
(B) N and Q are order-isomorphic;
(C) Z and Q are order-isomorphic;
(D) The sets N, Z and Q are pairwise non-order-isomorphic.

(2) Let xn = 1 − n1 sin nπ



3 , n ≥ 1. Write l = lim inf xn and s = lim sup xn . Choose the
correct √statement(s) √from below:
(A) − 23 ≤ l < s ≤ 23 ;
(B) − 21 ≤ l < s ≤ 12 ;
(C) l = −1 and s = 1;
(D) l = s = 0.

(3) Let f : R −→ R be defined as


(
x2 sin( x12 ), if x 6= 0;
f (x) =
0, otherwise.
Choose the correct statement(s) from below:
(A) f is continuous;
(B) f is discontinuous at 0;
(C) f is differentiable;
(D) f is continuously differentiable.
1
(4) Let A ∈ Mm×n (R) be of rank m. Choose the correct statement(s) from below:
(A) The map Rn −→ Rm given by v 7→ Av is injective;  
(B) There exist matrices B ∈ Mm (R) and C ∈ Mn (R) such that BAC = Im | 0n−m ;
(C) There exist matrices B ∈ GLm (R) and C ∈ GLn (R) such that BAC = Im | 0n−m ;
(D) For every (B, C) ∈ Mm (R) × Mn (R) such that BAC = m I | 0 n−m , C is uniquely
determined by B.

(5) Let f : C −→ C be an entire function such that f (z + 1) = f (z + ı) = f (z) for every


z ∈ C. Choose the correct statement(s) from below:
(A) f is constant;
(B) f (z) = 0 for every z ∈ C;
(C) There exist complex numbers a, b such that for every x, y ∈ R, f (x+ıy) = a sin(x)+
ıb cos(y);
(D) f is not necessarily constant but |f (z)| is constant.

(6) What is the cardinality of the centre of O2 (R)? (Definition: The centre of a group G is
{g ∈ G | gh = hg for every h ∈ G}. Hint: Reflection matrices and permutation matrices
are orthogonal.)
(A) 1;
(B) 2;
(C) The cardinality of N;
(D) The cardinality of R.

(7) Let U ⊆ R be a non-empty open subset. Choose the correct statement(s) from below:
(A) U is uncountable;
(B) U contains a closed interval as a proper subset;
(C) U is a countable union of disjoint open intervals;
(D) U contains a convergent sequence of real numbers.

(8) Let R be a commutative ring. The characteristic of R is the smallest positive integer n
such that a + a + · · · + a (n times) is zero for every a ∈ R, if such an integer exists, and
zero, otherwise. Choose the correct statement(s) from below:
(A) For every n ∈ N, there exists a commutative ring whose characteristic is n;
(B) There exists a integral domain with characteristic 57;
(C) The characteristic of a field is either 0 or a prime number;
(D) For every prime number p, every commutative ring of characteristic p contains Fp
as a subring.

Instructions: The answers to questions 9–10 are integers. You are required to write
the answers in decimal form in the attached bubble-sheet. Every question is worth four
(4) marks.

(9) Consider the Q-vector-space


{f : R −→ R | f is continuous and Image(f ) ⊆ Q}.
What is its dimension?

(10) Let p be a prime number and F a field of p23 elements. Let φ : F −→ F be the field
automorphism of F sending a to ap . Let K := {a ∈ F | φ(a) = a}. What is the value
of [K : Fp ]?
2
Part B
Instructions: Answer six (6) questions from below. Provide sufficient justification.
Write your solutions on the page assigned to each question. Each of the questions is worth
ten (10) marks. In order to qualify for the PhD Mathematics interview, you
must obtain at least fifteen (15) marks from among the starred questions
(17∗ )–(20∗ ). Clearly indicate which six questions you would like us to mark in the six
boxes in the bubble sheet. If the boxes are unfilled, we will mark the first six solutions
that appear in your answer-sheet. If you do not want a solution to be considered, clearly
strike it out.

(11) Let U = {(x, y) ∈ R2 | 1 < x2 + y 2 < 4}. Let p, q ∈ U . Show that there is a continuous
map γ : [0, 1] −→ U such that γ(0) = p and γ(1) = q and such that γ is differentiable
on (0, 1).
(12) If I, J are two maximal ideals in a PID that is not a field, then show that IJ is never a
prime ideal.
(13) Let f : C −→ C be an entire function. Suppose that f (z) ∈ R if z is on the real axis or
on the imaginary axis. Show that f 0 (z) = 0 at z = 0.
(14) Let A ⊆ Rn be a closed proper subset. For x, y ∈ Rn , denote the usual (Euclidean)
distance between them by d(x, y). Let x ∈ Rn r A; define δ := inf{d(x, y) | y ∈ A}.
Show that there exists y ∈ A such that δ = d(x, y).
(15) Let F be a field and V a finite-dimensional vector-space over F . Let T : V −→ V be a
linear transformation, such that for every v ∈ V , there exists n ∈ N such that T n (v) = v.
(A) Show that if F = C, then T is diagonalizable.
(B) Show that if char(F ) > 0, then there exists a non-diagonalizable T satisfying the
above hypothesis.

(16) Let F = Q(ω, 3 2), where ω ∈ C is a primitive cube-root of unity. Find a Q-basis for
F (with proof). Let µ : F −→ F be the Q-linear map given by µ(a) = ω 2 a. Find the
matrix of µ with respect to the basis obtained above.
(17∗ ) Let G be a non-trivial subgroup of the group (R, +). Show that either G is dense in R
or that G = Z · l where l = inf{x ∈ G | x > 0}.
(18∗ ) Let G be a subgroup of the group of permutations on a finite set X. Let F be the
C-vector-space of all the functions from X to C. G acts on F by (g · f ) : x 7→ f (g −1 (x)).
Show that there is an φ ∈ F such that g · φ = φ for every g ∈ G. Show that there is a
subspace F 0 of F such that F = F 0 ⊕ Chφi and such that g · f ∈ F 0 for every g ∈ G and
f ∈ F 0.
(19∗ ) (A) Let A and B be n × n matrices with entries in N. Show that if B = A−1 then A
and B are permutation matrices. (A permutation matrix is a matrix obtained by
permuting the rows of the identity matrix.)
(B) Let A be an n × n complex matrix that is not a scalar multiple of In . Show that
A is similar to a matrix B such that B1,1 (i.e. the top left entry of B) is 0.

(20∗ ) Let S 1 = {z ∈ C : |z| = 1}. Consider the map Sq : S 1 −→ S 1 ,


Sq(z) = z 2 .
Show that there does not exist a continuous map Sqrt : S 1 −→ S 1 such that Sq ◦ Sqrt =
IdS 1 ? (That is, (Sqrt(w))2 = w.) (Hint: If such a map existed, show that there would
be a bijective continuous map S 1 × {1, −1} −→ S 1 .)
3
CHENNAI MATHEMATICAL INSTITUTE
Postgraduate Programme in Mathematics
MSc/PhD Entrance Examination
18 May 2017
Important: Questions in Part A will be used for screening. There will be a cut-off for Part A,
which will not be more than twenty (20) marks (out of 40). Your solutions to the questions in
Part B will be marked only if your score in Part A places you over the cut-off. (In particular, if
your score in Part A is at least 20 then your solutions to the questions in Part B will be marked.)
However, note that the scores in both the sections will be taken into account while making the
final decision. In order to qualify for the PhD Mathematics interview, you must
obtain at least fifteen (15) marks from among the starred questions (17∗ )–(20∗ ).

Notation: N, Z, Q, R and C stand, respectively, for the sets of non-negative integers, of


integers, of rational numbers, of real numbers, and of complex numbers. For a prime number
p, Fp is the field with p elements. For a field F , GLn (F ) is the set of invertible n × n matrices
over F . The symbol ı denotes a square-root of −1. When considered as a topological space, Rn
is taken with the euclidean topology.

Part A
Instructions: Each of the questions 1–9 has one or more correct answers. Record your
answers on the attached bubble-sheet by filling in the appropriate circles. Every question is
worth four (4) marks. A solution receives credit if and only if all the correct answers are chosen,
and no incorrect answer is chosen.

(1) Let G be a finite subgroup of GLn (k) where k is an algebraically closed field. Choose
the correct statement(s) from below:
(A) Every element of G is diagonalizable;
(B) Every element of G is diagonalizable if k is an algebraic closure of Q;
(C) Every element of G is diagonalizable if k is an algebraic closure of Fp ;
(D) There exists a basis of kn with respect to which every element of G is a diagonal
matrix.

(2) Consider the ideal I := (ux, uy, vx, uv) in the polynomial ring Q[u, v, x, y], where
u, v, x, y are indeterminates. Choose the correct statement(s) from below:
(A) Every prime ideal containing I contains the ideal (x, y);
(B) Every prime ideal containing I contains the ideal (x, y) or the ideal (u, v);
(C) Every maximal ideal containing I contains the ideal (u, v);
(D) Every maximal ideal containing I contains the ideal (u, v, x, y).

(3) Let f be an irreducible cubic polynomial over Q with at most one real root and k the
smallest subfield of C containing the roots of f . Choose the correct statement(s) from
below:
(A) σ(K) ⊆ K where σ denotes complex conjugation;
(B) [K : Q] is an even number;
(C) [(K ∩ R) : Q] is an even number;
(D) K is uncountable.

1
(4) For a positive integer n, let Sn denote the permutation group on n symbols. Choose the
correct statement(s) from below:
(A) For every positive integer n and for every m with 1 ≤ m ≤ n, Sn has a cyclic
subgroup of order m;
(B) For every positive integer n and for every m with n < m < n!, Sn has a cyclic
subgroup of order m;
(C) There exist positive integers n and m with n < m < n! such that Sn has a cyclic
subgroup of order m;
(D) For every positive integer n and for every group G of order n, G is isomorphic to
a subgroup of Sn .

(5) Let A = {(x, y) ∈ R2 | x2 + y 2 < 1} and B = {(x, y) ∈ R2 | 1 < x2 + y 2 < 2}, both
taken with the subspace topology of R2 . Choose the correct statement(s) from below:
(A) Every continuous function from A to R has bounded image;
(B) There exists a non-constant continuous function from B to N (in the subspace
topology of R);
(C) For every surjective continuous function from A ∪ B to a topological space X, X
has at most two connected components;
(D) B is homeomorphic to the unit circle.

(6) Let (X, d) be a metric space. Choose the correct statement(s) from below:
(A) There exists a metric d˜ on X such that d and d˜ define the same topology and such
that d˜ is bounded (i.e., there exists a real number M such that d(x,
˜ y) < M for all
x, y ∈ X.);
(B) Every closed subset of X that is bounded with respect to d is compact;
(C) X is connected;
(D) For every x ∈ X, there exists y ∈ X such that d(x, y) is a non-zero rational number.

(7) Which of the following are equivalence relations on R?


(A) a ∼ b if and only if |a − b| ≤ 25;
(B) a ∼ b if and only if a − b is rational;
(C) a ∼ b if and only if a − b is irrational;
(D) a ∼ b if and only if f (a) = f (b) for every continuous f : R −→ R.

2
(8) Let f, g : R2 −→ R be two differentiable functions such that f (x + 1, y) = f (x, y + 1) =
f (x, y) and g(x + 1, y) = g(x, y + 1) = g(x, y) for all (x, y) ∈ R2 . Choose the correct
statement(s) from below:
(A) f is uniformly continuous;
(B) f is bounded;
(C) The function (f, g) : R2 −→ R2 is differentiable;
(D) If ∂f /∂x = ∂g/∂y and ∂f /∂y = −∂g/∂x, then the function C −→ C sending
(x + ıy) −→ f (x, y) + ıg(x, y) (with x, y ∈ R) is constant.

(9) Consider the equation


1 1 1
+ = .
a b a+b
Choose the correct statement(s) from below:
(A) There exists (a, b) ∈ R2 satisfying the above equation;
(B) There exists (a, b) ∈ C2 satisfying the above equation;
(C) There exists (a, b) ∈ C2 with a = b satisfying the above equation;
(D) There exists (a, b) ∈ (F3 )2 with a = b satisfying the above equation.

Instructions: The answer to Question 10 is an integer. You are required to write


the answer in decimal form in the attached bubble-sheet. The question is worth four
(4) marks.

(10) Let p = (0, 0), q = (0, 1), r = (ı, 0) be points of C2 . What is the dimension of the
C-vector space
{f (X, Y ) ∈ C[X, Y ] | deg f ≤ 2 and f (p) = f (q) = f (r) = 0},
where by deg f , we mean the total degree of the polynomial f ?

3
Part B
Instructions: Answer six (6) questions from below. Provide sufficient justification.
Write your solutions on the page assigned to each question. Each of the questions is worth
ten (10) marks. In order to qualify for the PhD Mathematics interview, you
must obtain at least fifteen (15) marks from among the starred questions
(17∗ )–(20∗ ). Clearly indicate which six questions you would like us to mark in the six
boxes in the bubble sheet. If the boxes are unfilled, we will mark the first six solutions
that appear in your answer-sheet. If you do not want a solution to be considered, clearly
strike it out.

(11) Let (X, τ ) be a topological space and d : X × X −→ R≥0 a continuous function where
X×X has the product topology and R≥0 is the set of non-negative real numbers, with the
subspace topology of the usual topology of R. Assume that d−1 (0) = {(x, x) | x ∈ X},
and that d(x, y) ≤ d(x, z) + d(y, z) for all x, y, z ∈ X. Show the following:
(A) (X, τ ) is Hausdorff.
(B) The sets Bx, := {y ∈ X | d(x, y) < }, 0 <  ∈ R is the basis for a topology τ 0 on
X.
(C) τ 0 is coarser than τ (i.e., every set open in τ 0 is open in τ ).

(12) (A) Let f be an entire function such that |f (z)| ≤ |z|. Show that f is a polynomial of
degree ≤ 1.
(B) Let Γ be a closed differentiable contour oriented counterclockwise and let
Z
z dz = A.
Γ
What is the integral Z
(x + y) dz
Γ
(where x and y, respectively, are the real and imaginary parts of z) in terms of A?

(13) Let fn , f be real-valued functions on [0, 1] with f continuous. Suppose that for all
convergent sequences {xn : n ≥ 1} ⊆ [0, 1] with x = limn→∞ xn one has
lim fn (xn ) = f (x).
n→∞
Show that fn converges to f uniformly.

(14) (A) Show that for any positive rational number r, the sequence { log n
nr : n ≥ 1} is
bounded.
(B) Show that the series
X (log n)2 (log log n)
n2
n≥10
is convergent.

4
(15) For a group G, let Aut(G) denote the group of group automorphisms of G. (The
group operation of Aut(G) is composition.) Let p be prime number. Show that the
multiplicative group Fp r {0} is isomorphic to Aut((Fp , +)) under the map a 7→ [b 7→ ab]
(a ∈ Fp r {0}, b ∈ Fp ).

(16) Let k be a field, X an indeterminate and R = k[X]/(X 7 − 1). Determine the set
{dimk R/m | m is a maximal ideal in R}
in the following three cases: k = Q; k = C; k is a field of characteristic 7.

(17∗ ) For a 3 × 3 matrix A, say that a point p on the unit sphere centred at the origin in R3
is a pole of A if Ap = p. Denote by SO3 the subgroup of GL3 (R) consisting of all the
orthogonal matrices with determinant 1.
(A) Show that if A ∈ SO3 , then A has a pole.
(B) Let G be a subgroup of SO3 . Show that G acts on the set
{p ∈ S2 | p is a pole for some matrix A ∈ G}.

(18∗ ) Let f : X −→ Y be a continuous surjective map such that for every closed A ⊆ X, f (A)
is closed in Y . Show that if Y and all the fibres f −1 (y), y ∈ Y are compact, then X
is compact. Show that if Y is Hausdorff and X is compact, then Y and the all fibres
f −1 (y), y ∈ Y are compact.

(19∗ ) Let k be an algebraically closed uncountable field and m a maximal ideal in the polyno-
mial ring R := k[x1 , . . . , xn ] in the indeterminates x1 , . . . , xn . Show that the composite
map k −→ R −→ R/m is a field isomorphism. You may use without proof the following
fact from linear algebra: If a vector space has a countable spanning set, it cannot have
a linearly independent uncountable set in it. (Hint: If t is transcendental over k, then
1
consider the set { t−α | α ∈ k}.)

P∞ sin(z/n)
(20∗ ) Prove that for every z ∈ C, the series n=1 n converges. For z ∈ C, let f (z) =
P∞ sin(z/n)
n=1 n . Prove that f is entire.

5
CHENNAI MATHEMATICAL INSTITUTE
Postgraduate Programme in Mathematics
MSc/PhD Entrance Examination
15 May 2018
Important: Questions in Part A will be used for screening. There will be a cut-off for Part A,
which will not be more than twenty (20) marks (out of 40). Your solutions to the questions in
Part B will be marked only if your score in Part A places you over the cut-off. (In particular, if
your score in Part A is at least 20 then your solutions to the questions in Part B will be marked.)
However, note that the scores in both the sections will be taken into account while making the
final decision. In order to qualify for the PhD Mathematics interview, you must
obtain at least fifteen (15) marks from among the starred questions (17∗ )–(20∗ ).

Notation: N, Z, Q, R, R+ and C stand, respectively, for the sets of non-negative integers,


of integers, of rational numbers, of real numbers, of positive real numbers, and of complex
numbers. For a prime number p, Fp is the field with p elements. For a field F , Mn (F ) stands
for the set of n × n matrices over F and GLn (F ) is the set of invertible n × n matrices over
F . The symbol ı denotes a square-root of −1. When considered as topological spaces, Rn or C
are taken with the euclidean topology. When Mn (R) is considered as a topological space, it is
2
identified with Rn .

Part A
Instructions: Each of the questions 1–9 has one or more correct answers. Record your
answers on the attached bubble-sheet by filling in the appropriate circles. Every question is
worth four (4) marks. A solution receives credit if and only if all the correct answers are chosen,
and no incorrect answer is chosen.

(1) Let G be a group of order 6. Let C1 , C2 , . . . , Ck be the distinct conjugacy classes of G.


Which of the following sequences of integers are possible values of (|C1 |, |C2 |, . . . , |Ck |)?
(A) (1, 1, 1, 1, 1, 1);
(B) (1, 5);
(C) (3, 3);
(D) (1, 2, 3).

(2) Let R = F2 [X]. Choose the correct statement(s) from below:


(A) R has uncountably many maximal ideals;
(B) Every maximal ideal of R has infinitely many elements;
(C) For all maximal ideals m of R, R/m is a finite field;
(D) For every integer n, every ideal of R has only finitely many elements of degree ≤ n.

(3) Which of the following spaces are connected?


(A) {(x, y) ∈ R2 | xy = 1} as a subspace of R2 ;
(B) The set of upper triangular matrices as a subspace of Mn (R);
(C) The set of invertible diagonal matrices as a subspace of Mn (R);
(D) {(x, y, z) ∈ R3 | z ≥ 0, z 2 ≥ x2 + y 2 } as a subspace of R3 .
(4) Let A be an n × n nilpotent real matrix A. Define
1 1
eA = In + A + A2 + A3 + · · · .
2! 3!
Choose the correct statement(s) from below:
(A) For every real number t, etA is invertible;
(B) There exists a basis of Rn such that eA is upper-triangular;
(C) There exist B, P ∈ GLn (R) such that B = P eA P −1 and trace(B) = 0;
(D) There exists a basis of Rn such that A is lower-triangular.

(5) Let f (w, x, y, z) = wz − xy. Choose the correct statement(s) from below:
(A) The directional derivative at (1, 0, 0, 1) in the direction (a, b, c, d) is 0 if a + d = 0;
(B) The directional derivative at (1, 0, 0, 1) in the direction (a, b, c, d) is 0 only if a+d =
0;
(C) The vector (0, −1, −1, 0) is normal to f −1 (1) at the point (1, 0, 0, 1);
(D) The set of points (a, b, c, d) where the total derivative of f is zero is finite.

(6) Choose the correct statement(s) from below:


(A) There exists a subfield F of C such that F * R and F ' Q[X]/(2X 3 − 3X 2 + 6);
(B) For every irreducible cubic polynomial f (X) ∈ Q[X], there exists a subfield F of
C such that F * R and F ' Q[X]/f (X);
(C) There exists a subfield F of R such that F ' Q[X]/(2X 3 − 3X 2 + 6);
(D) For every irreducible cubic polynomial f (X) ∈ Q[X], there exists a subfield F of
R such that F ' Q[X]/f (X).

R1
(7) For a continuous function f : [0, 1] −→ R, define an (f ) = 0 xn f (x)dx. Choose the
correct statement(s) from below:
(A) The sequence {an (f )} is bounded for every continuous function f : [0, 1] −→ R;
(B) The sequence {an (f )} is Cauchy for every continuous function f : [0, 1] −→ R;
(C) The sequence {an (f )} converges to 0 for every continuous function f : [0, 1] −→ R;
(D) There exists a continuous function f : [0, 1] −→ R such that the sequence {an (f )}
is divergent.

2
(8) Let f : C −→ C be a holomorphic function. Choose the correct statement(s) from
below:
(A) f (z) is holomorphic;
(B) Suppose that f (R) ⊆ R. Then f (R) is open in R;
(C) the map z 7→ ef (z) is holomorphic;
(D) Suppose that f (C) ⊂ R. Then f (A) is closed in C for every closed subset A of C.

(9) Let f : R −→ R be a twice-differentiable function such that f ( n1 ) = 0 for every positive


integer n. Choose the correct statement(s) from below:
(A) f (0) = 0;
(B) f 0 (0) = 0;
(C) f 00 (0) = 0;
(D) f is a nonzero polynomial.

Instructions: The answer to Question 10 is an integer. You are required to write


the answer in decimal form in the attached bubble-sheet. The question is worth four
(4) marks.

(10) Let A be a non-zero 4×4 complex matrix such that A2 = 0. What is the largest possible
rank of A?

3
Part B
Instructions: Answer six (6) questions from below. Provide sufficient justification.
Write your solutions on the page assigned to each question. Each of the questions is worth
ten (10) marks. In order to qualify for the PhD Mathematics interview, you
must obtain at least fifteen (15) marks from among the starred questions
(17∗ )–(20∗ ). Clearly indicate which six questions you would like us to mark in the six
boxes in the bubble sheet. If the boxes are unfilled, we will mark the first six solutions
that appear in your answer-sheet. If you do not want a solution to be considered, clearly
strike it out.

(11) A subspace Y of R is said to be a retract of R if there exists a continuous map r : R −→ Y


such that r(y) = y for every y ∈ Y .
(A) Show that [0, 1] is a retract of R.
(B) Determine (with appropriate justification) whether every closed subset of R is a
retract of R.
(C) Show that (0, 1) is not a retract of R.

(12) Let N be a positive integer and an be a complex number for every −N ≤ n ≤ N .


Consider the holomorphic function on {z ∈ C|z 6= 0} given by
n=N
X
F (z) = an z n .
n=−N
Consider the function f defined on the open unit disc {z ∈ C : |z| < 1} by
Z
1 F (ξ)
f (z) = dξ,
2πi Γ ξ − z
where Γ is the boundary of the disc, oriented counterclockwise. Write down an expres-
sion for f in terms of the coefficients an of F .

(13) Let φ : [0, 1] −→ R be a continuous function such that


Z 1
φ(t)e−at dt = 0
0
for every a ∈ R+ . Show that for every non-negative integer n,
Z 1
φ(t)tn dt = 0.
0

(14) Let U be a non-empty open subset of R. Suppose that there exists a uniformly contin-
uous homeomorphism h : U −→ R. Show that U = R.

4
(15) Let A be 2 × 2 orthogonal matrix such that det(A) = −1. Show that A represents
reflection about a line in R2 .

(16) A subgroup H of a group G is said to be a characteristic subgroup if σ(H) = H for


every group isomorphism σ : G −→ G of G.
(A) Determine all the characteristic subgroups of (Q, +) (the additive group).
(B) Show that every characteristic subgroup of G is normal in G. Determine whether
the converse is true.

(17∗ ) Write V for the space of 3 × 3 skew-symmetric real matrices.


(A) Show that for A ∈ SO3 (R) and M ∈ V , AM At ∈ V . Write A · M for this action.
(B) Let Φ : R3 −→ V be the map
   
u 0 w −v
 v  7→ −w 0 u .
w v −u 0
With the usual action of SO3 (R) on R3 and the above action on V , show that
Φ(Av) = A · Φ(v) for every A ∈ SO3 (R) and v ∈ R3 .
(C) Show that there does not exist M ∈ V , M 6= 0 such that for every A ∈ SO3 (R),
A · M belongs to the span of M .

(18∗ ) Let m > 1 be an integer and consider the following equivalence relation on C r {0}:
2πıa
z1 ∼ z2 if z1 = z2 e m for some a ∈ Z. Write X for the set of equivalence classes and
π : C r {0} −→ X for the map that takes z to its equivalence class. Define a topology on
X by setting U ⊆ X to be open if and only if π −1 (U ) is open in the euclidean topology
of C r {0}. Determine (with appropriate justification) whether X is compact.

(19∗ ) Let k be a field, n a positive integer and G a finite subgroup of GLn (k) such that |G| > 1.
Further assume that every g ∈ G is upper-triangular and all the diagonal entries of g
are 1.
(A) Show that char k > 0. (Hint: consider the minimal polynomials of elements of G.)
(B) Show that the order of g is a power of char k, for every g ∈ G.
(C) Show that the centre of G has at least two elements.

(20∗ ) Let f : [0, 1] −→ R be a continuous function. Determine (with appropriate justification)


the following limit: Z 1
lim nxn f (x)dx.
n−→∞ 0

5
CHENNAI MATHEMATICAL INSTITUTE
Postgraduate Programme in Mathematics
MSc/PhD Entrance Examination
15 May 2019
Important: Questions in Part A will be used for screening. There will be a cut-off for Part A,
which will not be more than twenty (20) marks (out of 40). Your solutions to the questions in
Part B will be marked only if your score in Part A places you over the cut-off. (In particular, if
your score in Part A is at least 20 then your solutions to the questions in Part B will be marked.)
However, note that the scores in both the sections will be taken into account while making the
final decision. In order to qualify for the PhD Mathematics interview, you must
obtain at least fifteen (15) marks from among the starred questions (17∗ )–(20∗ ).

Notation: N, Z, Q, R, R≥0 , R+ and C stand, respectively, for the sets of non-negative integers,
of integers, of rational numbers, of real numbers, of non-negative real numbers, of positive real
numbers, and of complex numbers. For a field F , Mn (F ) stands for the set of n×n matrices over
F and GLn (F ) is the set of invertible n×n matrices over F . The symbol ı denotes a square-root
of −1. When considered as topological spaces, Rn or C are taken with the euclidean topology.
2
When Mn (R) is considered as a topological space, it is identified with Rn .

Part A
Instructions: Each of the questions 1–9 has one or more correct answers. Record your
answers on the attached bubble-sheet by filling in the appropriate circles. Every question is
worth four (4) marks. A solution receives credit if and only if all the correct answers are chosen,
and no incorrect answer is chosen.

(1) For a field F , F × denotes the multiplicative group (F \ {0}, ×). Choose the correct
statement(s) from below:
(A) Every finite subgroup of R× is cyclic;
(B) The order of every non-trivial finite subgroup of R× is a prime number;
(C) There are infinitely many non-isomorphic non-trivial finite subgroups of R× ;
(D) The order of every non-trivial finite subgroup of C× is a prime number.

(2) Let R be a commutative ring with 1 and I and J ideals of R. Choose the correct
statement(s) from below:
(A) If I or J is maximal then IJ = I ∩ J;
(B) If IJ = I ∩ J, then I or J is maximal;
(C) If IJ = I ∩ J, then 1 ∈ I + J;
(D) If 1 ∈ I + J then IJ = I ∩ J.

(3) Let (X, d) and (Y, ρ) be metric spaces and f : X −→ Y a homeomorphism. Choose the
correct statement(s) from below:
(A) If B ⊆ Y is compact, then f −1 (B) is compact;
(B) If B ⊆ Y is bounded, then f −1 (B) is bounded;
(C) If B ⊆ Y is connected, then f −1 (B) is connected;
(D) If {yn } is Cauchy in Y , then {f −1 (yn )} is Cauchy in X.
(4) Let a, b ∈ R, and consider the R-linear map f : C −→ C, z 7→ az + bz. Choose the
correct statement(s) from below:
(A) f is onto (i.e., surjective) if ab 6= 0;
(B) f is one-one (i.e., injective) if ab 6= 0;
(C) f is onto if a2 6= b2 ;
(D) if a2 = b2 , f is not one-one.

(5) Let
x3 y 3
(
x2 +y 2
, (x, y) 6= (0, 0)
f (x, y) =
0, (x, y) = (0, 0)
Choose the correct statement(s) from below:
(A) f is continuous on R2 ;
(B) f is continuous at every point of R2 \ {(0, 0)};
(C) f is differentiable at every point of R2 \ {(0, 0)};
(D) f is not differentiable at (0, 0).

(6) Let K be the smallest subfield of C containing all the roots of unity. Choose the correct
statement(s) from below:
(A) C is algebraic over K;
(B) K has countably many elements;
(C) Irreducible polynomials in K[X] do not have multiple roots;
(D) The characteristic of K is zero.

(7) The power series



X n2 xn
n!
n=1
equals
(A) x2 ex ;
(B) xex ;
(C) (x2 + x)ex ;
(D) (x2 − x)ex ;

(8) Let f : R −→ R be twice continuously differentiable. Suppose further that f 00 (x) ≥ 0


for every x ∈ R. Choose the correct statement(s) from below:
(A) f is bounded;
(B) f is constant;
(C) RIf f is bounded, then it is infinitely differentiable;
x
(D) 0 f (t)dt is infinitely differentiable with respect to x.
2
(9) Let f (z) be a power-series (with complex coefficients) centred at 0 ∈ C and with a radius
of convergence 2. Suppose that f (0) = 0. Choose the correct statement(s) from below:
(A) f −1 (0) = {0};
(B) If f is a non-constant function on {|z| < 2}, then f −1 (0) = {0};
(C) If f is a non-constant function, then for all ζ ∈ C with sufficiently small |ζ|, the
equation f (z) = ζ has a solution;
(D) Z
f (n) (z)dz = 0
γ
for every n ≥ 1, where γ is a unit circle centred at 0, oriented clockwise, and f (n)
is the nth derivative of f (z).

Instructions: The answer to Question 10 is an integer. You are required to write the
answer in decimal form in the attached bubble-sheet. The question is worth four (4)
marks. )

(10) Let z be a complex variable, and write x = <(z) and y = =(z) for the real and the
imaginary parts, respectively. Let f (z) be a complex polynomial. Let R > 0 be a real
number and γ the circle in C of radius R and centre at 0, oriented in the counter-
clockwise direction. What is the value of
Z
1
(<(f (z))dx + =(f (z))dy)
2πıR γ

3
Part B
Instructions: Answer six (6) questions from below. Provide sufficient justification.
Write your solutions on the page assigned to each question. Each of the questions is worth
ten (10) marks. In order to qualify for the PhD Mathematics interview, you
must obtain at least fifteen (15) marks from among the starred questions
(17∗ )–(20∗ ). Clearly indicate which six questions you would like us to mark in the six
boxes in the bubble sheet. If the boxes are unfilled, we will mark the first six solutions
that appear in your answer-sheet. If you do not want a solution to be considered, clearly
strike it out.

(11) Fix a non-negative integer d. Let


Ad := {A ⊆ C : A is the zero-set of a polynomial of degree ≤ d in C[X]}.
Let T be the coarsest topology on C in which A is closed for every A ∈ Ad .
(A) Determine whether T is Hausdorff.
(B) Show that for every polynomial f (X) ∈ C[X], the function C −→ C defined by
z 7→ f (z) is continuous, where C (on both the sides) is given the topology T .

(12) Let an , n ≥ 0 be complexPnumbers such that limn an = 0.


(A) Show that F (z) := n≥0 an z n is a holomorphic function on {z ∈ C : |z| < 1}.
(B) Let G(z) be a meromorphic function on {z ∈ C : |z| < 2}, with a pole at 1. Show
that G 6= F on {z ∈ C : |z| < 1}. (Hint: consider the function (1 − z)F (z) as
z −→ 1.)

(13) Let | · | : R −→ R≥0 be a function such that for every x, y ∈ R, (i) |x| = 0 if and only if
x = 0; (ii) |x + y| ≤ |x| + |y|; (iii) |xy| = |x||y|. Show that the following are equivalent:
(A) The set {|n| : n ∈ Z} is bounded;
(B) |x + y| ≤ max{|x|, |y|} for every x, y ∈ R.

(14) Let f : [0, 1] −→ R be a continuous function. Show that the sequence


Z 1  n1
|f (x)|n dx
0
is convergent.

(15) Let V be a subspace of the complex vector space Mn (C). Suppose that every non-zero
element of V is an invertible matrix. Show that dimC V ≤ 1.
4
(16) Let n be a positive integer such that every group of order n is cyclic. Show the following.
(A) For all prime numbers p, p2 does not divide n.
(B) If p and q are prime divisors of n, then p does not divide q − 1. (Hint: Consider
2 × 2 matrices  
x y
0 1
p
with x, y ∈ Z/qZ and x = 1. )
(C) Show that (n, φ(n)) = 1, where φ(n) is the number of integers m such that 1 ≤
m ≤ n with gcd(n, m) = 1.

(17∗ ) Let F be a field and G = GLn (F ). For g ∈ G, write Cg = {hgh−1 | h ∈ G}. Let
X = {Cg | g ∈ G, the order of g is 2}. Determine |X|.

(18∗ ) A compactification of a topological space X is a compact topological space Y which


contains a dense subspace homeomorphic to X. Let X = (0, 1], in the subspace topology
of R and f : X −→ R, x 7→ sin x1 . Show the following:
(A) Y := [0, 1] is a compactification of X, but f does not extend to a continuous
function Y −→ R, i.e., there does not exist a continuous function g : Y −→ R such
that g|X = f .
(B) X is homeomorphic to the set X1 := {(t, sin 1t ) | t ∈ X} ⊆ R2 .
(C) The closure Y1 of X1 in R2 is a compactification of X
(D) f extends to a continuous function Y1 −→ R.

(19∗ ) Let f (X) ∈ Z[X] be a monic polynomial. Suppose that α ∈ C and 3α are roots of f .
(A) Show that f (0) 6= 1. (Hint: if ζ and ζ 0 are complex numbers satisfying monic
polynomials in Z[X], then ζζ 0 satisfies a monic polynomial in Z[X].)
(B) Assume that f is irreducible. Let K be the smallest subfield of C containing all
the roots of f . Let σ be a field automorphism of K such that σ(α) = 3α. Show
that σ has finite order and that α = 0.

(20∗ ) Let f : [0, 1] −→ R be a continuous function. Define g(0) = f (0) and g(x) = max{f (y) |
0 ≤ y ≤ x} for 0 < x ≤ 1. Show that g is well-defined and that g is monotone continuous
function.

5
CHENNAI MATHEMATICAL INSTITUTE
Postgraduate Programme in Mathematics
MSc/PhD Entrance Examination
4th October 2020
Important: Questions in Part A will be used for screening. There will be a cut-off for Part A,
which will not be more than twenty (20) marks (out of 40). Your solutions to the questions in
Part B will be marked only if your score in Part A places you over the cut-off. (In particular, if
your score in Part A is at least 20 then your solutions to the questions in Part B will be marked.)
However, note that the scores in both the sections will be taken into account while making the
final decision. In order to qualify for the PhD Mathematics interview, you must
obtain at least fifteen (15) marks from among the starred questions (17∗ )–(20∗ ).

Notation: N, Z, Q, R, R≥0 , R+ and C stand, respectively, for the sets of non-negative integers,
of integers, of rational numbers, of real numbers, of non-negative real numbers, of positive real
numbers, and of complex numbers. For a prime power q, Fq is the field with q elements. For
a field F , Mn (F ) stands for the set of n × n matrices over F . When considered as topological
spaces, Rn or C are taken with the euclidean topology.

Part A
Instructions: Each of the questions 1–9 has one or more correct answers. Record your
answers on the attached bubble-sheet by filling in the appropriate circles. Every question is
worth four (4) marks. A solution receives credit if and only if all the correct answers are chosen,
and no incorrect answer is chosen.
(1) Let G be a group and N be a proper normal subgroup. Pick the true statement(s) from
below.
(A) If N and the quotient G/N is finite, then G is finite.
(B) If the complement G \ N of N in G is finite, then G is finite.
(C) If both N and the quotient G/N are cyclic, then G is cyclic.
(D) G is isomorphic to N × G/N .
(2) Let R denote the ring of all continuous functions from R to R, where addition and
multiplication are given, respectively, by (f +g)(x) = f (x)+g(x) and (f g)(x) = f (x)g(x)
for every f, g ∈ R and x ∈ R. A zero-divisor in R is a non-zero f ∈ R such that f g = 0
for some non-zero g ∈ R. Pick the true statement(s) from below:
(A) R has zero-divisors.
(B) If f is a zero-divisor, then f 2 = 0.
(C) If f is a non-constant function and f −1 (0) contains a non-empty open set, then f
is a zero-divisor.
(D) R is an integral domain.
(3) Let U = {(x, y) ∈ R2 | x < y 2 < 4} and V = {(x, y) ∈ R2 | 0 < xy < 4}, both taken
with the subspace topology from R2 . Which of the following statement(s) is/are true?
(A) There exists a non-constant continuous map V −→ R whose image is not an inter-
val.
(B) Image of U under any continuous map U −→ R is bounded.
(C) There exists an  > 0 such that given any p ∈ V the open ball B (p) with centre p
and radius  is contained in V .
(D) If C is a closed subset of R2 which is contained in U , then C is compact.
(4) Let A and B be 5 × 5 real matrices with A2 = B 2 . Which of the following statements
is/are correct?
(A) Either A = B or A = −B.
(B) A and B have the same eigen spaces.
(C) A and B have the same eigen values.
1
(D) A13 B 3 = A3 B 13 .
(5) Consider the function f : R2 −→ R given by
x2 p 2
 
f (x, y) = 1 − cos x + y2
y
for y 6= 0 and f (x, 0) = 0. (The square root is chosen to be non-negative). Pick the
correct statement(s) from below:
(A) f is continuous at (0, 0).
(B) f is an open map.
(C) f is differentiable at (0, 0).
(D) f is a bounded function. P
(6) Which P of the following is/arePtrue for a series of real numbers an ?
2
(A) If P an converges then P an converges;
(B) If P a2n converges then P an converges;
1
(C) if a2 converges then a converges;
P n Pn 1 n
(D) If |an | converges then n an converges;
(7) Which of the following functions are uniformly continuous on R?
(A) f (x) = x;
(B) f (x) = x2 ;
(C) f (x) = (sin x)2 ;
(D) f (x) = e−|x| .
(8) Let U and V be non-empty open connected subsets of C and f : U −→ V a analytic
function. Which of the following statement(s) is/are true?
(A) f 0 (z) 6= 0 for every z ∈ U .
(B) If f is bijective, then f 0 (z) 6= 0 for every z ∈ U .
(C) If f 0 (z) 6= 0 for every z ∈ U , then f is bijective.
(D) If f 0 (z) 6= 0 for every z ∈ U , then f is injective.
(9) Let U denote the unit open disc centred at 0. Let f : U \ {0} −→ C be an analytic
function. Assume that limz−→0 zf (z) = 0.
(A) limz−→0 |f (z)| exists and is in R.
(B) f has a pole of order 1 at 0.
(C) zf (z) has a zero of order 1 at 0.
(D) There exists an analytic function g : U −→ C such that g(z) = f (z) for every
z ∈ U \ {0}.
Instructions: The answers to Question 10 is an integer. Please write the answer in
decimal form in the attached bubble-sheet. The question is worth four (4) marks.
(10) Let f (x) = x2 + ax + b ∈ F3 [X]. What is the number of non-isomorphic quotient rings
F3 [X]/(f (X))?

Part B
Instructions: Answer six (6) questions from below. Provide sufficient justification.
Write your solutions on the page assigned to each question. Each of the questions is worth
ten (10) marks. In order to qualify for the PhD Mathematics interview, you
must obtain at least fifteen (15) marks from among the starred questions
(17∗ )–(20∗ ). Clearly indicate which six questions you would like us to mark in the six
boxes in the bubble sheet. If the boxes are unfilled, we will mark the first six solutions
that appear in your answer-sheet. If you do not want a solution to be considered, clearly
strike it out.
(11) Let (X, d) be a compact metric space. For x ∈ X and  > 0, define B (x) := {y ∈
X | d(x, y) < }. For C ⊆ X and  > 0, define B (C) := ∪x∈C B (x). Let K be the
set of non-empty compact subsets of X. For C, C 0 ∈ K, define δ(C, C 0 ) = inf{ | C ⊆
B (C 0 ) and; C 0 ⊆ B (C)}. Show that (K, δ) is a compact metric space.
2
(12) Let f be a non-constant entire function with f (z) 6= 0 for all z ∈ C. Consider the set
U = {z : |f (z)| < 1}. Show that all connected components of U are unbounded.
(13) Let F ⊆ R3 be a non-empty finite set, and X = R3 \F , taken with the subspace topology
of R3 . Show that X is homeomorphic to a complete metric space. (Hint: Look for a
suitable continuous function from X to R.)
(14) Show that there is no differentiable function f : R −→ R such that f (0) = 1 and
f 0 (x) ≥ (f (x))2 for every x ∈ R.
(15) Let a1 , . . . , an be distinct complex numbers. Show that the functions ea1 z , . . . , aan z are
linearly independent over C.
(16) The Frattini subgroup of a finite group G is the intersection of all its proper maximal
subgroups. Let p be a prime number. Show that the Frattini subgroup of Z/pn , n ≥ 2,
is generated by p.
(17∗ ) Let M ∈ Mn (C). Show that M is diagonalizable if and only if for every polynomial
P (X) ∈ C[X] such that P (M ) is nilpotent, P (M ) = 0.
(18∗ ) X is said to have the universal extension property if for every normal space Y and
every closed subset A ⊂ Y and every continuous function f : A −→ X, f extends to
a continuous function from Y to X. You may assume, without proof, that R2 has the
universal extension property.
(A) Prove or find a counterexample: If X has the universal extension property, then X
is connected.
(B) Give an example (with justification) of a compact subset X of R2 that does not
have the universal extension property.
(C) Let X = {(x, sin x) | x ∈ R}. Then show that X has the universal extension
property.

(19 ) Let p be a prime number and q a power of p. Let K be an algebraic closure of Fq . Say
that a polynomial f (X) ∈ K[X] is a q-polynomial if it is of the form
n
i
X
f (X) = ai X q
i=0
Let f (X) be a q-polynomial of degree qn,
with a0 6= 0. Show that the set of zeros of
f (X) is an n-dimensional vector-space over Fq .
(20∗ ) Let an , n ≥ 1 be a sequence of real numbers. If an → a, show that
a1 + 2a2 + 3a3 + · · · + nan a
bn = 2
→ .
n 2

3
CHENNAI MATHEMATICAL INSTITUTE
Postgraduate Programme in Mathematics
MSc/PhD Entrance Examination
1st August 2021
Important information and instructions:
(1) Questions in Part A (Questions 1 – 10) will be used for screening. There will be a cut-off for Part
A, which will not be more than 20 marks (out of 40).
(2) Each question in Part A has one or more correct answers. Enter your answers to these questions
into the computer as instructed. Every question is worth 4 marks. A solution will receive credit if and
only if all the correct answers are chosen, and no incorrect answer is chosen.
(3) Your solutions to the questions in Part B (Questions 11– 20∗ ) will be marked only if your score in
Part A places you over the cut-off. (In particular, if your score in Part A is at least 20 then your solutions
to the questions in Part B will be marked.)
(4) Answer 6 questions from Part B, on the pages assigned to them, with sufficient justification. Each
question is worth 10 marks. Clearly indicate which six questions you would like us to mark in the six
boxes on the front sheet. If the boxes are unfilled, we will mark the first six solutions that appear in your
answer-sheet. If you do not want a solution to be considered, clearly strike it out.
(5) The scores in both the sections will be taken into account while making the final decision. In order
to qualify for the PhD Mathematics interview, you must obtain at least 15 marks from among the starred
questions 17∗ –20∗ .

Notation: N, Z, Q, R, and C stand, respectively, for the sets of non-negative integers, of integers, of ra-
tional numbers, of real numbers, of non-negative real numbers, of positive real numbers, and of com-
plex numbers. For a field 𝐹 and a positive integer 𝑛, 𝑀𝑛 (𝐹 ) stands for the set of 𝑛 × 𝑛 matrices over 𝐹
and GL(𝑛, 𝐹 ) for the set of invertible 𝑛 × 𝑛 matrices over 𝐹 . 𝐼𝑛 denotes the 𝑛 × 𝑛 identity matrix; the
field will be clear from context. When considered as topological spaces, R𝑛 or C. are taken with the
euclidean topology. Similarly, as topological spaces, 𝑀𝑛 (R) and 𝑀𝑛 (C) are taken with the euclidean
topology. GL(𝑛, R) is considered as a topological subspace of 𝑀𝑛 (R).

Part A
(1) Which of the following can not be the class equation for a group of appropriate order?
(A) 14 = 1 + 1 + 1 + 1 + 1 + 1 + 1 + 7.
(B) 18 = 1 + 1 + 1 + 1 + 2 + 3 + 9.
(C) 6 = 1 + 2 + 3.
(D) 31 = 1 + 3 + 6 + 6 + 7 + 8.
∫∞ ∞
(2) Consider the improper integral 𝑥 (log1 𝑥) 2 𝑑𝑥 and the infinite series 1
Í
𝑘 (log 𝑘) 2 . Which of the
2 𝑘=2
following is/are true?
(A) The integral converges but the series does not converge.
(B) The integral does not converge but the series converges.
(C) Both the integral and the series converge.
(D) The integral and the series both fail to converge.

(3) Let 𝐴 ∈ 𝑀2 (R) be a nonzero matrix. Pick the correct statement(s) from below.
(A) If 𝐴2 = 0, then (𝐼 2 − 𝐴) 5 = 0.
(B) If 𝐴2 = 0, then (𝐼 2 − 𝐴) is invertible.
(C) If 𝐴3 = 0, then 𝐴2 = 0.
(D) If 𝐴2 = 𝐴3 ≠ 0, then 𝐴 is invertible.
(4) Let 𝑓 : [0, 1] −→ [0, 1] be a continuous function. Which of the following is/are true?
(A) For every continuous 𝑔 : [0, 1] −→ R with 𝑔(0) = 0 and 𝑔(1) = 1 there exists 𝑥 ∈ [0, 1] with
𝑓 (𝑥) = 𝑔(𝑥).
1
(B) For every continuous 𝑔 : [0, 1] −→ R with 𝑔(0) < 0 and 𝑔(1) > 1 there exists 𝑥 ∈ [0, 1] with
𝑓 (𝑥) = 𝑔(𝑥).
(C) For every continuous 𝑔 : [0, 1] −→ R with 0 < 𝑔(0) < 1 and 0 < 𝑔(1) < 1 there exists
𝑥 ∈ [0, 1] with 𝑓 (𝑥) = 𝑔(𝑥).
(D) For every continuous 𝑔 : [0, 1] −→ [0, 1] there exists 𝑥 ∈ [0, 1] with 𝑓 (𝑥) = 𝑔(𝑥).
(5) Let 𝐼, 𝐽 be nonempty open intervals in R. Let 𝑓 : 𝐼 −→ 𝐽 and 𝑔 : 𝐽 −→ R be functions. Let
ℎ : 𝐼 −→ R be the composite function 𝑔 ◦ 𝑓 . Pick the correct statement(s) from below.
(A) If 𝑓 , 𝑔 are continuous, then ℎ is continuous.
(B) If 𝑓 , 𝑔 are uniformly continuous, then ℎ is uniformly continuous.
(C) If ℎ is continuous, then 𝑓 is continuous.
(D) If ℎ is continuous, then 𝑔 is continuous.
(6) Let 𝐴, 𝐵 be non-empty subsets of R2 . Pick the correct statement(s) from below:
(A) If 𝐴 is compact, 𝐵 is open and 𝐴 ∪ 𝐵 is compact, then 𝐴 ∩ 𝐵 ≠ ∅.
(B) If 𝐴 and 𝐵 are path-connected and 𝐴 ∩ 𝐵 ≠ ∅ then 𝐴 ∪ 𝐵 is path-connected.
(C) If 𝐴 and 𝐵 are connected and open and 𝐴 ∩ 𝐵 ≠ ∅, then 𝐴 ∩ 𝐵 is connected.
(D) If 𝐴 is countable with |𝐴| ≥ 2, then 𝐴 is not connected.
(7) Pick the correct
Î∞ statement(s) from below.𝑛
(A) 𝑋 = Î 𝑛=1 𝑋𝑛 where 𝑋𝑛 = {1, 2, . . . , 2 } for 𝑛 ≥ 1 is not compact in the product topology.

(B) 𝑌 = Î𝑛=1 𝑌𝑛 where 𝑌𝑛 = [0, 2𝑛 ] ⊆ R for 𝑛 ≥ 1 is path-connected in the product topology.

(C) 𝑍 = Î 𝑛=1 𝑍𝑛 where 𝑍𝑛 = (0, 𝑛1 ) ⊆ R for 𝑛 ≥ 1 is compact in the product topology.

(D) 𝑃 = 𝑛=1 𝑃𝑛 where 𝑃𝑛 = {0, 1} for 𝑛 ≥ 1 (with product topology) is homeomorphic to (0, 1).
𝑧 −1
(8) Let 𝑓 (𝑧) = 𝑧𝑒(𝑧−1) be defined on the extended complex plane C ∪ {∞}. Which of the following
is/are true?
(A) 𝑧 = 0, 𝑧 = 1, 𝑧 = ∞ are poles.
(B) 𝑧 = 1 is a simple pole.
(C) 𝑧 = 0 is a removable singularity.
(D) 𝑧 = ∞ is an essential singularity

(9) For 𝐴 ∈ 𝑀3 (C), let 𝑊𝐴 = {𝐵 ∈ 𝑀3 (C) | 𝐴𝐵 = 𝐵𝐴}. Which of the following is/are true?
(A) For all diagonal 𝐴 ∈ 𝑀3 (C), 𝑊𝐴 is a linear subspace of 𝑀3 (C) with dimC 𝑊𝐴 ≥ 3.
(B) For all 𝐴 ∈ 𝑀3 (C), 𝑊𝐴 is a linear subspace of 𝑀3 (C) with dimC 𝑊𝐴 > 3.
(C) There exists 𝐴 ∈ 𝑀3 (C) such that 𝑊𝐴 is a linear subspace of 𝑀3 (C) with dimC 𝑊𝐴 = 3.
(D) If 𝐴 ∈ 𝑀3 (C) is diagonalizable, then every element of 𝑊𝐴 is diagonalizable.
(10) Let 𝐾 be a field of order 243 and let 𝐹 be a subfield of 𝐾 of order 3. Pick the correct statement(s)
from below.
(A) There exists 𝛼 ∈ 𝐾 such that 𝐾 = 𝐹 (𝛼).
(B) The polynomial 𝑥 242 = 1 has exactly 242 solutions in 𝐾.
(C) The polynomial 𝑥 26 = 1 has exactly 26 roots in 𝐾.
(D) Let 𝑓 (𝑥) ∈ 𝐹 [𝑥] be an irreducible polynomial of degree 5. Then 𝑓 (𝑥) has a root in 𝐾.

2
Part B
(11) Let 𝐺 be a finite group and 𝑋 the set of all abelian subgroups 𝐻 of 𝐺 such that 𝐻 is a maximal
subgroup of 𝐺 (under inclusion) and is not normal in 𝐺. Let 𝑀 and 𝑁 be distinct elements of 𝑋 .
Show the following:
(A) The subgroup of 𝐺 generated by 𝑀 and 𝑁 is contained in the centralizer of 𝑀 ∩ 𝑁 in 𝐺.
(B) 𝑀 ∩ 𝑁 is the centre of 𝐺.
(12) Let 𝑓 : R2 −→ R2 be a smooth function whose derivative at every point is non-singular. Suppose
that 𝑓 (0) = 0 and for all 𝑣 ∈ R2 with |𝑣 | = 1, |𝑓 (𝑣)| ≥ 1. Let 𝐷 denote the open unit ball
{𝑣 : |𝑣 | < 1}. Show that 𝐷 ⊂ 𝑓 (𝐷). (Hint: Show that 𝑓 (𝐷) ∩ 𝐷 is closed in 𝐷.)
(13) Let 𝑋 be a topological space and 𝑥 0 ∈ 𝑋 . Let S = {𝐵 ⊆ 𝑋 | 𝑥 0 ∈ 𝐵 and 𝐵 is connected}. Let
Ø
𝐴= 𝐵.
𝐵 ∈S
Show that 𝐴 is closed.
Í
(14) Let 𝑓 : [1, ∞) −→ R \ {0} be uniformly continuous. Show that the series 𝑛 ≥1 1/𝑓 (𝑛) is diver-
gent.
∫∞ √ 1/100
(15) Show that 𝑥 10𝑒 −𝑥 𝑑𝑥 < ∞.
0
(16) Consider the following statement: Let 𝐹 be a field and 𝑅 = 𝐹 [𝑋 ] the polynomial ring over 𝐹 in
one variable. Let 𝐼 1 and 𝐼 2 be maximal ideals of 𝑅 such that the fields 𝑅/𝐼 1 ' 𝑅/𝐼 2 ; 𝐹 . Then
𝐼1 = 𝐼2 .
Prove or find a counterexample to the following claims:
(A) The above statement holds if 𝐹 is a finite field.
(B) The above statement holds if 𝐹 = R.

(17 ) Let O(2, R) be the subgroup of GL(2, R) consisting of orthogonal matrices, i.e., matrices 𝐴 satis-
fying 𝐴tr𝐴 = 𝐼 . Let B+ (2, R) be the subgroup of GL(2, R) consisting of upper triangular matrices
with positive entries on the diagonal.
(A) Let 𝐴 ∈ GL(2, R). Show that there exist 𝐴𝑜 ∈ O(2, R) and 𝐴𝑏 ∈ B+ (2, R) such that 𝐴 = 𝐴𝑜 𝐴𝑏 .
(Hint: use appropriate elementary column operations.)
(B) Show that the map
𝜙 : O(2, R) × B+ (2, R) −→ GL(2, R) (𝐴 0, 𝐴 00) ↦→ 𝐴 0𝐴 00
is injective.
(C) Show that GL(2, R) is homeomorphic to O(2, R) × B+ (2, R). (Hint: first show that the map
𝐴 ↦→ 𝐴𝑏 is continuous.)
(18∗ ) Let 𝐹 be a field of characteristic 𝑝 > 0 and𝑉 a finite-dimensional 𝐹 -vector-space. Let 𝜙 ∈ GL(𝑉 )
be an element of order 𝑝 3 . Show that there exists a basis of 𝑉 with respect to which 𝜙 is given by
an upper-triangular matrix with 1’s on the diagonal.

√5
(19 ) Let 𝜁 5 ∈ C be a primitive 5th root √5 of unity; let 2 denote a real 5th root of 2, and let 𝚤 denote a
square root of −1. Let 𝐾 = Q(𝜁 5, 2).
(A) Find the degree [𝐾 : Q] of the field 𝐾 over Q.
(B) Determine if 𝚤 ∈ Q(𝜁 5 ). (Hint: You may use, without proof, the following fact: if 𝜁 20 ∈ C is a
primitive 20th root of unity, then [Q(𝜁 20 ) : Q] > 4.)
(C) Determine if 𝚤 ∈ 𝐾.
(20∗ ) Let 𝑎 0 and 𝑎 1 be complex numbers and define 𝑎𝑛 = 2𝑎𝑛−1 + 𝑎𝑛−2 for 𝑛 ≥ 2. Í
(A) Show that there are polynomials 𝑝 (𝑧), 𝑞(𝑧) ∈ C[𝑧] such that 𝑞(0) ≠ 0 and 𝑛 ≥0 𝑎𝑛 𝑧𝑛 is the
𝑝 (𝑧)
Taylor series expansion (around 0) of 𝑞 (𝑧) .
(B) Let 𝑎 0 = 1 and 𝑎 1 = 2. Show that there exist complex numbers 𝛽 1, 𝛽 2, 𝛾 1, 𝛾 2 such that
𝑎𝑛 = 𝛽 1𝛾 1𝑛+1 + 𝛽 2𝛾 2𝑛+1
for all 𝑛.
3
CHENNAI MATHEMATICAL INSTITUTE
Postgraduate Programme in Mathematics
MSc/PhD Entrance Examination
22nd May 2022
Important information and instructions:
(1) Questions in Part A (Questions 1 – 10) will be used for screening. There will be a cut-off for Part
A, which will not be more than 20 marks (out of 40).
(2) Each question in Part A has one or more correct answers. Enter your answers to these questions
into the computer as instructed. Every question is worth 4 marks. A solution will receive credit if and
only if all the correct answers are chosen, and no incorrect answer is chosen.
(3) Your solutions to the questions in Part B (Questions 11– 20∗ ) will be marked only if your score in
Part A places you over the cut-off. (In particular, if your score in Part A is at least 20 then your solutions
to the questions in Part B will be marked.)
(4) Answer 6 questions from Part B, on the pages assigned to them, with sufficient justification. Each
question is worth 10 marks. Clearly indicate which six questions you would like us to mark in the six
boxes on the front sheet. If the boxes are unfilled, we will mark the first six solutions that appear in your
answer-sheet. If you do not want a solution to be considered, clearly strike it out.
(5) The scores in both the sections will be taken into account while making the final decision. You are
advised to spend about 90 minutes on Part B. In order to qualify for the PhD Mathematics interview,
you must obtain at least 15 marks from among the starred questions 17∗ –20∗ .
(6) Time: 3 hours.

Notation: N, Z, Q, R, and C stand, respectively, for the sets of non-negative integers, of integers, of
rational numbers, of real numbers, and of complex numbers. For a field 𝐹 and a positive integer 𝑛,
GL(𝑛, 𝐹 ) stands for the set of invertible 𝑛 × 𝑛 matrices over 𝐹 . 𝐼𝑛 denotes the 𝑛 × 𝑛 identity matrix;
the field will be clear from context. When considered as topological spaces, R𝑛 or C are taken with the
euclidean topology.

Part A
(1) By a simple group, we mean a group 𝐺 in which the only normal subgroups are {1𝐺 } and 𝐺. Pick
the correct statement(s) from below.
(A) No group of order 625 is simple.
(B) GL(2, R) is simple.
(C) Let 𝐺 be a simple group of order 60. Then 𝐺 has exactly six subgroups of order 5.
(D) Let 𝐺 be a group of order 60. Then 𝐺 has exactly seven subgroups∫ ∞ of order 3.
(2) Let 𝑓 : R −→ (0, ∞) be an infinitely differentiable function with −∞ 𝑓 (𝑡)𝑑𝑡 = 1. Pick the correct
statement(s) from below.
(A) 𝑓 (𝑡) is bounded.
(B) lim |𝑡 |−→∞ 𝑓 0 (𝑡) = 0.
(C) There exists 𝑡 0 ∈ R such that 𝑓 (𝑡 0 ) ≥ 𝑓 (𝑡) for all 𝑡 ∈ R.
(D) 𝑓 00 (𝑎) = 0 for some 𝑎 ∈ R.
(3) Let P𝑛 = {𝑓 (𝑥) ∈ R[𝑥] | deg 𝑓 (𝑥) ≤ 𝑛}, considered as an (𝑛 + 1)-dimensional real vector space.
d𝑓
Let 𝑇 be the linear operator 𝑓 ↦→ 𝑓 + d𝑥 on P𝑛 . Pick the correct statement(s) from below.
(A) 𝑇 is invertible.
(B) 𝑇 is diagonalizable.
(C) 𝑇 is nilpotent.
(D) (𝑇 − 𝐼 ) 2 = (𝑇 − 𝐼 ) where 𝐼 is the identity map.
(4) Pick the correct statement(s) from below.
(A) There exists a finite commutative ring 𝑅 of cardinality 100 such that 𝑟 2 = 𝑟 for all 𝑟 ∈ 𝑅.
(B) There is a field 𝐾 such that the additive group (𝐾, +) is isomorphic to the multiplicative
group (𝐾 ×, ·).
1
(C) An irreducible polynomial in Q[𝑥] is irreducible in Z[𝑥].
(D) A monic polynomial of degree 𝑛 over a commutative ring 𝑅 has at most 𝑛 roots in 𝑅.
(5) Pick the correct statement(s) from below.
(A) if 𝑓 is continuous and bounded on (0, 1), then 𝑓 is uniformly continuous on (0, 1).
(B) If 𝑓 is uniformly continuous on (0, 1), then 𝑓 is bounded on (0, 1).
(C) If 𝑓 is continuous on (0, 1) and lim𝑥−→0+ 𝑓 (𝑥) and lim𝑥−→1− 𝑓 (𝑥) exists, then 𝑓 is uniformly
continuous on (0, 1).
(D) Product of a continuous and a uniformly continuous function on [0, 1] is uniformly contin-
uous.
(6) Let 𝑋 be the metric space of real-valued continuous functions on the interval [0, 1] with the
“supremum distance”:
𝑑 (𝑓 , 𝑔) = sup{|𝑓 (𝑥) − 𝑔(𝑥)| : 𝑥 ∈ [0, 1]} for all 𝑓 , 𝑔 ∈ 𝑋 .
Let 𝑌 = {𝑓 ∈ 𝑋 : 𝑓 ( [0, 1]) ⊂ [0, 1]} and 𝑍 = {𝑓 ∈ 𝑋 : 𝑓 ( [0, 1]) ⊂ [0, 21 ) ∪ ( 21 , 1]}. Pick the
correct statement(s) from below.
(A) 𝑌 is compact.
(B) 𝑋 and 𝑌 are connected.
(C) 𝑍 is not compact.
(D) 𝑍 is path-connected.
(7) Let 𝑋 := {(𝑥, 𝑦, 𝑧) ∈ R3 | 𝑧 ≤ 0, or 𝑥, 𝑦 ∈ Q} with subspace topology. Pick the correct
statement(s) from below.
(A) 𝑋 is not locally connected but path connected.
(B) There exists a surjective continuous function 𝑋 −→ Q ≥0 (the set of non-negative rational
numbers, with the subspace topology of R).
(C) Let 𝑆 be the set of all points 𝑝 ∈ 𝑋 having a compact neighbourhood (i.e. there exists a
compact 𝐾 ⊂ 𝑋 containing 𝑝 in its interior). Then 𝑆 is open.
(D) The closed and bounded subsets of 𝑋 are compact.
(8) Consider the complex polynomial 𝑃 (𝑥) = 𝑥 6 + 𝑖𝑥 4 + 1. (Here 𝑖 denotes a square-root of −1.) Pick
the correct statement(s) from below.
(A) 𝑃 has at least one real zero.
(B) P has no real zeros.
(C) 𝑃 has at least three zeros of the form 𝛼 + 𝑖𝛽 with 𝛽 < 0.
(D) 𝑃 has exactly three zeros 𝛼 + 𝑖𝛽 with 𝛽 > 0.
(9) Let 𝑣 a (fixed) unit vector in R3 . (We think of elements of R𝑛 as column vectors.) Let 𝑀 = 𝐼 3 −2𝑣𝑣 𝑡 .
Pick the correct statement(s) from below.
(A) 0 is an eigenvalue of 𝑀.
(B) 𝑀 2 = 𝐼 3 .
(C) 1 is an eigenvalue of 𝑀.
(D) The eigenspace
Í for the eigenvalue −1 is 2-dimensional.
(10) Let 𝑓 (𝑧) = Í 𝑛 ≥0 𝑛 𝑧 be an analytic function on the open unit disc 𝐷 around 0 with 𝑎 1 ≠ 0.
𝑎 𝑛

Suppose that 𝑛 ≥2 |𝑛𝑎𝑛 | < |𝑎 1 |. Then which of the following are true?
(A) There are only finitely many such 𝑓 .
(B) |𝑓 0 (𝑧)| > 0 for all 𝑧 ∈ 𝐷. Í
(C) If 𝑧, 𝑤 ∈ 𝐷 are such that 𝑧 ≠ 𝑤 and 𝑓 (𝑧) = 𝑓 (𝑤), then 𝑎 1 = − 𝑛 ≥2 𝑎𝑛 (𝑧𝑛−1 + 𝑧𝑛−2𝑤 + · · · +
𝑤 𝑛−1 ).
(D) 𝑓 is one-one on 𝐷.

2
Part B
(11) Let 𝐴 ∈ GL(3, Q) with 𝐴𝑡 𝐴 = 𝐼 3 . Assume that
1 1
 
𝐴 1 = 𝜆 1
 
1 1
 
for some 𝜆 ∈ C.
(A) Determine the possible values of 𝜆.
(B) Determine 𝑥 + 𝑦 + 𝑧 where 𝑥, 𝑦, 𝑧 is given by
𝑥  1
   
𝑦  = 𝐴 −1
   
𝑧  0
   
(12) Consider the function 𝑆 (𝑎) defined by the limit below:
1𝑎 + 2𝑎 + 3𝑎 + · · · + 𝑛𝑎
𝑆 (𝑎) := lim .
𝑛−→∞ (𝑛 + 1) 𝑎−1 [(𝑛𝑎 + 1) + (𝑛𝑎 + 2) + · · · + (𝑛𝑎 + 𝑛)]

Find the sum of all values 𝑎 such that 𝑆 (𝑎) = 601 .


(13) Let 𝑈 and 𝑉 be non-empty open connected subsets of C and 𝑓 : 𝑈 −→ 𝑉 an analytic function.
Suppose that for all compact subsets 𝐾 of 𝑉 , 𝑓 −1 (𝐾) is compact. Show that 𝑓 (𝑈 ) = 𝑉 .
(14) Let 𝐺 be a finite group that has a non-trivial subgroup 𝑁 (i.e. {1𝐺 } ≠ 𝑁 ≠ 𝐺) that is contained
in every non-trivial subgroup of 𝐺. Show that
(A) 𝐺 is a 𝑝-group for some prime number 𝑝;
(B) 𝑁 is a normal subgroup of 𝐺.
(15) Let 𝑓 be an entire function such that 𝑓 maps the open unit ball 𝐷 around 0 to itself. Suppose
further that 𝑓 (0) = 0 and 𝑓 (1) = 1. Show that 𝑓 0 (1) ∈ R and that |𝑓 0 (1)| ≥ 1.
(16) Let 𝐹 be a field such that it has a finite non-Galois extension field. Let 𝑉 be a finite-dimensional
vector-space overÐ 𝐹 . Let 𝑉1, . . . , 𝑉𝑟 be proper subspaces of 𝑉 . Prove or disprove the following
assertion: 𝑉 ≠ 𝑟𝑖=1 𝑉𝑖 .

3
(17∗ ) For a ring homomorphism 𝑅 −→ 𝑆 (of commutative rings) and an ideal 𝐼 of 𝑅, the fibre over
𝐼 is the ring 𝑆/𝐼𝑆, i.e., the quotient of 𝑆 by the 𝑆-ideal generated by the image of 𝐼 in 𝑆. Let
𝑆 = C[𝑋, 𝑌 ]/(𝑋𝑌 − 1) and 𝑅 = C[𝑥 + 𝛼𝑦] where 𝛼 ∈ C and 𝑥, 𝑦 are the images of 𝑋, 𝑌 in 𝑆.
Consider the ring homomorphism 𝑅 ⊆ 𝑆. Let 𝐼 = (𝑥 + 𝛼𝑦 − 𝛽)𝑅, where 𝛽 ∈ C. For each non-
negative integer 𝑛, determine the set of (𝛼, 𝛽) such that the fibre over 𝐼 has exactly 𝑛 maximal
ideals.
(18∗ ) Let 𝑄 be the space of infinite sequences
x := (𝑥 1, 𝑥 2, . . . , 𝑥𝑛 , . . . )
of real numbers 𝑥𝑛 ∈ [0, 1], with the product topology coming from the identification 𝑄 =
[0, 1] N . ([0, 1] has the euclidean topology.) Let 𝑆 : 𝑄 −→ R be the map
∑︁ 𝑥𝑛
𝑆 (x) := .
𝑛
𝑛2
(A) Let 𝑄 2 := {(𝑦1, 𝑦2, . . . , 𝑦𝑛 , . . . ) | 0 ≤ 𝑦𝑛 ≤ 𝑛1 }. Show that 𝑄 2 is compact.
(B) Let 𝐷 : 𝑄 2 −→ 𝑄 be the map
(𝑦1, 𝑦2, . . . , 𝑦𝑛 , . . . ) ↦→ (𝑦1, 2𝑦2, . . . , 𝑛𝑦𝑛 , . . . ).
Show that 𝐷 is a homeomorphism. (Hint: first show that 𝑄 is Hausdorff.)
(C) Show that 𝑆 ◦𝐷 : 𝑄 2 −→ R is continuous. (Hint: Show that there is a suitable inner-product
space (𝐿, h−, −i) and a vector a ∈ 𝐿 such that (𝑆 ◦ 𝐷) (x) = hx, ai for each x ∈ 𝑄 2 .)
(D) Show that 𝑆 is continuous.
(19∗ ) Let 𝐸 be a finite extension of the field Q. We say that a homomorphism of fields 𝜙 : 𝐸 −→ C is
real if 𝜙 (𝐸) ⊂ R. Prove or disprove each of the following assertions:
(A) For each prime number 𝑝, the field Q(𝑝 1/12 ) has exactly one real embedding in C. (𝑝 1/12 is
the unique real number 𝑟 > 0 such that 𝑟 12 = 𝑝.)
(B) If [𝐸 : Q] = 11, there exists a real embedding of 𝐸.
(C) If 𝐸 is a Galois extension of Q and [𝐸 : Q] = 11, then every embedding 𝐸 −→ C is a real
embedding.
(20∗ ) A continuous map 𝑓 : 𝐴 −→ 𝐵 between two metric space (𝐴, 𝑑𝐴 ), (𝐵, 𝑑𝐵 ) is said to be a bilip-
schitz map if there exists a real number 𝜆 > 0 such that (1/𝜆)𝑑𝐴 (𝑎 0, 𝑎 1 ) ≤ 𝑑𝐵 (𝑓 (𝑎 0 ), 𝑓 (𝑎 1 )) ≤
𝜆𝑑𝐴 (𝑎 0, 𝑎 1 ) for each 𝑎 0, 𝑎 1 ∈ 𝐴.
Let 𝑋 = R2 \ {0} and 𝑌 = S1 × R = {(𝑥, 𝑦, 𝑧) ∈ R3 | 𝑥 2 +𝑦 2 = 1}. Let 𝑑𝑋 (respectively, 𝑑𝑌 ) be the
euclidean metric on 𝑋 induced from R2 (respectively, on 𝑌 induced from R3 ). Let 𝑓 : 𝑋 −→ 𝑌 be
a bilipschitz map.
(A) Let 𝑅 > 0 and 𝐶𝑅 ⊆ 𝑋 the circle of radius 𝑅 with centre at 0. Let 𝑓¯ : 𝑋 −→ R be the
composite of 𝑓 and the projection from 𝑌 = S1 × R to its second factor R. Let 𝐿𝑅 be the
length of the interval 𝑓¯(𝐶𝑅 ) ⊆ R. Let 𝑎, 𝑏 ∈ 𝑋 be such that 𝑓¯(𝑏) = 𝑓¯(𝑎) + 𝐿𝑅 . Show that
𝑑𝑋 (𝑎, 𝑏) ≥ (2𝑅 − 2𝜆)/𝜆 2 .
(B) Let 𝐶 1 and 𝐶 2 be the two arcs of 𝐶𝑅 , joining 𝑎 and 𝑏. Show that there exists 𝑥 1 ∈ 𝐶 1 and
𝑓 (𝑎)+𝑓 (𝑏)
𝑥 2 ∈ 𝐶 2 such that 𝑓¯(𝑥 1 ) = 𝑓¯(𝑥 2 ) = 2 . Show that 𝑑𝑌 (𝑓 (𝑥 1 ), 𝑓 (𝑥 2 )) ≤ 2.
(C) Show that for all sufficiently large 𝑅, 𝑑𝑌 (𝑓 (𝑥 1 ), 𝑓 (𝑥 2 )) > 2. (Hint: Let 𝑎𝑖 ∈ 𝐶𝑖 be such that
𝑑𝑋 (𝑎, 𝑎𝑖 ) = 𝑅/𝜆 2 ; show that 𝑑𝑋 (𝑥 1, 𝑥 2 ) ≥ 𝑑𝑋 (𝑎 1, 𝑎 2 ).)
(D) What is your conclusion about 𝑓 ?

4
CHENNAI MATHEMATICAL INSTITUTE
Postgraduate Programme in Mathematics
MSc/PhD Entrance Examination
7th May 2023

Part A
(1) Let 𝑅 be an integral domain containing C such that it is a finite-dimensional C-vector-space.
Pick the correct statement(s) from below.
(A) For every 𝑎 ∈ 𝑅, the set {1, 𝑎, 𝑎 2, . . .} is linearly dependent over C.
(B) 𝑅 is a field.
(C) 𝑅 = C.
(D) The transcendence degree of 𝑅 over C is 1.
(2) Let 𝑅 be a euclidean domain that is not a field. Let 𝑑 : 𝑅 \{0} −→ N be the euclidean size (degree)
function. Write 𝑅 × for the invertible elements of 𝑅. Pick the correct statements from below.
(A) 𝑅 = 𝑅 × ∪ {0}.
(B) There exists 𝑎 ∈ 𝑅 \ (𝑅 × ∪ {0}) such that 𝑑 (𝑎) = inf {𝑑 (𝑟 ) | 𝑟 ∈ 𝑅 \ (𝑅 × ∪ {0})}.
(C) With 𝑎 defined as above, for all 𝑟 ∈ 𝑅, there exists 𝑢 ∈ 𝑅 × ∪ {0} such that 𝑎 divides (𝑟 − 𝑢).
(D) With 𝑎 defined as above, the ideal generated by 𝑎 is a maximal ideal.
(3) Let 𝑋 be a compact topological space. Let 𝑓 : 𝑋 −→ R be a function satisfying 𝑓 −1 ( [𝑛, ∞)) is
closed for all 𝑛 ∈ N. Pick the correct statements from below.
(A) 𝑓 is continuous.
(B) 𝑓 (𝑈 ) is open for each open subset 𝑈 of 𝑋 .
(C) 𝑓 (𝑈 ) is closed for each closed subset 𝑈 of 𝑋 .
(D) 𝑓 is bounded above.
(4) Let 𝑓 : [0, 1] −→ R be a continuous function and 𝐸 ⊆ [0, 1]. Which of the following are true?
(A) If 𝐸 is closed, then 𝑓 (𝐸) is closed.
(B) If 𝐸 is open, then 𝑓 (𝐸) is open.
(C) If 𝐸 is a countable union of closed sets, then 𝑓 (𝐸) is a countable union of closed sets.
(D) If 𝑓 injective and 𝐸 is a countable intersection of open sets, then 𝑓 (𝐸) is a countable inter-
section of open sets.
(5) Consider the real matrix  
𝜆 2
𝐴= .
3 5
Assume that −1 is an eigenvalue of 𝐴. Which of the following are true?
(A) The other eigenvalue is in C \ R.
(B) 𝐴 + 𝐼 2 is singular.
(C) 𝜆 = 1.
(D) Trace of 𝐴 is 5.
(6) Let 𝑎𝑛 , 𝑛 ≥ 1, be a sequence of positive real numbers such that 𝑎𝑛 −→ ∞ as 𝑛 −→ ∞. Then which
of the following are true?
(A) There exists a natural number 𝑀 such that

∑︁ 1
∈ R.
𝑛=1
(𝑎𝑛 ) 𝑀
(B)

∑︁ 1
∈ R.
𝑛=1
(𝑛 2𝑎 𝑛)

(C)

∑︁ 1
∈ R.
𝑛=1
(𝑛𝑎𝑛 )
1
(D) For all positive real numbers 𝑅,

∑︁ 𝑅𝑛
∈ R.
𝑛=1
(𝑎𝑛 )𝑛
(7) Let 𝐴 be the ring of all entire functions under point-wise addition and multiplication. Then
which of the following are true?
(A) 𝐴 does not have non-zero nilpotent elements.
(B) In the group of the units of 𝐴 (under multiplication), every element other than 1 has infinite
order.
(C) For every 𝑓 ∈ 𝐴, there is a sequence of polynomials which converges to 𝑓 uniformly on
compact sets.
(D) The ideal generated by 𝑧 and sin 𝑧 is principal.
(8) Which of the following groups are cyclic?
(A) Z/2Z ⊕ Z/9Z
(B) Z/3Z ⊕ Z/9Z
(C) Every group of order 18.
(D) (Q× , ·)
(9) Let 𝑝, 𝑞 be distinct prime numbers and let 𝜁𝑝 , 𝜁𝑞 denote (any) primitive 𝑝-th and 𝑞-th roots of
unity, respectively. Choose all the correct statements.
(A) 𝜁 13 ∉ Q(𝜁 31 ).
(B) If 𝑝 divides 𝑞 − 1, then 𝜁𝑝 ∈ Q(𝜁𝑞 ).
(C) If 𝜁𝑝 ∈ Q(𝜁𝑞 ), then 𝑝 − 1 divides 𝑞 − 1.
(D) If there exists a field homomorphism Q(𝜁𝑝 ) −→ Q(𝜁𝑞 ), then 𝑝 − 1 divides 𝑞 − 1.
(10) Let 𝑓 , 𝑔 : R2 −→ R be functions. Let 𝐹 = (𝑓 , 𝑔) : R2 −→ R2 . Assume that 𝐹 is infinitely
differentiable and that 𝐹 (0, 0) = (0, 0). Suppose further that the function 𝑓 𝑔 : R2 −→ R is
everywhere non-negative. Then
(A) 𝑓𝑥 (0, 0) = 0, 𝑓𝑦 (0, 0) = 0.
(B) 𝑔𝑥 (0, 0) = 0, 𝑔𝑦 (0, 0) = 0.
(C) The image of 𝐹 is not dense in R2 .
(D) det 𝐽 (0, 0) = 0 where 𝐽 is the matrix of first partial derivatives (i.e., the jacobian matrix).

Part B
(11) Let 𝑓 : R ≥0 −→ R be the function
(
1, 𝑥 =0
𝑓 (𝑥) = −𝑥
𝑥 , 𝑥>0
Determine whether the following statement is true:
∫ 1 ∞
∑︁
𝑓 (𝑥)d𝑥 = 𝑛 −𝑛 .
0 𝑖=0

(12) (A) (3 marks) Let 𝐺 be a group such that |𝐺 | = 𝑝 𝑎𝑑 with 𝑎 ≥ 1 and (𝑝, 𝑑) = 1. Let 𝑃 be a Sylow
𝑝-subgroup and let 𝑄 be any 𝑝-subgroup of 𝐺 such that 𝑄 is not a subgroup of 𝑃. Show that
𝑃𝑄 is not a subgroup of 𝐺.
(B) (7 marks) Let Γ be a group that is the direct product of its Sylow subgroups. Show that every
subgroup of Γ also satisfies the same property.
(13) (A) (5 marks) Let 𝑛 ≥ 2 be an integer. Let 𝑉 be the R-vector-space of homogeneous real polyno-
mials in three variables 𝑋, 𝑌 , 𝑍 of degree 𝑛. Let 𝑝 = (1, 0, 0). Let
𝜕𝑓
𝑊 = {𝑓 ∈ 𝑉 | 𝑓 (𝑝) = (𝑝)}
𝜕𝑋
Determine the dimension of 𝑉 /𝑊 .
2
(B) (5 marks) A linear transformation 𝑇 : R9 −→ R9 is defined on the standard basis 𝑒 1, . . . , 𝑒 9
by
𝑇 𝑒𝑖 = 𝑒𝑖 −1, 𝑖 = 3, . . . , 9
𝑇 𝑒2 = 𝑒3
𝑇 𝑒1 = 𝑒1 + 𝑒3 + 𝑒8 .
Determine the nullity of 𝑇 .
(14) Let 𝐹 be a field and 𝑅 a subring of 𝐹 that is not a field. Let 𝑥 be a variable. Let 𝑆 = {𝑎 0 + 𝑎 1𝑥 + · · · +
𝑎𝑛 𝑥 𝑛 | 𝑛 ≥ 0 and 𝑎 0 ∈ 𝑅, 𝑎 1, · · · 𝑎𝑛 , ∈ 𝐹 }.
(A) (2 marks) Show that, with the natural operations of addition and multiplication of polyno-
mials, 𝑆 is an integral domain.
(B) (4 marks) Let 𝐼 = {𝑓 (𝑥) ∈ 𝑆 |𝑓 (0) = 0}. Determine whether 𝐼 is a prime ideal.
(C) (4 marks) Determine whether 𝑆 is a PID.
(15) (A) (6 marks) Let 𝑓 , 𝑔 : [0, 1] ↦→ R be monotonically increasing continuous functions. Show
that ∫ 1  ∫ 1  ∫ 1
𝑓 (𝑥)𝑑𝑥 𝑔(𝑥)𝑑𝑥 ≤ 𝑓 (𝑥)𝑔(𝑥)𝑑𝑥 .
0 0 0
(Hint: try double integrals.)
(B) (4 marks) Let 𝑓 : R −→ R be an infinitely differentiable function such that 𝑓 (1) = 𝑓 (0) = 0.
Also, suppose that for some 𝑛 > 0, the first 𝑛 derivatives of 𝑓 vanish at zero. Then prove that
for the (𝑛 + 1)th derivative of 𝑓 , 𝑓 (𝑛+1) (𝑥) = 0 for some 𝑥 ∈ (0, 1).
(16) (A) (5 marks) Consider the eucliean space R𝑛 with the usual norm and dot product. Let x, y ∈ R𝑛
be such that
∥x + 𝑡y∥ ≥ ∥x∥, for all 𝑡 ∈ R.
Show that x · y = 0.
(B) (5 marks ) Consider the vector field 𝑣® = (𝑣 𝑥 , 𝑣 𝑦 ) (with components (𝑣 𝑥 , 𝑣 𝑦 )) on R2 :
𝑣 𝑥 (𝑥, 𝑦) = 𝑥 − 𝑦, 𝑣 𝑦 (𝑥, 𝑦) = 𝑦 + 𝑥
Compute the line integral of 𝑣® along the unit circle (counterclockwise). Is there a function 𝑓
such that 𝑣® = grad𝑓 ?
(17∗ ) Denote by 𝑉 the Q-vector-space Q[𝑋 ] (polynomial ring in one variable 𝑋 ). Show that 𝑉 ∗ is not
isomorphic to 𝑉 .
(18∗ ) Let 𝑓 be a non-constant entire function with 𝑓 (0) = 0. Let 𝑢 and 𝑣 be the real and imaginary
parts of 𝑓 respectively. Let 𝑅 > 0 and
𝐵 = sup{𝑢 (𝑧) : |𝑧| = 𝑅}.
(A) (2 marks) Show that 𝐵 > 0.
(B) (2 marks) Consider the function
𝑓 (𝑧)
𝐹 (𝑧) := .
𝑧 (2𝐵 − 𝑓 (𝑧))
Show that 𝐹 is analytic on the open ball with radius 𝑅 and continuous on the boundary {𝑧 :
|𝑧| = 𝑅}.
(C) (3 marks) Show that sup{|𝐹 (𝑧)| : |𝑧| = 𝑅} ≤ 𝑅1 .
(D) (3 marks) Show that  
𝑅
sup |𝑓 (𝑧)| : |𝑧| = ≤ 2𝐵.
2
(19∗ ) Let 𝑈 (𝑛) be the group of 𝑛 × 𝑛 unitary complex matrices. Let 𝑃 ⊂ 𝑈 (𝑛) be the set of all finite
order elements of 𝑈 (𝑛), that is, 𝑃 = {𝑋 ∈ 𝑈 (𝑛) | 𝑋 𝑚 = 1 for some 𝑚 ≥ 1}. Show that 𝑃 is dense
in 𝑈 (𝑛).
(20∗ ) Let 𝐴 be a non-trivial subgroup of R generated by finitely many elements. Let 𝑟 be a real number
such that 𝑥 −→ 𝑟𝑥 is an automorphism of 𝐴. Show that 𝑟 and 𝑟 −1 are zeros of monic polynomials
with integer coefficients.
3
CHENNAI MATHEMATICAL INSTITUTE
Graduate Programme in Mathematics - M.Sc./Ph.D.
Solutions to the Entrance Examination, 2012
Part A

1. True. Take δ = . Then k(x1 , x2 · · · xn ) − (y1 , y2 · · · yn )k2 < δ implies that |xi − yi | < 
for all i = 1, 2 · · · n, and hence M ax{|xi |} < . So f is uniformly continuous.

2. True. If not, there exists an 0 > 0, such that for all k, there there exists an nk > k
satisfying |xk −x| ≥ 0 . The subsequence xnk does not have any subsequence converging
to x.

3. False. Consider f (x) = x1 . Then { n1 } is Cauchy but {n} is not Cauchy.

4. Trues. Since fn converges to f uniformly, f is also continuous. So f (xn ) converges to


f (x). Now given  > 0

|fn (xn ) − f (x)| ≤ sup |fn (x) − f (x)| + |f (xn ) − f (x)|.


x∈R

5. True. If K is unbounded then the function f (x) = kxk for x ∈ Rn is continuous and
unbounded. On the other hand if K is not closed, that is there exists a sequence
1
{xn } ⊆ K such that xn 7→ x and x ∈ K, then the function f (x) = kx−x nk
is continuous
and unbounded.

6. True. It suffices to show that X := R2 \ A is path-connected. Let x, y ∈ X. There are


uncountably many lines passing through either of the points x or y; only countably
many can go though a point in A; so there are lines lx though x and ly through y such
that lx ⊆ X and ly ⊆ X and lx ∩ ly 6= ∅.

7. False, as for any z ∈ C, there exists a w such that z 2 + w2 = 1

8. True, as the zeros of f − g are isolated.

9. True. 121 is a prime power: 121 = 112 .


22 22
10. True. The minimal polynomial A is (t − π)(t − 7
). It has distinct roots since π 6= 7
.

11. False: take the subgroup 5Z ⊂ Z, which is of index 5. (Z being the group of integers).

12. True: any group can be embedded in Sn for some n (by Cayley’s theorem) and we
embed Sn in an alternating group as follows: i : Sn ⊂ An+2 defined as follows: — if
σ ∈ Sn is even define i(σ) = σ, — if σ is odd define i(σ) = σ.(n + 1, n + 2) and hence
in both case i(σ) is even.

13. False: S3 is a group of order 6 which is non-abelian.

1
14. False: there are two non-isomorphic groups of order 4 and these are abelian, one is
cyclic and the other a direct product of Z2 with itself.

15. False. For any x 6= y the interval (x, y) is connected and hence its image under the
continuous function f . If f (x) 6= f (y) then the interval contains irrational points and
contained in the image of f , since any connected subset of R is an interval,

2
Part B
Each question carries 10 marks. Try to answer 5 questions.

1. Let f (t) = (f1 , f2 (t), f3 (t)) then f 0 (t) = (f10 , f20 (t), f30 (t)). kf (t)k = 1 implies that
f1 (t)2 + f2 (t)2 + f3 (t)2 = 1
for all t ∈ R. Differentiating we get
2f1 (t)f10 (t) + 2f2 (t)f20 (t) + 2f3 (t)f30 (t) = 0
and hence hf (t), f 0 (t)i = 0.
2. Since addition is continuous, B is open implies a + B is open for any fixed a ∈ A. Now
[
A+B = (a + B) .
a∈A

Since arbitrary union of open sets is open A + B is open. Note that it is enough if
either A or B is open.
For closed sets this is not true. Take A = {(a, 0) : a ∈ R} and B = {(b, 1b ) : b ∈ R \{0}}.
Then both A, B ⊆ R2 are closed. But A + B = {(a + b, 1b ) : a ∈ R, b ∈ R \ {0}}. The
sequence {(0, n1 )} = {(n − n, n1 } ⊆ A + B but the limit (0, 0) ∈
/ A + B.
3. We assume X and Y to be metric spaces. Let U ⊆ Z be open, then (g ◦ f )−1 (U ) =
f −1 (g −1 (U ) is open since g ◦ f is continuous. Since X is compact and f is continuous,
f is an open map ((i.e) f takes open sets to open sets. This is because continuous
functions takes compact subsets to compact subsets, so when the space is compact, it
takes closed sets to closed sets.) Now since f is onto, we have g −1 (U ) = f (f −1 (g −1 (U ))
and is open. Hence g is also continuous.
4. It is clear that B is a symmetric matrix and thus B can be written as B = QΛQt
where Λ is a diagonal matrix and Q is an orthogonal matrix (with Q−1 = Qt ). Now it
can be seen that and B j = QΛj Qt , Ck = QDk Qt where
k
Λj .
X
Dk = I +
j=1

Since Λ is a diagonal matrix, say


Λ = diag(λ1 , λ2 , . . . λn )
it follows that
Dk = diag(φk (λ1 ), φk (λ2 ), . . . φk (λn ))
where φk (x) = 1 + x + . . . xk . Since B = AAt , it follows that the eigenvalues λj of B
are non negative and thus α = maxi λi .
Since φk (x) converges if and only if |x| < 1 it follows that Dk (and hence Ck ) converges
if and only if α < 1.

3
5. Without loss of generality, assume that z = 1 is a pole. Take z = r real and let
r → 1− . Now |f (z)| ≤ |an |rn and since an can be made arbitrarily small for all large
P

n, conclude that f (r) = ◦(1 − r)−1 as r → 1− . This contradicts that z = 1 is a pole.


The statement is false if (an ) is only bounded, take an = 1 for all n, for instance.

6. Apply Schwarz’s lemma to conclude that any automorphism f of the unit ball is neces-
z−a
sarily of the form f (z) = c 1−az where |c| = 1. Since origin is mapped to itself, conclude
that the map is a rotation.

7. (i) Note that number of elements in G is (p − 1)2 p(p + 1). Hence any subgroup of G
having exactly p elements is a Sylow-p subgroup of G.
!
1 x
Now let H = { |x ∈ Fp }. Since |H| = p, H is a Sylow-p subgroup of G.
0 1
Let N be the group of all upper triangular matrices in G. Clearly H is a normal
subgroup of G.
!
a b
If ∈ NG (H), the normalizer of H in G, then
c d
! ! ! !
a b 1 1 1 y a b
= , for some y ∈ Fp .
c d 0 1 0 1 c d
!
a b
Comparing the (2,2) entry both sides, we get c = 0. So ∈ N . This proves
c d
(i).
(ii) Let S denote the set of all one-dimensional subspaces of F2p . Then |S|
!
= p + 1.
a
Consider the action of G on S given by left multiplication: A ∈ G, ∈ S, then
b
! !
a a
A· is defined to be the matrix multiplication A ∈ S.
b b
This action of G on S is!transitive. Moreover, the stabilizer of the
! one-dimensional
! sub-
!
1 1 a b 1
space spanned by is N . Indeed, clearly N stabilizes . If =
0 0 c d 0
! !
y a b
, then a = y and c = 0. So ∈ N.
0 c d
Hence we conclude that |G/N | = |S| = p + 1.
√ 2πi 2 4πi πi πi
8. Let α = 2e 3 and β = −α 2
. Then β = −e 3 = −eπi e 3 = e 3 . So β 3 = −1 and β
satisfies the polynomial Y 3 + 1. Hence the minimal polynomial of β is Y 2 − Y + 1.
2 2 α2
Thus α satisfies the equation ( −α
2
) + 2
+ 1 = 0. So α is a root of the polynomial
W4 W2
4
+ 2 + 1.

4
Note that the degree of the minimal polynomial of α can not be smaller than 4. Indeed,
2 divides the degree and α does not satisfy a quadratic equation.
Clearing the denominators we conclude that the minimal polynomial of α is X 4 +2X 2 +
4.

9. Embed G inside a suitable symmetric group; represent the symmetric group as the set
of permutation matrices.

10. Let n = dimF R and αi , 1 ≤ i ≤ n be an F -basis of R. Let β 6= 0 ∈ R. Consider


the vectors αi β, 1 ≤ i ≤ n. They must be linearly independent; for, otherwise, there
will be a nonzero element γ ∈ R such that βγ = 0. Hence they form an F -basis of R.
Therefore there exists γ ∈ R such that βγ = 1.

5
MSC/PHD MATHEMATICS ANSWERS

Part A
(1) a, b
(2) a, b, d
(3) a
(4) c
(5) b, d
(6) c, d
(7) c, d
(8) a, c
(9) a
(10) a, d
(11) d
(12) b
(13) b, c
(14) a, c, d
(15) b, d

Part B
(1) Let H and X be the subgroups of G generated by the elements h and x, respectively. The given
equation implies that H acts on X by conjugation; in other words, we have a homomorphism
φ : H → Aut(X), where Aut(X) is the group of automorphisms of X. This latter group has order
p − 1, whence φ is the trivial homomorphism. This means that hxh−1 = x. But we are given that
hxh−1 = x10 . It follows that x9 is the identity element, so p = 3.
(2) (a) Note that T 7 − 1 = (T − 1)(T 3 + T 2 + 1)(T 3 + T + 1). Consider the field K = F2 [T ]/(T 3 + T + 1).
It has a basis 1, T, T 2 over F2 . Multiplication by T on K is F2 -linear, so it can be represented
by a 3 × 3 matrix over F2 . In the above order of the basis vectors, it is
 
0 0 1
A = 1 0 1 .
0 1 0
Note that A3 + A + I3 = 0, so A7 = I3 .
(b) (i) No. For example 0x = 2 has a solution modulo 2 but does not have a real solution.
(ii) Yes. Suppose that the system does not have real solutions. Then rank(A) < rank([A|b]),
where [A|b] denotes the augmented matrix. Denote these two ranks by r and s respectively.
Then there is an s × s submatrix of [A|b] that is invertible; let its determinant (which is
an integer) be d. Then for every prime p > d, the corresponding s × s submatrix of [A|b]
has an invertible determinant modulo p, i.e., rank([A|b]) = s mod p for all large enough
primes p. On the other hand, rank of any integer matrix modulo a prime number can
only be at most its rank considered as a real matrix, so for all primes p, rank(A) ≤ r
mod p for all primes p. Therefore for all sufficiently large primes p, rank(A) < rank([A|b]),
mod p contradicting the hypothesis that there is solution modulo every prime.
(3) Let A ∈ Mn (C). We want to show that for every  > 0, the -ball around A contains a diagonalizable
matrix. First note that this is true for A if and only if it is true P AP −1 for every invertible
P ∈ Mn (C); therefore we may assume that A is in its Jordan canonical form. For 1 ≤ i ≤ n, pick
0 < δi < n , all distinct. Let B be the sum of A and the diagonal matrix with δ1 , . . . , δn on the
1
diagonal. We claim that B is diagonalizable. Indeed, the eigen-values of B are ai,i + δi , 1 ≤ i ≤ n.
We may choose the δi to further satisfy that these are distinct, thus making B diagonalizable.
(4) The integrand extended to a function on the complex plane has simple poles at ±2i, and −1 ± i. We
shall compute the integral by contour integration. Let Cn be the contour given by the square with
vertices (−n, 0), (n, 0), (n, n), (n, −n), and denote by In the value of the contour integral about Cn
(traversed counterclockwise). On one hand, for n  0, In = In+1 = 2πi{sum of the residues in Cn }.
On the other hand, limn→∞ In converges to the value of the required integral on the real line as the
integrand goes to zero on the three sides of Cn not on the real line, as n → ∞.
(5) By way of contradiction, suppose that there exists such an analytic function f . Then f has an
P∞ (n)
expansion as a convergent power series around 0. It is of the form f (z) = n=0 f n!(0) z n . From
the given information, we can, however, conclude that f (n) (0) = 0 for all n ≥ 0; therefore, f ≡ 0,
contradiction.
(6) The function satisfies the hypotheses of Lagrange’s Mean Value Theorem over [0,1] and [1,2], using
which we’ll prove that the difference quotient ∆(h) = [f (1 + h) − f (1)]/h has a limit as h → 0. Let
 > 0 be given. Then, ∃δ > 0 such that |f 0 (x) − 2013| <  whenever 0 < 1 − x < δ. For −1 < h < 0,
the MVT says that ∃c ∈ (0, 1) such that ∆(h) = f 0 (c). But then |∆(h) − 2013| <  whenever
0 < −h < δ. This says that limh→0− ∆(h) = 2013. One similarly shows that limh→0+ ∆(h) = 2013.
(7) (a) By way of contradiction, suppose that f and f −1 are differentiable. Then Df ◦D(f −1 ) : R3 → R3
is D(idR3 ) = I3 at every point in R3 . However, rank(Df ) ≤ 2 at every point in R2 , contradiction.
(b) No. Consider f : R → R, t 7→ et . Then (Df )(x) = ex for all x ∈ R. Therefore (Df )(x) induces
an isomorphism of of tangent spaces for all x.
(8) (a) For a prime p, set S(p) := {a ∈ Z : a ≥ 1 and there exist b ∈ Z, b ≥ a and n ∈ Z such that pa |
f (pb n)}. By hypothesis S(p) is empty except for finitely many p, so for some p, S(p) is infinite.
Choose such a p. Then for all a ∈ S(p), there exist b, n such that pa | f (pb n) so pa | f (0) since
f (0) = f (x) − xg(x) for some g(x) ∈ Z[x]. Therefore f (0) = 0, a contradiction since q | f (q) for
all primes q.
(b) Let F be a finite field; denote its group of units by F × . Let n = |F × |. Let d > 0 be a divisor
of n. We want to show that there exists at most one subgroup of F × of order d. By way of
contradiction, suppose that there are two distinct subgroups G and H of F × of order d. Note
that for all x ∈ G ∪ H, xd = 1, so in F , there at least d + 1 elements that satisfy the polynomial
T d − 1 = 0, a contradiction.
(9) Suppose, by way of contradiction, that K admits a disconnection, that is open subsets U, V ⊂ R2
such that K ⊂ U ∪ V , U ∩ K 6= ∅, V ∩ K 6= ∅, but U ∩ V ∩ K = ∅. Let Ki0 = Ki \ (U ∩ V ),
i ≥ 1. If Ki0 6= ∅ ∀i, then by the Finite Intersection Property, ∩∞ 0
i=1 Ki 6= ∅, contradicting the fact
that K ⊂ U ∪ V . So, ∃n such that Kn ⊂ U ∪ V . Similarly, working with Ki00 = Ki \ (U ∪ V ) we can
show that ∃m such that U ∩ V ∩ Km = ∅. If we set N = max{m, n}, then U, V form a disconnection
of KN , contradicting the hypothesis that the Ki are connected.
(10) It suffices to show that A is bounded and closed. The projection maps (x, y) 7→ x and (x, y) 7→ y
are continuous. Therefore A is bounded. Since every continuous R-valued function on A attains
its maximum, it also attains its minimum. Let p be any limit point of A. The continuous function
A → R, q 7→ d(q, p), where d is the usual metric on R2 attains its minimum, so p ∈ A. Hence A is
closed.

2
MSC/PHD MATHEMATICS 2014 SOLUTIONS

Part A

1. B, D.

2. A, B.

3. B.

4. B, C.

5. C, D.

6. B, C.

7. A.

8. A, B, D.

9. 2.

10. 2.
Part B
3
(11) The set of rank 2 matrices in M2×3 (R) is open: Consider the map f : M2×3 (R) → R
given by sending a matrix to the triple of its 2 × 2 minors. This is a continuous map.
3
The set of rank 2 matrices is the inverse image of the set {(x1 , x2 , x3 ) ∈ R | xi 6=
0 for some 1 ≤ i ≤ 3}. This set is open in R3 , hence the set of rank 2 matrices is open
in M2×3 (R).

(12) (A) The kernel of φ is a proper ideal in F . Hence it is zero, as there are no nonzero
proper ideals in a field. Since φ(1) = 1, using properties of a field homomorphism
we conclude that φ(r) = r for every r ∈ Q. Now if r ∈ Q and x ∈ F , then
φ(rx) = φ(r)φ(x) = rφ(x). Thus φ is a homomorphism of vector spaces over Q.
Since φ is injective and dimension of F over Q is finite, it follows that φ is also
surjective. Thus φ is a field isomorphism.

1
(B) Consider the map φ : F × → F × defined by φ(x) = x2 . Then φ is a group
homomorphism and the set of squares in F × is the image of φ. The kernel of φ
is the set of all x ∈ F × such that x2 = 1. Since F is a field this equation has at
most 2 solutions. Further since the characteristic of F is different from 2, it has
exactly two solutions. Hence the kernel of φ contains two elements and we have
an isomorphism F × /ker(φ) ∼ = im(φ). Hence the cardinality of im(φ) is half of the
cardinality of F × .
(13) (a) For A ∈ Mn (C), det(A) is a polynomial in the entires of A, so all the multiple
partial derivatives exist and are continuous.
(b) Let Aij ∈ Mn−1 (C) be the matrix with ith row and jth column removed from A.
Then the total derivative d(det) is the matrix in Mn (C) whose (i, j)-th entry is
(−1)i+j det(Aij ).
(c) d(det)=0 if and only if det(Aij ) = 0 for all i, j. This is equivalent to rank(Aij ) ≤
n − 2 for all i, j. We prove that this is equivalent to rank(A) ≤ n − 2.
Suppose rank(A) ≤ n − 2. This means the rows (or columns) of A have at most
n − 2 linearly independent vectors. After removing a row and column there will
be at most n − 2 linearly independent vectors. Hence rank(Aij ) ≤ n − 2 for all
i, j. Conversely if rank(A) > n − 2, the rows of A have at least n − 1 linearly
independent vectors. Choose n − 1 linearly independent row vectors and form
a matrix of size (n − 1) × n. Then n − 1 columns of this matrix are linearly
independent. So we can remove one column and the remaining (n − 1) × (n − 1)
matrix will still have (n − 1) linearly independent vectors. So there exits a pair
(i, j) such that rank(Aij ) = n − 1.
Hence d(det)(A)=0 if and only if rank(A) ≤ n − 2.

(14) Let F0 = {i | ai 6= 0} then F0 = ∪∞n=1 Fn . Each Fn is a finite set, otherwise


X X N
sup{ ai : F ⊆ R finite subset} ≥ sup{ ai : F ⊆ Fn finite subset} ≥ ∀N ∈ N.
i∈F i∈F
n

We can not replace countability by finiteness, since any convergent series with infinitely
many non-zero entries will satisfy the condition. Take for example ai = i12 for i ∈ N
and ai = 0 for i ∈ R \ N.
(15) Since the order of G is divisible by 2, G has an element x of order 2, by Cauchy’s
theorem. Since x 6= 1 and φ(x) = 1, φ is not injective. As φ is a function from a finite
set to itself, it can not be surjective as well.
(16) Let zn = 2πin. Then |zn | → ∞ and ezn → 1. Suppose g(z) = f (ez ) is a non-constant
polynomial. We have g(2πni) = f (1) for all n ∈ N. So g − f (1) has infinitely many
zeros and hence g = f (1). Thus f is equal to the constant f (1) on C \ {0}, which is
the range of the function ez . It follows by continuity that f is constant on all of C.

2
(17*) (A) On (R) is compact : Consider the map f : Mn×n (R) → Mn×n (R) given by f (A) =
t −1
AA − In . This map is continuous and On (R) is closed since it is equal to f (0).
t
On (R) ⊂ Mn×n (R) is bounded: the condition AA = In expressed in terms  of the 
a b
entries of A gives (for example when n = 2) the following: let A = .
c d
2 2 2 2
The we get a + b = 1 = c + d , and ac + bd = 0. This implies |a|, |b|, |c|, |d| ≤ 1.
(B) Consider det : On (R) → {1, −1}. This is continuous and surjective. This shows
that On (R) is disconnected.
(C) On (C) is not compact, since it is not bounded as a subset of Mn×n (C). The
conditions expressed above for a, b, c, d when the entries are complex numbers
show this. For example, fixing a, |a|  0, there are solutions for b such that
2 2
a + b = 1. Similarly for c, d.

(18*) f is given entire. ExpandPit as a power series in a neighbourhood of 0. f = ∞


P n
n=0 rn .z .
We are given: f (aj ) = ∞ n
n=0 rn .aj = bj for all j. Taking limits as j → ∞ on either
side and using uniform convergence, we see that f (0) = lim bj . But we are given
j→∞
b
lim ajk = 0 for all k ≥ 0. Taking k = 0, we get lim bj = 0, hence r0 = f (0) = 0.
j→∞ j j→∞
P∞ n bj
Now write f (z) = n=1 rn .z . Proceeding as before and using lim ak = 0 for k = 1,
j→∞ j
we get r1 = 0. Similarly rj = 0 for all j and hence f = 0 in a neighbourhood of 0.
Since f is entire, we get f = 0 everywhere and bn = 0 for all n.

(19*) Following Cayley’s theorem, use the action of G on the set of left cosets G/H. This
n
gives a homomorphism f : G → SN , where N = m = |G/H|.
|S |
Since |SN | = N ! < 2n, it implies that 2N < n = |G|. If f is injective, then by the
above inequality, f is actually an isomorphism. In this case, G is not simple because
SN is not, as it has the alternating group AN as a normal subgroup. On the other
hand, if f is not injective, it has a non-trivial kernel K ⊂ G. K is a proper nonzero
normal subgroup of G, hence again G is not simple.

(20*) Let φ : R → R be a continuous function such that |φ(x)| → ∞ as |x| → ∞. Then by


the formula for the limit of composition of two maps, one gets f ◦ φ ∈ C0 (R) for every
f ∈ C0 (R).
Suppose that f ◦ φ is infinitely differentiable for every f in C0∞ (R). Let a ∈ R. The
image φ[a − 1, a + 1] is a compact interval. Choose f ∈ C0∞ (R) such that f = 1 on
φ[a − 1, a + 1]. Then f ◦ φ = φ on (a − 1, a + 1). Thus φ agrees with the infinitely
differentiable function f ◦ φ on the neighbourhood (a − 1, a + 1) of a. Hence φ is
infinitely differentiable in a neighbourhood of a. As a is an arbitrary point of R, φ is
differentiable.

3
2015 MSc/PhD Mathematics Solutions
Part A
(1). B, D.
(2). B.
(3). A, B, C, D.
(4). A, C.
(5). C, D.
(6). A, C.
(7). B, C.
(8). 2.
(9). 0.
(10). 0.

Part B
(11). After changing coordinates, we may assume that f vanishes in an open neighbourhood
of the origin. Hence
∂n
 
f (0, 0) = 0
∂xi ∂y n−i
for every n ≥ 0 and 0 ≤ i ≤ n. Therefore all the coefficients of f are zero, so f = 0.
(12). (a). Let v 6= 0 ∈ ker A, so (In + A)v = v; hence 1 is an eigenvalue of (In + A).
Conversely, suppose that (In + A)v = λv for some nonzero v. If Av = 0, then v = λv, so λ = 1.
Otherwise, multiplying on the left by A, we see that Av = λ(Av); hence, again, λ = 1.
(b). A2 + 1 = 0, so the minimal polynomial of A divides t2 + 1 which has distinct roots.
Hence the minimal polynomial of A has distinct roots, so A is diagonalizable.
(13). For r ∈ R, r > 0, let Cr := {z ∈ C : |z| = r}, Ur := {z ∈ C : |z| < r} and
r = min{|f (z)| : z ∈ Cr }. As f it nonconstant, the origin is an isolated zero of f . Choose r > 0
such that the origin is the only zero in Cr ∪ Ur . As the set {z : |f (z)| < r } intersects Ur but
not Cr , it must be in Ur , by the second condition on f . Hence the origin is the only zero of f .
Now letting r −→ ∞, we see that for all z 6∈ Ur , |f (z)| ≥ r . Hence f has a pole at infinity, i.e.,
there exists a non-negative integer m (z −1 ) is analytic at z = 0. Suppose that f
P∞ mi such thatmz f−1
is a power-series expression i=n ai z . Since z f (z ) is analytic at z = 0, we see that ai = 0
for all i ≥ m. Hence f is a polynomial that vanishes exactly at z = 0. Thus f = cz n for some
positive interger n and non-zero c ∈ C.
(14). For each positive integer k, using a), choose Nk such that

|bk − am,k | < ∀m ≥ Nk .
2
Let mk = N1 + · · · + Nk . Then mk is increasing. Now, using b), choose a positive integer K,
such that

|amk ,k − 1| < ∀k ≥ K.
2
By triangle inequality
|bk − 1| <  ∀k ≥ K.
(15). May assume that f is homogeneous of degree d. Induct on d. If xd appears in f ,
then so does y d (with the same coefficient. Hence we may write f = α(xd + y d ) + xyf1 (x, y),
where α could be zero. Note that f1 (x, y) = f1 (y, x), so by induction, there exists g1 such
that f1 (x, y) = g1 (x + y, xy). Write xd + y d = (x + y)d − xyf2 (x, y); f2 (x, y) = f2 (y, x), so by
induction, there exists g2 such that f2 (x, y) = g2 (x + y, xy).
(16). Note that for every a ∈ [0, 1] and for every open subset U of X containing f −1 (a),
there is an open neighbourhood W (for example, take W = [0, 1] \ f (X \ U )) of a such that
f −1 (W ) ⊆ U . Now let Ui , i ∈ I be an open
S covering of X. For every a ∈ [0, 1], there exists a
finite subset Ia of I such that f −1 (a) ⊆ i∈Ia Ui . Let Wa be an open neighbourhood of a such
1
that f −1 (Wa ) ⊆ i∈Ia Ui . There exist finitely many a1 , . . . , an such that [0, 1] = nj=1 Waj .
S S

Then X = nj=1 f −1 (Waj ) = nj=1 i∈Ia Ui . This is a finite subcovering of the given open
S S S
j
covering.
(17∗ ). For each such P , let RP be the smallest subring of Q that contains { p1 |p ∈ P }. If
P 6= Q, then RP 6= RQ . The set of all the primes is countably infinite, and the power-set of a
countably infinite set is uncountable. Hence Q has uncountably many subrings.
(18∗ ). Let |x| ≤ M . Since sin x ≤ x,
X sin x X 1
n
f (x) = ≤M < ∞,
n n2
n≥1 n≥1
and hence f is well-defined.
P sin x
Also the same argument shows, if we set fN = N ≥n≥1 n n , then fN converges uniformly
to f as N 7→ ∞ in the interval (−M, M ), and hence f is continuous. Since M is arbitrary f is
continuous everywhere.
x
cos n
Since cos nx ≤ 1, g is well-defined, and
P
Again let |x| ≤ M and we set g = n≥1 n2
x
0 =
P cos n
fN N ≥n≥1 n2 uniformly converges to g, by the same arguments. Since fN converges
uniformly to f and since fN 0 converges uniformly to g in the interval (−M, M ), f 0 = g. Since

M is arbitrary, f is differentiable everywhere.


(19∗ ). By the usual computation, pm(m−1)/2 is the largest power of p dividing the order
of GLm (Fp ). Also, one may check directly that U is a subgroup of GLm (Fp ) and its order is
pm(m−1)/2 . Therefore, U is a Sylow p-subgroup of GLm (Fp ). Now, by Sylow’s Theorem 2 and
its Corollary, there exists A ∈ GLn (Fp ) such that AGA−1 ⊂ U .
(20∗ ).
Let A ∈ Mm×n (C) be the matrix that consists of 1s on the diagonal on rows and columns
1, . . . , k and 0s elsewhere. Then the space of all matrices of rank k inside Mm×n (C) is
X := {B1 AB2 : B1 ∈ GLm (C); B2 ∈ GLn (C)} ;
we need to show that this space is connected. Fix B2 ∈ GLn (C). Then the space
XB2 := {B1 AB2 : B1 ∈ GLm (C)}
is connected since it is a continuous image of the connected space GLm (C). Similarly, the space
Y := {AB2 : B2 ∈ GLn (C)}
is connected. Since [
X=Y ∪ XB2
B2 ∈GLn (C)
and each XB2 intersects Y , we see that X is connected.

2
CHENNAI MATHEMATICAL INSTITUTE
Postgraduate Programme in Mathematics
MSc/PhD Entrance Examination
18 May 2016
Solutions

Part A
(1) A,D.
(2) A.
(3) A,C.
(4) B, C.
(5) A.
(6) B.
(7) A, B, C, D.
(8) A, C, D.
(9) 1.
(10) 1.

Part B
(11) In polar coordinates, the region U is given by U 0 := (1, 2) × [0, 2π). Write p0 and q 0 for
the points in U 0 corresponding to p and q respectively. Let γ 0 : [0, 1] −→ U 0 be given by
t 7→ t · p0 + (1 − t) · q 0 . This is continuous, and is such that γ 0 (0) = p0 and γ 0 (1) = q 0 and
such that γ 0 is differentiable on (0, 1). Let f be the map (converting polar coordinates
to Cartesian coordinates),
R × [0, 2π) −→ R2 , (r, θ) 7→ (r cos θ, r sin θ).
Let γ = f ◦ γ 0 . Since f is differentiable, γ has the desired properties.
(12) By way of contradiction, suppose that IJ is a prime ideal. In an integral domain, a
product of two non-zero ideals is non-zero, so IJ is a non-zero prime ideal. In a PID,
every non-zero prime ideal is maximal, so IJ is maximal. Since IJ ⊆ I ∩ J, we conclude
that I = IJ = J. Let a be a generator for I and J; hence IJ is generated by a2 .
Therefore we see that a ∈ (a2 ). Write a = ba2 , so a(1 − ba) = 0. Since a 6= 0, ba = 1,
i.e., a is a unit, so I is not a proper ideal, a contradiction.
(13) Write z = x + ıy with x, y ∈ R and f (z) = u(x, y) + ıv(x, y), where u and v are maps
from R2 to R. Then v(0, t) = 0 = v(t, 0) for every t ∈ R. Therefore
∂v ∂v
(0, 0) = 0 = (0, 0).
∂x ∂y
Since f is entire,
∂u ∂u
(0, 0) = 0 = (0, 0).
∂x ∂y
Therefore f 0 (0) = 0.
(14) Let  > δ, and consider the closed ball
B := {y ∈ Rn |d(x, y) ≤ }.
There exists y ∈ A such that d(x, y) ≤  so
δ = inf{d(x, y) | y ∈ A ∩ B}.
Since A∩B is closed and bounded, it is compact. Consider the function f : A∩B −→ R,
y 7→ d(x, y). It is continuous, so it attains its infimum and supremum, i.e., there exists
y ∈ A ∩ B ⊆ A such that δ = d(x, y).
(15) (A): Fix a basis v1 , . . . , vn of V . Let n1 , . . . , nd be such that T ni vi = vi . Let n = lcm{ni |
1 ≤ i ≤ d}. Then T n (v) = v for every v ∈ V  , so, over C, the minimal polynomial of T
1 1
has distinct roots. For (B), take T = over F2 .
0 1
(16) [Q(ω) : Q] = 2, since the minimal polynomial √ of ω is X 2 + X + 1. A basis √ √of Q(ω)
over Q is {1, 3 3 3 3
√ ω}. The minimal polynomial √ √of 2 over √ Q√is X − 2, so {1, 2, 4} is a
basis of Q( 3 2) over Q. Therefore {1, 3 2, 3 4, ω, ω 3 2, ω 3 4} span F as Q-vector-space.
Since X√ 3 − 2 is irreducible over Q(ω), [F : Q(ω)] = 3, and, therefore [F : Q] = 6. Hence
√ √ √
{1, 2, 3 4, ω, ω 3 2, ω 3 4} is a Q-basis for F . Since ω 2 = −(1 + ω) and ω 3 = 1, we see
3

that the matrix of µ with respect to the above basis (in the given order) is
 
−1 0 0 1 0 0
 0 −1 0 0 1 0
 
0
 0 −1 0 0 1 .
−1 0 0 0 0 0
 
 0 −1 0 0 0 0
0 0 −1 0 0 0
(17∗ ) Suppose that G is not dense. Let l := inf{x ∈ G | x > 0}. We first show that l > 0
and that l ∈ G. If l = 0, then there exists a small open neighbourhood U of l = 0 that
contains non-zero elements of G; Now, for every g ∈ G, {g + u | u ∈ U } is an open
neighbourhood of g that contains elements of G other than g, so G is dense; hence l > 0.
Now, if l 6∈ G, then there exists  > 0 such that (l, l + ) ⊆ G. We may assume, without
loss of generality, that  < l. Let x, y ∈ (l, l + ), with x < y. Then y − x ∈ G and
0 < y − x < . This contradicts the choice of l, so l ∈ G.
Now let g ∈ G, and let n be the largest integer such that nl ≤ g < (n + 1)l. Hence
0 ≤ g − nl < l, so by the minimality of l, g = nl, i.e., G = Z · l.
(18∗ ) Let φ : X −→ C be the constant function taking the value 1. Then for every g ∈ G,
(g · φ)(x) = φ(g −1 (x)) = 1 = φ(x) for every x ∈ X, so g · φ = φ for every g ∈ G. Let
X
F 0 := {f ∈ F | f (x) = 0}.
x∈X

It is a subspace of F . To show that F = F 0 ⊕ Chφi, we need to show 0


P that F ∩ Chφi = 0
0 0
and that F = F + Chφi. If αφ ∈ F for some α ∈ C, then 0 = x∈X (αφ)(x) = α|X|,
so α = 0, i.e., F 0 ∩ Chφi = 0. Let f ∈ F . Set α = |X| 1 P
x∈X f (x). Then f − αφ ∈ F ,
0

so F = F 0 + Chφi.

(19 ) (A) Write A = (aij ) and B = (bij ). If aij > 1 then for every k, bjk = 0, for otherwise,
the (ik)th entry of AB would be greater than 1. However, if there exists j such
that for every k bjk = 0, then B is not invertible. Hence aij ∈ {0, 1} for every i, j.
Similarly bij ∈ {0, 1} for every i, j. Now suppose that aij = aik = 1 with j < k.
Then exactly one of bji , bki is 1 and for l 6= i, bjl = bkl = 0. Therefore either the
jth row or the kth row of B is zero, a contradiction. Hence, for every i, there is a
unique ki such that aiki = 1. Therefore A is a permutation matrix. Since B = A−1 ,
B is a permutation matrix.
(B) Let T : Cn −→ Cn be the linear transformation given by ei 7→ Aei , where the ei are
the standard basis of Cn . Let v ∈ Cn be a non-zero vector that is not an eigenvector
of A (and of T ). (Such a vector exists by the hypothesis on A.) Then v and Av are
linearly independent. Extend it to a basis v, Av, v3 , . . . , vn of Cn . In this basis, T
is given by a matrix B that satisfies B1,1 = 0. Since A and B represent the same
linear transformation (in two different bases), they are similar to each other.
(20∗ ) Define f : S 1 × {1, −1} −→ S 1 by setting f (w, 1) = Sqrt(w) and f (w, −1) = − Sqrt(w).
It is continuous. Surjectivity: Let w ∈ S 1 . If Sqrt(w2 ) = w, then f (w, 1) = w; otherwise,
Sqrt(w2 ) = −w, and hence f (w, −1) = w. Injectivity: If f (w1 , 1) = f (w2 , −1), then
w1 = (f (w1 , 1))2 = (f (w2 , −1))2 = w2 ; however f (w, 1) 6= f (w, −1) for any w. Therefore
2
there do not exist w1 , w2 such that f (w1 , 1) = f (w2 , −1). If f (w1 , 1) = f (w2 , 1) then
w1 = (f (w1 , 1))2 = (f (w2 , 1))2 = w2 ; similarly if f (w1 , −1) = f (w2 , −1) then w1 =
w2 . Since S 1 × {1, −1} and S 1 are Hausdorff and compact, f is a homeomorphism, a
contradiction, since S 1 × {1, −1} is not connected, while S 1 is.

3
CHENNAI MATHEMATICAL INSTITUTE
Postgraduate Programme in Mathematics
MSc/PhD Entrance Examination
18 May 2017
Solutions

Part A
(1) B.
(2) B.
(3) A, B.
(4) A, C, D.
(5) C
(6) A.
(7) B, D.
(8) A, B, C, D.
(9) B, D.
(10) 3.

Part B
(11) (A) X is Hausdorff since the diagonal is closed.
(B) For all x, y, d(x, y) ≤ d(x, x) + d(y, x) = d(y, x), so d(x, y) = d(y, x); hence d is a
metric. In particular, τ 0 is the metric topology given by d.
(C) Identify Bx0 , with d−1 ([0, )) ∩ ({x0 } × X). Since d−1 ([0, )) is open in X × X, we
see that d−1 ([0, )) ∩ ({x0 } × X) is open in the subspace topology of ({x0 } × X)
induced from the product topology of X × X, which is the same as the topology τ
on X. Hence Bx0 , is open in the topology τ , so τ 0 is coarser than τ .
(12) (A) f has a power-series expansion around 0 that converges everywhere on C. Since
|f (0)| ≤ 0, f (0) = 0, so f (z)/z is entire, and |f (z)/z| ≤ 1. Hence, by Liouville’s
theorem, f (z)/z = C for some C ∈ C, i.e., f (z) = Cz.
(B) Z
z dz = 0
Γ
and use the fact that x = z+z 2 and y = 2ı .
z−z

(13) By way of contradiction, assume that there exist  > 0, an increasing sequence n1 <
n2 < · · · of integers and x1 , x2 , . . . ∈ [0, 1] such that |fnk (xk ) − f (xk )| >  for every
k ≥ 1. Let xki , i ≥ 1 be a convergent subsequence, converging to y ∈ [0, 1]. Construct a
new sequence yj as follows:
(
xki , ifj = nki
yj =
y, otherwise.
Then the sequence fj (yj ) does not converge to f (y), a contradiction.
(14) (A) If r is a positive rational number, then log(x) < xr for all real numbers x  0.
(B) The two sequences below are bounded and the series is convergent:
log n log log n X 1
0.1
; 0.1
; .
n n n3/2
(15) Write φa : Fp −→ Fp for the map b 7→ ab. We check the following:
(A) For each a ∈ Fp r {0}, φa is a group automorphism.
(B) The map a 7→ φa is a group homomorphism: 1 7→ idFp ; φa0 a (b) = a0 ab = (φa0 ◦
φa )(b).
(C) The map a 7→ φa is injective: Indeed if φa = idFp then a = 1.
1
(D) The map a 7→ φa is surjective: Let φ be any group automorphism of (Fp , +), which
is a cyclic group, generated by 1. Then φ is determined by φ(1). Since φ is an
automorphism, φ(1) 6= 0. Hence φ(b) = φ(1)b for every b ∈ Fp . Therefore φ = φφ(1) .
(E) A bijective group homomorphism is a group isomorphism.
(16) In each of the three cases, if m is a maximal ideal of R, then it is generated by (the
residue class of) an irreducible polynomial dividing X 7 − 1. Further, if f (X) is an
irreducible polynomial of degree d, then dimk k[X]/f (X) = d.
k = Q: The irreducible factors of X 7 − 1 are (X − 1) and (X 6 + X 5 + · · · + 1). (To see
that (X 6 + X 5 + · · · + 1) is irreducible over Q, write it as ((Y + 1)6 + (Y + 1)5 + · · · + 1)
(where Y = X − 1) and apply the Eisenstein criterion.) Hence the dimensions are 1 and
6.
k = C: Every irreducible polynomial is linear, so the dimension is 1.
chark = 7: X 7 − 1 = (X − 1)7 , so the dimension is 1.
(17∗ ) A has a pole if and only if 1 is an eigenvalue. If A ∈ SO3 , then its real eigenvalues are
±1, and its determinant is 1. Hence if all the eigenvalues are real, then at least one
eigenvalue is 1. If it has exactly one real eigenvalue λ1 , then 1 = λ1 (a + ıb)(a − ıb), so
λ1 > 0, i.e., λ1 = 1. For the second part, we need to show that for p ∈ S2 if Ap = p for
some A ∈ G then for every B ∈ G, Bp is a pole for some C ∈ G. Take C = BAB −1 .
(18∗ ) First suppose that Y and the all fibres f −1 (y), y ∈ Y are compact. We prove a ‘Tube
lemma’: Let y ∈ Y ; if U is an open neighbourbood of f −1 (y), then there exists an open
neighbourbood V of y such that f −1 (V ) ⊆ U . Proof of lemma: X \ U is closed, so
f (X \ U ) is closed. Since f −1 (y) ⊆ U , y 6∈ f (X \ U ). Let Vy = Y \ f (X \ U ). One can
check immediately that f −1 (Vy ) ⊆ U , finishing the proof of the lemma.
Let Uλ , λ ∈ Λ be an open cover of X. For each y ∈ Y , we see, using the above
lemma and the fact that f −1 (y) is compact, that there is a finite subset Λy ⊂ Λ and
an open neighbourbood Vy of y such that f −1 (y) ⊆ f −1 (Vy ) ⊆
S
Uλ . Since Y is
λ∈Λy
compact, there exist y1 , . . . , yn ∈ Y such that Y = Vy1 ∪ · · · ∪ Vyn . Thus X = f −1 (Y ) =
n
f −1 (Vy1 )∪· · ·∪f −1 (Vyn ) ⊆
S S
Uλ , so the open cover Uλ , λ ∈ Λ has a finite subcover.
i=1 λ∈Λyi
Hence X is compact.
Conversely, assume that X is compact and Y is Hausdorff. Then Y is compact. Let
y ∈ Y . Then {y} is closed in Y , so f −1 (y) is closed in X, and hence compact, as X is
compact.

(19 ) The images in F := R/m of the monomials in x1 , . . . , xn form a countable spanning set
1
of F over k. If t ∈ F is transcendental over k, then { t−α | α ∈ k} is linearly independent
over k, which is not possible, so F/k is algebraic. Hence F ' k.
(20∗ ) Let R > 0 be given and B(0, R) be the open unit ball of radius R. By the mean value
inequality, | sin( nz ) − sin(0)| ≤ supw∈B(0,R) | cos(w)|| nz − 0| for every z ∈ B(0, R). Hence
P∞ sin nz P∞ 2
n=1 n is dominated by the series n=1 |z|/n as cos(w) is bounded on P B(0, R).
Therefore the series converges uniformly on compact sets. Hence the series ∞ z
n=1 sin( n )/n
defines a holomorphic function. It is a standard fact that if a sequence of holomorphic
functions converges uniformly on compact sets then the limit is a holomorphic function.

2
CHENNAI MATHEMATICAL INSTITUTE
Postgraduate Programme in Mathematics
MSc/PhD Entrance Examination
15 May 2018

Part A
(1) A, D.
(2) B, C, D.
(3) B, D.
(4) A, B, D.
(5) A, B, D.
(6) A, C, D.
(7) A, B, C.
(8) C, D.
(9) A, B, C.
(10) 2.

Part B
(11) (A) Consider r : R −→ [0, 1]

0, r < 0;

r(x) = x, r ∈ [0, 1];

1, r > 1.

(B) No. Every retract Y of R must be connected because the map r is continuous.
(C) Every retract Y of R is closed. To see this, consider φ : R −→ R × R, x 7→ (r(x), x).
Then Y = φ−1 (diagonal). Since R is Hausdorff, the diagonal is closed, and so is Y .
(12) Write
ξn
Z
1
gn (z) = dξ.
2πi Γ ξ − z
Note that 
1,
 n = 0, z = 0
gn (z) = 0, n 6= 0, z = 0
 n

z , z 6= 0
Hence
n=N
(
X a0 , z = 0;
f (z) = an gn (z) =
n=−N
F (z), z 6= 0.
R1
φ(t)tn dt
(13) Write Fn = (−1)n 0 n! . Then

Z 1 !
1
(−at)n
Z X
−at
0= φ(t)e dt = φ(t) dt
0 0 n!
n=0
N
!
1
(−at)n
Z X
= lim φ(t) dt
N −→∞ 0 n!
n=0
N Z 1
(−at)n
X  
= lim φ(t) dt
N −→∞ n!
n=0 0

X
= Fn an
n=0

1
Since ∞ n
P
n=0 Fn a = 0 for every a ∈ R+ , we see that Fn = 0 for every n ≥ 0.
(14) By way of contradiction assume that U ( R. Then, since U is open, U ( U . Pick
x ∈ U r U and a sequence {xn } ⊆ U converging to x. Since h is uniformly continuous,
{h(xn )} is a Cauchy sequence in R, it converges to y ∈ R. Hence {xn } converges go
h−1 (y) ∈ U , a contradiction.
(15) Since det A = −1, the characteristic polynomial of A is of the form X 2 + bX − 1 for
some b ∈ R, so A has real eigenvalues, λ1 , λ2 . Let vi be an eigenvector for λi , i = 1, 2.
For i = 1, 2, λ2i vit vi = λi vit λi vi = vit At Avi = vit vi , so λi is 1 or −1. Without loss of
generality, λ1 = 1 and λ2 = −1. Then A gives a reflection about the line spanned by v1
sending v2 to −v2 .
(16) (A) For any G, 0 is a characteristic subgroup. Let 0 6= H ⊆ Q be a characteristic
subgroup. Let 0 6= x ∈ H and y ∈ Q. Then the map r 7→ ry/x is an automorphism
of Q, and it takes x to y. Hence y ∈ H, so H = Q. Hence 0 and Q are the only
characteristic subgroups of Q.
(B) For any g ∈ G, the map G −→ G, g1 7→ gg1 g −1 is an isomprphism, so, for every
characteristic subgroup H of G, gHg −1 = H. Hence H is normal. The converse is
false: take H = Z inside G = Q.
(17∗ ) (A) (At M A)t = At M t A = −At M A.
(B) Check directly with v = e1 , e2 , e3 after noting tha the action of SOE (R) and Φ are
k-linear. Also use the fact that the entries At = A−1 are the signed minors of A,
coming from the adjoint of A.
(C) Using (B) it is enough to show that there does not exist a nonzero v ∈ R3 whose
span is stable under the action of SO3 . This is true: if v, w ∈ R3 are non-zero
vectors of the same length, there exists A ∈ SO3 such that Av = w.
(18∗ ) For positive integers n, write Un = {z ∈ C r {0} : |z| < n}. These form an open cover
of C r {0}. Note that for every z1 ∼ z2 , |z1 | = |z2 |, so for every n, π −1 (π(Un )) = Un .
Hence π(Un ), n ≥ 1 is an open cover of X. This does not have a finite sub-cover since
the open cover Un , n ≥ 1 does not have a finite sub-cover.
(19∗ ) First note that the minimal polynomial of g divides X |G| − 1. (A) If char k = 0 then for
every g, g has distinct eigenvalues and hence is diagonalizable, so g = 1, contradicting
the hypothesis that |G| > 1. (B) Let p = char k. Let g ∈ G and write its order as pe m
e
with m = 1 or m > 1 and p - m. The the minimal polynomial of g p is X m − 1 which
e
has distinct roots, so, again, by the above argument, g p = 1, so m = 1. (C) Hence G
is a p-group. Use class equation.
(20∗ ) The limit is f (1). This is true for xk , k ≥ 0, and hence also for polynomials. By
Weierstrass’ theorem, it is true for all continuous functions.

2
CHENNAI MATHEMATICAL INSTITUTE
Postgraduate Programme in Mathematics
MSc/PhD Entrance Examination
15 May 2019
(1) A, B.
(2) D.
(3) A, C.
(4) C, D.
(5) A, B, C.
(6) B, C, D.
(7) C.
(8) C.
(9) C, D.
(10) 0.

Part B
(11) T is not Hausdorff; it is the co-finite topology on C. For any polynomial f , f −1 (A) is a
closed set for every finite set A, so f is continuous.
n
P
(12) n≥0 n z is convergent on {z ∈ C : |z| < 1}.
a
For each real number  > 0, there exists N such that for all n ≥ N , |an | < . Then
we can write
X |z|N
|F (z)| ≤ C + |z|n = C +
(1 − |z|)
n≥N
for some C ∈ R that does not depend on z. Hence for z ∈ C with |z| < 1, (1 − |z|)|F (z)|
can be made to take values arbitrarily close to , for any  > 0, by taking |z| −→ 1.
By way of contradiction assume that G = F . Let ζ ∈ C with |ζ| = 1 is a pole of G.
Let M be the order of the pole at ζ. Write
c−M c−1
G(z) = M
+ ··· + + G1 (z)
(z − ζ) (z − ζ)
where G1 (z) is an analytic function. As z −→ ζ, (1 − |z|)|G(z)| = |(z − ζ)G(z)| exhibits
one of the following behaviours: if M > 1, then it approaches infinity; if M = 1 (which
implies that c−1 6= 0), it approaches c−1 6= 0. This is a contradiction.
(13) Assume that {|n| : n ∈ Z} is bounded. Let N be such that |n| ≤ N for every n ∈ Z. Let
x, y ∈ R. Without loss of generality, |x| ≥ |y|, and we want to show that |x + y| ≤ |x|.
n  
n
X n
|x + y| ≤ | ||x|r |y|n−r
r
r=0
≤ (n + 1)N |x|n for every n
1 1
Hence |x + y| ≤ N n (n + 1) n |x| for every n, so |x + y| ≤ |x|.
hR i1
1 n
(14) Write an = 0 |f (x)|n dx . Let M = sup{|f (x)| : 0 ≤ x ≤ 1}. Then an ≤ M for every
n, so lim sup an ≤ M . Since [0, 1] is compact, for every  > 0, there exists an interval
I ⊆ [0, 1] of positive length such that M −  ≤ |f (x)| ≤ M for every x ∈ I . Then
Z 1
n 1 1
an ≥ |f (x)| dx ≥ [(M − )n · length(Ie )] n = (M − )(length(Ie )) n .
n
I
Hence lim inf an ≥ M −  for every  > 0; therefore lim inf an ≥ M , so lim an = M .
(15) Without loss of generality, we may assume that V 6= 0. Let M, N ∈ V be non-zero
elements. Let λ be an eigenvalue of N M −1 . Then det(λM − N ) = det M det(λIn −
N M −1 ) = 0. However, (λM − N ) ∈ V , so λM − N = 0.
1
(16) If p2 divides n, then there is a non-cyclic group Z/pZ × Z/pZ × Z/(n/p2 )Z. There are
exactly pq such matrices, and theyQform a nonabelian group. If p1 , . . . , pr are the distinct
prime divisors of n, then φ(n) = ti=1 (pi − 1), which is coprime to n, by above.
(17∗ ) Let g ∈ G be of order 2. Then the minimal polynomial of g divides X 2 −1. If char F 6= 2,
then the minimal polynomial of g is X + 1 or X 2 − 1. In either case, g is diagonalizable
and the conjugacy class of g is determined by the number of −1s on the diagonal; there
must be at least one −1. Hence |X| = n. If char F = 2, then X 2 − 1 = (X − 1)2 , which
must be the minimal polynomial of g. (It cannot be X−1.) Hence g is not diagonalizable,
and the conjugacy class of g is determined by the number of 2 × 2 Jordan blocks (with
eigenvalue 1). Hence |X| = b n2 c.
(18∗ ) Consider the sequences (4n+1)π 2
and nπ 1
. Both converge to 0, but f ( (4n+1)π 2
) = 1 and
f ( nπ ) = 0 for every n ≥ 1, so f does not extend to Y . Let h : X −→ X1 , t 7→ (t, sin 1t ).
1

It is a homeomorphism. Since X1 ⊆ [0, 1] × [−1, 1], Y1 is compact. Identifying X with


X1 using h, we get that Y1 is a compactification of X. Further, f = π2 ◦ h, where π2 is
the projection Y1 −→ R on to the second component. Note that f extends to the map
π2 (after identifying X with X1 using h).
(19∗ ) Let α, 3α, β3 , . . . , βn be the roots of f , so f (0) = 3α2 β3 · · · βn . If f (0) = 1, then 13 =
α2 β3 · · · βn satisfies a monic irreducible polynomial g ∈ Z[X]. Since g is irreducible in
Q[X], g = (X − 31 ), which is a contradiction. Hence f (0) 6= 1.
Note that K = Q(α, 3α, β3 , . . . , βn ) and that every field automorphism of K permutes
these generators of K. Hence there are at most n! distinct field automorphisms of K,
so σ is of finite order, which we denote by m. Then α = σ m (α) = σ(σ m−1 (α)) =
3(3m−1 α) = 3m α), so α = 0.

(20 ) Since [0, x] is compact and f is continuous, one can use max, so g is well-defined. Note
that f is uniformly continuous, so for every  > 0, there exists δ > 0 such that for every
x, y with |x−y| < δ |f (x)−f (y)| < . Let u < v < u+δ. Then f (u)− ≤ f (x) ≤ f (u)+
for every x ∈ [u, v], so g(v) ≤ g(u)+. On the other hand g(v) ≥ g(u), so |g(v)−g(v)| < .
Hence g is continuous.

2
CHENNAI MATHEMATICAL INSTITUTE
Postgraduate Programme in Mathematics
MSc/PhD Entrance Examination
4th October 2020

Part A
(1) A, B.
(2) A, C.
(3) A.
(4) D.
(5) A.
(6) C, D.
(7) A, C, D.
(8) B.
(9) A, D.
(10) 3.

Part B
(11) It is easily checked that δ is a metric. Let Ck be a sequence of compact subsets of X.
We must show that there is a convergent subsequence.
Given any  > 0, using there exists an N = N () such that the every Ck is covered by
(atmost) N () many open balls of radius  with centres in Ck . We do this first for X with
/2 balls and then take, for each such ball B, an -ball about a point in Ck in case the
intersection B ∩Ck is non-empty. We let  vary over 1/n, n ∈ N. This way we get a finite
set Fk,n ⊂ Ck of centres of balls used to cover Ck . We may assume that Fk,n ⊂ Fk,n+1
by including
P balls of radius (n+1) around points of Fk,n . The cardinality of Fk,n is at
most m≤n N (1/m) =: Nn which is independent of k.) Then δ(Fk,n , Ck ) < 1/n. We
list the points of ∪n≥1 Fk,n in a sequence: xk,1 , xk,2 , . . . , xk,j , . . . , viewed as kth row of a
matrix whose rows and columns are labelled by N. It is understood that in the above
sequence we list members of Fk,i before those of Fk,i+1 ∀i ≥ 1 for each k ≥ 1.
We consider the first column. Now xk,1 , k ∈ N is a sequence of points in X which has
a convergent subsequence xkr ,1 ∈ Ckr , r ∈ N . Set y1 to be the limit of this subsequence.
We will denote Ck1 as C11 .
We consider second column entries corresponding to the rows labelled by the sub-
sequence kr —that is the sequence xkr ,2 . This has a convergent subsequence, say with
limit y2 . We will denote the first term of the corresponding subsequence by C22 .
Proceeding thus we obtain a sequence of points y1 , . . . , yn , . . .. We let C be the closure
of {yk | k ≥ 1}. Then C is compact.
Our claim is that the ”diagonal” sequence Ckk converges to C.
Let n > 0 be a positive integer. Given any m ∈ N we have δ(Cm , Fm,n ) < 1/n
and so δ(xm,r , Fm,n ) < 1/n for all r. P So yr is at a distance at most 1/n from Fn :=
{y1 , y2 , . . . , ym | m ≤ Nn } where Nn = l≤n N (1/l). So it follows that δ(C, Fn ) ≤ 1/n.
If Ckk = Cr , (r = r(k) depends on k) for sufficiently large k we have δ(xr,j , yj ) < 1/n
for j ≤ Nn . Hence we have δ(Fr,n , Fn ) < 1/n. So

δ(Cr , C) ≤ δ(Cr , Fr,n ) + δ(Fr,n , Fn ) + δ(C, Fn ) < 3/n.

(12) Let C be a component.Then C is open. If C is bounded, the value of |f | on its boundary


∂C is 1 ( by continuity, open mapping theorem and because ∂C is disjoint with C).
Hence same for 1/|f |. But this contradicts |f (z)| < 1 on C.
1
1
(13) Let f : X −→ R be given by x 7→ min{d(x,p)|p∈F } . It is continuous. The graph Γf :=
{(x, f (x)) | x ∈ X} is a closed subset of X × R, hence a complete metric space. X 7→
Γf , x 7→ (x, f (x)) is a homeomorphism.
(14) Note that f [0, ∞) ⊂ [0, ∞) as f 0 ≥ 0. Integrating f 0 /f 2 on [0, x] gives
1 − 1/f (x) ≥ x
and hence f is unbounded in [0, P 1).
(15) Let c1 , . . . , cn ∈ C be such that i ci eai z = 0. Differentiating this n − 1 times, we see
that i ci aji eai z = 0 for every 0 ≤ j ≤ n − 1. Substitute z = 0 to get i ci aji = 0 for
P P

every 0 ≤ j ≤ n − 1. The Vandermonde matrix (aji )i,j is invertible, so ci = 0 for every i.


(16) contain m for any integer m with gcd(m, pn ) = 1, so H = {jp | 0 ≤ j < p}.
(17∗ ) If M is diagonalizable, then we may assume that M is a diagonal matrix. Then P (M )
is a diagonal matrix. Such a matrix is nilpotent if and onlyQif it is zero. Conversely, let
α1 , . . . , αm be the distinct eigenvalues of M . Let P (X) = m i=1 (X − αi ). Let µ(X) be
the minimal polynomial of M . Since the roots of µ(X) are exactly α1 , . . . , αm , there
exists a positive integer r such that (P (X))r is divisible by µ(X). Hence (P (M ))r = 0.
Therefore P (M ) = 0, i.e., P (X) = µ(X). Therefore M is diagonalizable.
(18∗ ) (1) True: In fact, X is path connected. We know the closed interval [0,1] is normal.
Given two points x, y ∈ X, consider the function f : {0, 1} −→ X defined by f (0) =
x, f (1) = y. Then f is continuous and extends to all of [0, 1], by the hypothesis on X.
(2) False: take a finite set of cardinality at least 2. Then it is compact and not
connected. So it can’t have universal extension property.
(3) Note that X ⊂ R2 is a retract: r : R2 −→ X given by f (x, y) = (x, sin x)
is a retraction. Since R2 has universal extension property, any map A −→ R2 can be
extended to all of Y (for any given pair of a normal space Y and a closed subset A ⊂ Y ).
If we are given a function f : A −→ X, composing with the inclusion X −→ R2 , we
have an extension g : Y −→ R2 . Composing this with r, we get the desired extension
Y −→ X.
Note that it is not required to know about the language of retracts for (3). They will
have to notice that a continuous map like r exists.
(19∗ ) If f (a) = f (b) = 0 for some a, b ∈ K, then so f (λa + b) = 0 for every λ ∈ Fq . Hence the
set of zeros is an Fq -vector-space. It is n-dimensional since f is separable.
(20∗ ) First n(n + 1) and n2 are of the same order (the ratio converges to one). So you can as
well take denominator as n(n + 1). Thus need to show
a1 + 2a2 + 3a3 + · · · + nan
→ a.
n(n + 1)/2
Imitate blindly Cesaro theorem. Given  > 0, choose K so that |an − a| < /2 for
n ≥ K. Then choose N > K so that the finite sum
|a1 − a| + 2|a2 − a| + 3|a3 − a| + · · · + K|aK − a|
< /2.
N (N + 1)/2
If now n > N then
a1 + 2a2 + 3a3 + · · · + nan
|bn − a| = | − a|
n(n + 1)/2
(Use 1 + · · · + n = n(n + 1)/2 to distribute a to each term in numerator.)
|a1 − a| + 2|a2 − a| + 3|a3 − a| + · · · + n|an − a|

n(n + 1)/2
(split first K terms and the remaining.)
 
< +
2 2

2
CHENNAI MATHEMATICAL INSTITUTE
Postgraduate Programme in Mathematics
MSc/PhD Entrance Examination
1st August 2021

Part A
(1) A, B, D.
(2) C.
(3) B, C.
(4) A, B.
(5) A, B.
(6) B, D.
(7) B.
(8) B, C, D.
(9) A, C.
(10) A, B, D.

1
Part B
(11) Write 𝑍 for the centre of 𝐺. If 𝑍 ⊄ 𝑀, then 𝑀𝑍 > 𝑀 and by maximality 𝑀𝑍 = 𝐺. But then 𝑀
is a normal subgroup of 𝐺, a contradiction. Conversely h𝑀, 𝑁 i = 𝐺 by maximality. Also since 𝑀
and 𝑁 are abelian, we have
𝐺 = h𝑀, 𝑁 i ⊂ 𝐶𝐺 (𝑀 ∩ 𝑁 )
and hence (𝑀 ∩ 𝑁 ) ⊂ 𝑍 .

(12) 𝑓 (𝐷) ∩ 𝐷 is open in 𝐷 by Inverse mapping theorem. 𝑓 (𝐷) ∩ 𝐷 is also closed in 𝐷. Let 𝑧𝑛 = 𝑓 (𝑥𝑛 )
be a sequence in 𝑓 (𝐷) ∩ 𝐷 converging to 𝛼 ∈ 𝐷 with 𝑥𝑛 ∈ 𝐷. Then 𝛼 = 𝑓 (𝛽) for some 𝛽 ∈ 𝐷. But
the hypothesis ensures that 𝛽 ∈ 𝐷. 𝑓 (𝐷) ∩𝐷 is non-empty as 0 belongs to this. So 𝑓 (𝐷) ∩𝐷 = 𝐷.

(13) Union of connected sets having a point in common is connected. So 𝐴 is connected. Closure of
a connected set is connected. So 𝐴 is connected and, hence, 𝐴 ∈ S so 𝐴 ⊆ 𝐴.

(14) Without loss of generality, we may assume that Image(𝑓 ) ⊆ (0, ∞).
Let 𝜖 = 1. There exists a positive integer 𝑁 such that for all 𝑥, 𝑥 0 ∈ [1, ∞) with |𝑥 − 𝑥 0 | < 𝑁1 ,
|𝑓 (𝑥) − 𝑓 (𝑥 0)| < 1. Hence |𝑓 (𝑛 + 1) − 𝑓 (𝑛)| < 𝑁 for each positive integer 𝑛. Therefore 𝑓 (𝑛 + 1) <
𝑓 (1) + 𝑛𝑁 for each positive integer 𝑛. Hence
∑︁ 1 ∑︁ 1

𝑛 ≥1
𝑓 (𝑛) 𝑛 ≥0 𝑓 (1) + 𝑛𝑁
which is a divergent series.

(15) On [0, 1] it is a continuous function. Need only show integral from 1 to infinity is finite. Change
∫∞
variable 𝑥 = 𝑦 100 , 𝑑𝑥 = 100𝑦 99𝑑𝑦 the integral becomes 100 𝑦𝑐 𝑒 −𝑦 𝑑𝑦 for some 𝑐 and if you take
1
∫∞
an integer 𝑁 > 𝑐 the integral is bounded by 𝑦 𝑁 𝑒 −𝑦 𝑑𝑦 which is finite.
1

(16) False in both cases. Take 𝐹 = F3 the finite field of order 3 and 𝐼 1 = (𝑥 2 + 1) and 𝐼 2 = (𝑥 2 + 2𝑥 +
2). Both these are maximal ideals (check that the generating polynomials are irreducible since
they have no roots) and hence the quotients 𝑅/𝐼 1, 𝑅/𝐼 2 are both finite fields of order 9 and they
isomorphic. But 𝐼 1 ≠ 𝐼 2 . If 𝐹 = R, then for every irreducible quadratic polynomial 𝑓 (𝑋 ) ∈ 𝑅,
𝑅/(𝑓 (𝑋 )) ' C.

(17∗ ) (A) Let 𝑣 1, 𝑣 2 be the columns of 𝐴; they form a basis of R2 . Let 𝑢 1 = 𝑣 1 /|𝑣 1 |, 𝑎 = (𝑢 1 · 𝑣 2 ), 𝑣 20 =
𝑣 2 − 𝑎𝑢 1 , 𝑢 2 = 𝑣 20 /|𝑣 20 |. Since 𝑢 1 and 𝑢 2 form an orthonormal basis of R2 , define 𝐴𝑜 to be the
matrix with columns 𝑢 1, 𝑢 2 . Define 𝐴𝑏 to be the inverse of
 1   
|𝑣1 | 0 1 −𝑎 1 0
1 .
0 1 0 1 0 |𝑣20 |
Then 𝐴 = 𝐴𝑜 𝐴𝑏 .
(B) Suppose 𝐴 0𝐴 00 = 𝐴10𝐴100, with 𝐴 0, 𝐴10 ∈ O(2, R) and 𝐴 00, 𝐴100 ∈ B+ (2, R). Then 𝐴 00 (𝐴100) −1 ∈
O(2, R). Note that
𝑎 𝑏 𝑎0 𝑏 0
  0
𝑎𝑎 𝑎𝑏 0 + 𝑏𝑐 0
  
= ∈ O(2, R)
0 𝑐 0 𝑐0 0 𝑐𝑐 0
if and only if
 0 0     −1
𝑎 𝑏 𝑎 𝑏
= .
0 𝑐0 0 𝑐
Therefore 𝐴 00 = 𝐴100, and, hence, 𝐴 0 = 𝐴100.
2
(C) By above, there is a well-defined function
𝜓 : GL(2, R) −→ O(2, R) × B+ (2, R) 𝐴 ↦→ (𝐴𝑜 , 𝐴𝑏 )
that is the inverse of 𝜙.
We first show that the map 𝐴 ↦→ (𝐴𝑏 ) −1 is continuous. From the description of (𝐴𝑏 ) −1
given above, we see that its entries are rational functions of the entries of 𝐴, with non-zero
denominators (since 𝐴 ∈ GL(2, R). Hence the map 𝐴 ↦→ (𝐴𝑏 ) −1 is continuous. Hence the
maps 𝐴 −→ 𝐴𝑜 = 𝐴(𝐴𝑏 ) −1 and 𝐴 −→ 𝐴𝑏 are continuous. Therefore 𝜓 is continuous.
𝜙 is continuous: The entries of 𝐴 0𝐴 00 are polynomial functions of the entries of 𝐴 0 and of 𝐴 00.

3
(18∗ ) 𝜙 satisfies 𝑇 𝑝 − 1 = 0. Hence the only eigenvalue of 𝜙 is 1. Let 𝑣 1 be an eigenvector for the
eigenvalue 1. Then 𝜙 induces an invertible linear transformation of 𝑉 /h𝑣 1 i; proceed by induction
on dimension.

(19∗ ) (A) First,


√5 [Q(𝜁 5 ) : Q] = 4 since the polynomial 𝑥 4 + 𝑥 3 + 𝑥 2 + 𝑥 + 1 is irreducible over Q. Next,
5
[Q( 2) : Q] = 5 since 𝑥 − 2 is irreducible over Q. For both use the Eisenstein criterion.
So [𝐾 : Q] is divisible by both 4 and 5. On the other hand, we have [𝐾 : Q(𝜁 5 )] ≤ 5. So we
conclude [𝐾 : Q] = 20.
(B) If 𝚤 ∈ Q(𝜁 5 ), then 𝛼 := 𝚤𝜁 5 ∈ Q(𝜁 5 ). Note that 𝛼 20 = 1 and 𝛼 𝑛 ≠ 1 for 1 ≤ 𝑛 ≤ 19. So 𝛼 is a
primitive 20th root of unity and it is contained in Q(𝜁 5 ). But we know that [Q(𝜁 5 ) : Q] = 4
and by the fact given in the hint, we have [Q(𝛼) : Q] > 4. So 𝚤 ∉ Q(𝜁 5 ).
(C) Suppose 𝚤 ∈ 𝐾. Consider the tower Q ⊂ Q(𝜁 5 ) ⊂ Q(𝜁 5, 𝚤) ⊂ 𝐾. The degree of the first
extension is 4; the degree of the second extension is 2 (since 𝚤 ∉ Q(𝜁 5 )). So [𝐾 : Q] is
divisible by 8. This is not possible since [𝐾 : Q] = 20.

(20∗ ) (A) Write 𝑓 (𝑧) =


Í 𝑛.
𝑛 ≥0 𝑎𝑛 𝑧 Then
𝑓 (𝑧) − 2𝑧 𝑓 (𝑧) − 𝑧 2 𝑓 (𝑧) = 𝑎 0 + 𝑎 1𝑧 + 𝑎 2𝑧 2 + 𝑎 3𝑧 3 + · · ·
− 2𝑎 0𝑧 − 2𝑎 1𝑧 2 − 2𝑎 2𝑧 3 + · · ·
− 𝑎 0𝑧 2 − 𝑎 1𝑧 3 − 𝑎 2𝑧 4 + · · ·
= 𝑎 0 + (𝑎 1 − 2𝑎 0 )𝑧
Let
𝑎 0 + (𝑎 1 − 2𝑎 0 )𝑧
𝑔(𝑧) =
1 − 2𝑧 − 𝑧 2 Í
This is analytic in a neighbourhood of the origin. Let 𝑛 ≥0 𝑏𝑛 𝑧𝑛 be the Taylor series expan-
sion of 𝑔(𝑧) around 0. Then (1 − 2𝑧 − 𝑧 2 )𝑓 (𝑧) = (1 − 2𝑧 − 𝑧 2 )𝑔(𝑧), from which we see that
𝑎𝑛 = 𝑏𝑛 for every 𝑛.
(B) Note that
1
𝑔(𝑧) = .
1 − 2𝑧 − 𝑧 2
Let 𝛾 1, 𝛾 2 be the roots of 1 − 2𝑧 − 𝑧 2 . Note that 𝛾 1 ≠ 𝛾 2 . Then there exist 𝛽 1, 𝛽 2 such that
𝛽1 𝛽2
𝑔(𝑧) = + .
𝑧 − 𝛾1 𝑧 − 𝛾2
Therefore
(−1)𝑛 𝑛!𝛽 1 (−1)𝑛 𝑛!𝛽 2
𝑔 (𝑛) (𝑧) = + .
(𝑧 − 𝛾 1 )𝑛+1 (𝑧 − 𝛾 2 )𝑛+1
Hence
𝛽1 𝛽2
𝑎𝑛 = − 𝑛+1 − 𝑛+1 .
𝛾1 𝛾2
1
Now replace 𝛽𝑖 by −𝛽𝑖 and 𝛾𝑖 by 𝛾𝑖 .
3
CHENNAI MATHEMATICAL INSTITUTE
Postgraduate Programme in Mathematics
MSc/PhD Entrance Examination
22nd May 2022

Part A
(1) A, C.
(2) D.
(3) A.
(4) A.
(5) B, C, D.
(6) B, C.
(7) A, D.
(8) B, C, D .
(9) B, C.
(10) B, C, D.

1
Part B
(11) (A) Multiplying by the transpose, we get

 𝑡 1 
 1
3 = 1 1 1 𝐴 𝐴 1 = 1 1 1 𝜆 2 1 = 3𝜆 2
  

1 1
 
so 𝜆 = ±1.
(B) Similarly,

 𝑡 1
𝑥   
    
𝜆(𝑥 + 𝑦 + 𝑧) = 1 1 1 𝜆 𝑦  = 1 1 1 𝐴 𝐴 −1 = 0
 
𝑧  0
   
so 𝑥 + 𝑦 + 𝑧 = 0 since 𝜆 ≠ 0.
(12) The ratio above can be expressed as
1 Í𝑛 𝑗 𝑎
𝑛 𝑗=1 ( 𝑛 )
.
(1 + 𝑛1 )𝑎−1 · 𝑛1 𝑛𝑗=1 (𝑎 + 𝑛𝑗 )
Í
∫1
In the limit, the numerator becomes the integral 0 𝑥 𝑎𝑑𝑥. Likewise, the denominator tends to
∫1
0
(𝑎 + 𝑥)𝑑𝑥. Setting 𝑆 (𝑎) = 1/60 gives the quadratic equation 2𝑎 2 + 3𝑎 − 119 = 0. So the answer
is −3/2.
(13) By way of contradiction, let 𝛼 ∈ 𝑉 \ 𝑓 (𝑈 ). Find a countable collection of nested closed balls {𝐶𝑛 }
around 𝛼 whose intersection is {𝛼 }. Then
∩𝑛 𝑓 −1 (𝐶𝑛 ) = ∅.
But then 𝑓 −1 (𝐶𝑛 ) = 𝜙 for some 𝑛 and hence the interior of 𝐶𝑛 contains 𝛼 and is in 𝑉 \ 𝑓 (𝑈 ).
So 𝑉 \ 𝑓 (𝑈 ) is open, it is also closed, contradicting the hypothesis that 𝑉 is connected. Thus
𝑓 (𝑈 ) = 𝑉 .
(14) (A) By way of contradiction, assume that 𝑝 and 𝑞 are distinct prime numbers that divide |𝐺 |.
Then 𝐺 has subgroups of orders 𝑝 and 𝑞, so such an 𝑁 cannot exist. Hence 𝐺 is a finite
𝑝-group for some 𝑝.
(B) Hence the centre 𝑍 (𝐺) ≠ {1𝐺 }. Since 𝑍 (𝐺) is abelian, and a normal subgroup of 𝐺 and 𝑁 a
subgroup of 𝑍 (𝐺), it follows that 𝑁 is a normal subgroup of 𝐺.
(15) Chose 𝛼 ∈ C such that <(𝛼) < 0 and <(𝛼 𝑓 0 (1)) > 0. This is possible if 𝑓 0 (1) ∉ R. Since
𝑓 (𝐷) ⊂ 𝐷, for small 𝑡 > 0
𝑓 (1 + 𝑡𝛼) − 1
< <0
𝑡
and hence as 𝑡 −→ 0+ , <(𝛼 𝑓 0 (1)) ≤ 0.
For the second part, note that by Schwartz lemma and 0 < 𝑡 < 1, |𝑓 (𝑡)| ≤ 𝑡 and hence
|𝑓 (𝑡) − 1| 1 − |𝑓 (𝑡)|
≥ ≥1
1−𝑡 1−𝑡
and as 𝑡 −→ 1, we have the inequality.
(16) Since everyÐ𝑟finite extension of a finite field is Galois, 𝐹 is infinite By way of contradiction, assume
that 𝑉 = 𝑖=1 𝑉𝑖 . Pick 𝑥 ≠ 0 ∈ 𝑉1 and 𝑦 ∈ 𝑉 \ 𝑉1 . Then there exists 𝑗 > 1Ðsuch that 𝑥 + 𝛼𝑦 ∈ 𝑉𝑗
for infinitely many 𝛼 ∈ 𝐹 × . Hence 𝑦 ∈ 𝑉𝑗 and, so, 𝑥 ∈ 𝑉𝑗 . Therefore 𝑉 = 𝑟𝑖=2 𝑉𝑖 . Repeating this
argument, we get a contradiction.

2
(17∗ ) Fibre is
C[𝑋, 𝑌 ]/(𝑋𝑌 − 1, 𝑋 + 𝛼𝑌 − 𝛽).
If 𝛼 = 𝛽 = 0, then it is the zero ring since (𝑋𝑌 − 1, 𝑋 ) = C[𝑋, 𝑌 ]. Otherwise,
C[𝑋, 𝑌 ]/(𝑋𝑌 − 1, 𝑋 + 𝛼𝑌 − 𝛽) ' C[𝑋 ]/(−(𝛼𝑌 − 𝛽)𝑌 − 1) ' C[𝑌 ]/((𝛼𝑌 − 𝛽)𝑌 + 1)
The equation 𝛼𝑌 2 − 𝛽𝑌 + 1 has at least one solution; it has exactly one solution if and only if
𝛽 2 = 4𝛼. Hence for 𝑛 = 0, the answer is {(0, 0)}. For each (𝛼, 𝛽) with 𝛽 2 = 4𝛼, the fibre has
exactly one prime ideal; for all other values of (𝛼, 𝛽), the fibre has exactly two prime ideals.
(18∗ ) (A) Consider the projection maps 𝜋𝑖 : 𝑄 −→ R, (𝑦1, . . .) ↦→ 𝑦𝑖 . Then
Ù 1
𝑄2 = 𝜋𝑖−1 ( [0, ])
𝑖 ≥1
𝑛
so it is a closed subset of 𝑄. Since 𝑄 is compact by Tychonoff’s theorem, 𝑄 2 is compact.
(B) The maps (𝑦1, . . .) ↦→ 𝑖𝑦𝑖 is continuous for each 𝑖, so 𝐷 is continuous. It is bijective. Since 𝑄
is Hausdorff and 𝑄 2 compact, 𝐷 is continuous.
(C) Take 𝐿 = ℓ2 and a = (1, 21 , . . . , 𝑛1 , . . . ). (Note that 𝑄 2 ⊆ ℓ2 .)
(D) 𝐷 is a homeomorphism and 𝑆 ◦ 𝐷 is continuous.
(19∗ ) (A) is false since the polynomial 𝑋 12 − 𝑝 has two real roots.
(B) is a consequence of the fact that any degree 11 polynomial has a real root.
(C) If the group Aut(𝐸) of all field automorphisms of 𝐸 has odd order, then every embedding
𝐸 −→ C is real. note that if 𝜎 is a complex embedding, then complex conjugation yields a
nontrivial automorphism 𝐸 of order 2.
(20∗ ) (A) 𝑑𝑋 (𝑎, −𝑎) = 2𝑅. Hence 2𝑅/𝜆 ≤ 𝑑𝑌 (𝑓 (𝑎), 𝑓 (−𝑎)) ≤ 𝐿𝑅 + 2, so 𝐿𝑅 ≥ 2𝑅/𝜆 − 2. 𝐿𝑅 =
𝑑𝑌 (𝑓 (𝑎), 𝑓 (𝑏)) ≤ 𝜆𝑑𝑋 (𝑎, 𝑏). Hence 𝑑𝑋 (𝑎, 𝑏) ≥ 𝐿𝑅 /𝜆 ≥ 2𝑅/𝜆 2 − 2/𝜆.
(B) Note that 𝑓¯(𝐶 1 ) = 𝑓¯(𝐶 2 ) = 𝑓¯(𝐶𝑅 ). Hence there exist such 𝑥𝑖 as asserted. Since {𝑓 (𝑥 1 ), 𝑓 (𝑥 2 )} ⊆
𝑓 (𝑎)+𝑓 (𝑏)
S1 × { 2 }, it follows that 𝑑𝑌 (𝑓 (𝑥 1 ), 𝑓 (𝑥 2 )) ≤ 2.
(C) 𝑑𝑌 (𝑓 (𝑥 1 ), 𝑓 (𝑎)) ≥ 𝐿𝑅 /2, so 𝑑𝑋 (𝑥 1, 𝑎) ≥ 𝐿𝑅 /2𝜆 ≥ (𝑅 − 𝜆)/𝜆 2 . Similarly 𝑑𝑋 (𝑥 2, 𝑎) ≥ 𝐿𝑅 /2𝜆 ≥
(𝑅 − 𝜆)/𝜆 2 . Choose 𝑎𝑖 ∈ 𝐶𝑖 be such that 𝑑𝑋 (𝑎, 𝑎𝑖 ) = (𝑅 − 𝜆)/𝜆 2 ; Elementary trigonometric
arguments show that for all 𝑅  0, 𝑑𝑋 (𝑎 1, 𝑎 2 ) > 2𝜆. Hence 𝑑𝑋 (𝑥 1, 𝑥 2 ) > 2𝜆. Therefore
𝑑𝑌 (𝑓 (𝑥 1 ), 𝑓 (𝑥 2 )) > 2.
(D) There does not exist any bilipschitz map 𝑋 −→ 𝑌 .

3
CHENNAI MATHEMATICAL INSTITUTE
Postgraduate Programme in Mathematics
MSc/PhD Entrance Examination
7th May 2023

Part A
(1) A, B, C.
(2) B, C, D.
(3) D.
(4) A, C, D.
(5) B, D.
(6) B, D.
(7) A, C, D.
(8) A.
(9) A, C, D.
(10) C, D.

Part B
(11) For 0 < 𝑥 < 1, write 𝑥 −𝑥 = 𝑒 ln 𝑥 −𝑥
Í∞
= 𝑛=0 1/𝑛! (−𝑥 ln 𝑥)𝑛 . It converges pointwise. Since the
terms in the summation are positive, by Dini’s theorem, the convergence is uniform.
Change order of the summation and the integral using uniform convergence. Now integrate
each term as follows to get the result.
∫ 1 ∫ ∞ ∫ ∞
𝑛 − (𝑛+1)𝑢 1 1
(−𝑥 ln 𝑥) 𝑑𝑥 =
𝑛
𝑢 𝑒 𝑑𝑢 = 𝑛+1
𝑣 𝑛 𝑒 −𝑣𝑑𝑢 = 𝑛! .
0 0 (𝑛 + 1) 0 (𝑛 + 1)𝑛+1
(First equality by substitution𝑢 = − ln 𝑥, second by (𝑛+1)𝑢 = 𝑣, and then by repeated integration
by parts.)
(12) (A) If 𝑃𝑄 were a group, then, its order would be |𝑃 ||𝑄 |/|𝑃 ∩ 𝑄 |.
Note that 𝑃 ∩ 𝑄 ⊂ 𝑄. If 𝑃 ∩ 𝑄 = 𝑄, then 𝑄 ⊂ 𝑃, a contradiction. If 𝑃 ∩ 𝑄 ⊄ 𝑄, then
|𝑄 |/|𝑃 ∩ 𝑄 | = 𝑝 𝑒 for some 𝑒 ≥ 1, so 𝑃𝑄 is a 𝑝-group bigger than 𝑃, a contradiction.
(B) The condition on Γ implies that all Sylow subgroups are normal and hence unique.
𝑓 𝑓
Write |Γ| = 𝑝 𝑒1 1 · · · 𝑝𝑟𝑒𝑟 with 𝑒𝑖 > 0 for each 1 ≤ 𝑖 ≤ 𝑟 . Write |𝐻 | = 𝑝 1 1 · · · 𝑝𝑟 𝑟 with 𝑓𝑖 ≥ 0 for
each 1 ≤ 𝑖 ≤ 𝑟 . For 1 ≤ 𝑖 ≤ 𝑟 , let 𝑃𝑖 be the Sylow 𝑝𝑖 -subgroup of Γ. Then 𝑃𝑖 ∩ 𝐻 = ⟨1⟩ or
is a 𝑝𝑖 -subgroup of 𝐻 . Conversely, if 𝑓𝑖 > 0, then every 𝑝𝑖 -subgroup of 𝐻 is inside 𝑃𝑖 , so is
inside 𝑃𝑖 ∩ 𝐻 . Hence if 𝑓𝑖 > 0, then 𝑃𝑖 ∩ 𝐻 is the unique Sylow 𝑝𝑖 -subgroup of 𝐻 .
Now consider the group homomorphism
Ö
(𝑃𝑖 ∩ 𝐻 ) −→ 𝐻, (𝑔1, . . . , 𝑔𝑟 ) ↦→ 𝑔1 · · · 𝑔𝑟 .
𝑖
(This indeed is a group homomorphism.) The orders of the groups on the two sides are the
same, so it suffices to show that the homomorphism is injective. Suppose that 𝑔1 · · · 𝑔𝑟 = 1.
Then 𝑔1 = (𝑔2 · · · 𝑔𝑟 ) −1 ∈ 𝑃 1 ∩ 𝑃 2 · · · 𝑃𝑟 = ⟨1⟩, so 𝑔1 = 1. Repeatedly doing this, we see that
𝑔𝑖 = 1 for each 𝑖.
(13) (A) Write 𝑓 = 𝑎𝑋 𝑛 + 𝑔 where 𝑔 is a homogeneous polynomial of degree 𝑛 that does have 𝑋 𝑛 as
its term. Then 𝑓 (𝑝) = 𝑎. Moreover,
𝜕𝑓 𝜕𝑔
= 𝑎𝑛𝑋 𝑛−1 +
𝜕𝑋 𝜕𝑋
so
𝜕𝑓
(𝑝) = 𝑎𝑛.
𝜕𝑋
Hence 𝑎 = 𝑎𝑛, so 𝑎 = 0. Hence dim 𝑉 /𝑊 = 1.
1
(B) Let𝑉 be the subspace of R9 spanned by𝑒 2, 𝑒 3, . . . , 𝑒 9 and𝑊 the subspace spanned by𝑒 2, 𝑒 3, . . . , 𝑒 8 .
It is clear that 𝑇 (𝑉 ) = 𝑊 . In greater detail, by the formulas defining 𝑇 , 𝑇 𝑒𝑖 ∈ 𝑊 for
𝑖 = 2, . . . , 9, whence 𝑇 (𝑉 ) ⊂ 𝑊 . Since the elements spanning 𝑊 occur in 𝑇 (𝑉 ), we get
𝑇 (𝑉 ) = 𝑊 . Since R9 is spanned by 𝑉 and 𝑒 1 , it follows that 𝑇 (R9 ) is spanned by 𝑇 (𝑉 ) and
𝑇 𝑒 1 . In other words𝑇 (R9 ) is the linear span of𝑊 and 𝑒 1 +𝑒 3 +𝑒 8 . Since 𝑒 1 ∉ 𝑊 , it follows that
𝑇 𝑒 1 = 𝑒 1 +𝑒 3 +𝑒 8 ∉ 𝑊 . Thus𝑊 +R𝑇 𝑒 1 = 𝑊 ⊕R𝑇 𝑒 1 , whence dim𝑇 (R9 ) = dim𝑊 +1 = 7+1 = 8.
The nullity of 𝑇 is therefore 1 by the rank-nullity theorem.
(14) (A) Can be checked directly.
(B) 𝐼 is a prime ideal. In fact, 𝐼 = ker 𝜙 where 𝜙 : 𝑆 → 𝑅 is the surjective ring map 𝜙 (𝑓 (𝑥)) =
𝑓 (0).
(C) 𝐼 is a non-zero prime ideal that is not maximal. Hence 𝑆 is not a PID.
(15) (A) We have (𝑓 (𝑥) − 𝑓 (𝑦)) (𝑔(𝑥) − 𝑔(𝑦)) ≥ 0 for all 𝑥, 𝑦 ∈ [0, 1]. So the double integral
∫ ∫
(𝑓 (𝑥) − 𝑓 (𝑦)) (𝑔(𝑥) − 𝑔(𝑦)) 𝑑𝑥𝑑𝑦 ≥ 0.
[0,1] × [0,1]

Iterating and expanding we get the inequality.


(B) By Rolle’s Theorem, there exists𝑦1 ∈ (0, 1) such that 𝑓 ′ (𝑦1 ) = 0. Since 𝑓 ′ (0) = 0 (by hypothe-
sis), we have, once again by Rolle’s Theorem, an element 𝑦2 ∈ (0, 𝑦1 ) such that 𝑓 (2) (𝑦2 ) = 0.
If 𝑛 = 1, we are done. Assume 𝑛 > 1 and assume, by way of induction, that there exists
𝑦 ∈ (0, 1) such that 𝑓 (𝑛) (𝑦) = 0. Since 𝑓 (𝑛) (0) = 0, one more application of Rolle’s Theo-
rem gives us an 𝑥 ∈ (0, 𝑦) ⊂ (0, 1) such that 𝑓 (𝑛+1) (𝑥) = 0.
(16) (A) For 𝑡 ∈ R we have, by squaring both sides of the given inequality of non-negative numbers,
(x + 𝑡y) · (x + 𝑡y) ≥ x · x
whence
∥x∥ 2 + 2𝑡 (x · y) + 𝑡 2 ∥y∥ 2 ≥ ∥x∥ 2
which means
2𝑡 (x · y) + 𝑡 2 ∥y| 2 ≥ 0. (∗)
For 𝑡 > 0 this yields 2x · y ≥ −𝑡 ∥y∥ 2 ,
and letting 𝑡 −→ 0+ ,
we get x · y ≥ 0. For 𝑡 < 0, (∗)
implies 2x · y ≤ −𝑡 ∥y∥ 2 , whence, letting 𝑡 −→ 0− , we get x · y ≤ 0. This gives the result.
(B) Parametrise the circle as 𝛾 : [0, 1] −→ R2 ,
𝛾 (𝑡) = (𝛾𝑥 (𝑡), 𝛾 𝑦 (𝑡)) = (cos 2𝜋𝑡, sin 2𝜋𝑡)
Then, by definition, the required line integral is
∫ 1
𝑑𝛾𝑥 (𝑡) 𝑑𝛾 𝑦 (𝑡)
{𝑣 𝑥 (cos 2𝜋𝑡, sin 2𝜋𝑡)) + 𝑣 𝑦 (cos 2𝜋𝑡, sin 2𝜋𝑡) }𝑑𝑡
0 𝑑𝑡 𝑑𝑡
∫ 1
= 2𝜋 {−(cos 2𝜋𝑡 − sin 2𝜋𝑡) sin 2𝜋𝑡 + (sin 2𝜋𝑡 + cos 2𝜋𝑡) cos 2𝜋𝑡
0
∫ 1
= 2𝜋 {sin2 2𝜋𝑡) + cos2 2𝜋𝑡 }𝑑𝑡 = 2𝜋
0

On the other hand, if 𝑣® were a gradient, this line integral would vanish, so there is no func-
tion such that 𝑣® = 𝑔𝑟𝑎𝑑 𝑓 .
(17∗ ) {1, 𝑋, 𝑋 2, . . . , } is a Q-basis of 𝑉 . Every Q-linear map from 𝑉 to Q is determined by what it does
to the above basis. I.e., 𝑉 ∗ as a set is the same as the set of functions from N to Q. Hence 𝑉 ∗
is an uncountable set, and, therefore, it cannot have a countable basis over Q. Hence 𝑉 ∗ is not
isomorphic to 𝑉 .
(18∗ ) (A) By open mapping theorem, a nhd around zero will go to a nhd around zero and hence max-
imum modulus for 𝑈 ensures positivity.
(B) The real part of 𝑓 is at most 𝐵, hence real part of 2𝐵 − 𝑓 does not vanish on the closed ball.
(C) On the boundary, |2𝐵 − 𝑓 (𝑧)| 2 = (2𝐵 − 𝑈 ) 2 + 𝑉 2 ≥ 𝑈 2 + 𝑉 2 = |𝑓 | 2 .
2
(D) Applying maximum modulus to note that for any 𝑤 in the interior,
𝑓 (𝑤) 1
| |≤
𝑤 (2𝐵 − 𝑓 (𝑤)) 𝑅
|𝑤 |
|𝑓 (𝑤)| ≤ (2𝐵 + |𝑓 (𝑤)|)
𝑅
2|𝑤 |
|𝑓 (𝑤)| ≤ 𝐵
𝑅 − |𝑤 |
For |𝑤 | = 𝑅/2, we have |𝑓 (𝑤)| ≤ 2𝐵.
(19∗ ) When 𝑛 = 1, the statement is valid since 𝑈 (1) is the group of unit complex numbers and 𝑃 equals
set 𝜇 of all the roots of unity {𝑒 2𝜋𝑖𝑡 | 𝑡 ∈ Q}.
Now consider any 𝑛 ≥ 1. The diagonal subgroup 𝑇 ⊂ 𝑈 (𝑛) equals 𝑈 (1) × · · · × 𝑈 (1) and
so 𝑃 ∩ 𝑇 = 𝜇 × · · · × 𝜇 (𝑛 factors) is dense in 𝑇 . Now let 𝑋 ∈ 𝑈 (𝑛) and let 𝑉 ⊂ 𝑋 be any
open neighbourhood of 𝑋 . There exists a unitary matrix 𝐴 such that 𝐴𝑋𝐴 −1 = 𝐴𝑋 𝐴¯𝑡 ∈ 𝑈 (𝑛) is
diagonal. Choose such an 𝐴. Now the set 𝑊 := 𝐴𝑉 𝐴¯𝑡 = {𝐴𝑌 𝐴¯𝑡 | 𝑌 ∈ 𝑉 } is an open subset of
𝑈 (𝑛). Since𝑊 ∩𝑇 is a non-empty open set in𝑇 and since 𝑃 ∩𝑇 is dense in𝑇 , there exists a finite
order element 𝐵 ∈ 𝑊 ∩ 𝑇 . It follows that 𝐴 −1 𝐵𝐴 ∈ 𝑉 has finite order.

(20 ) Since 𝐴 ⊂ R, 𝐴 has no nontrivial finite order element. Since 𝐴 is also a finitely generated abelian
group, it is isomorphic to Z𝑛 for some 𝑛 ≥ 1.
Let 𝜌 : 𝐴 −→ 𝐴 be the endomorphism of 𝐴 defined by multiplication by 𝑟 . Thus 𝜌 (𝑎) = 𝑟𝑎
and 𝜌 𝑘 (𝑎) = 𝑟 𝑘 𝑎 for all 𝑎 ∈ 𝐴 and 𝑘 ≥ 1. The matrix of 𝜌 with respect to a basis of 𝐴 has integer
entries and with determinant equal to = ±1.
Note that 𝜌 𝑘 (𝑎) = 𝑟 𝑘 𝑎 for all 𝑎 ∈ 𝐴. Suppose that 𝑃 (𝑇 ) = 𝑇 𝑛 + 𝑐 1𝑇 𝑛−1 + · · · + 𝑐𝑛 be the
characteristic polynomial of 𝜌. Then 0 = 𝑃 (𝜌)𝑎 = 𝑃 (𝑟 ) ·𝑎 for all 𝑎 ∈ 𝐴. Choosing 𝑎 ∈ 𝐴 to be non-
zero, we see that 𝑃 (𝑟 ) = 0. Since 𝑃 (𝑇 ) is monic with integer coefficients, 𝑟 is an algebraic integer.
Since the constant term of 𝑃 (𝑇 ) equals (−1)𝑛 det(𝜌) = ±1, 𝑟 −1 is also an algebraic integer. (Or
apply the same argument to 𝑟 −1 to arrive at the same conclusion.)

You might also like